MRCS Part B OSCE: Anatomy Jeremy Lynch, Susan Shelmerdine, Vishy Mahadevan
INDEX
×
Chapter Notes

Save Clear


Thorax and trunkChapter 1

 
Syllabus topics
The following topics are listed within the Intercollegiate MRCS examination syllabus for trunk and thorax anatomy. Tick them off as you revise these topics to ensure you have covered the syllabus.
Thorax
Development:
  • Heart and great vessels
  • Fetal circulation
  • Oesophagus
  • Diaphragm
Wall:
  • Thoracic wall
  • Mechanics of breathing
Thoracic cavity and viscera:
  • Superior and inferior mediastinum
  • Heart and pericardium
  • Lungs
  • Pleurae
Surface and imaging anatomy:
  • Heart & heart valves
  • Auscultation sites
  • Lungs and pleurae
  • Surface plane of sternal angle
  • Dermatomes
  • Chest drains
  • Incisions
  • Chest X-ray
  • CT/MRI
Abdomen and pelvis
Development:
  • Foregut/midgut/hindgut
  • Gut rotation
  • Anal canal
  • Kidneys and ureters
  • Bladder and urethra
  • Testis
Walls and spaces and associated structures:
  • Anterior abdominal wall
  • Posterior abdominal wall
  • Inguinal canal, spermatic cord, Inguinal hernia
  • Pelvic floor and wall
  • Lumbar plexus
  • Sacral plexus
  • Peritoneal cavity
  • Intra-abdominal spaces
Abdominal viscera:
  • Oesophagus
  • Stomach
  • Small and large intestine
  • Appendix
  • Liver
    2
  • Gall bladder
  • Bile ducts
  • Pancreas
  • Spleen
  • Kidney and ureter
  • Adrenal gland
Pelvic viscera:
  • Rectum
  • Bladder
  • Prostate
  • Seminal vesicles
  • Uterus
  • Uterine tubes
  • Ovaries
  • Vagina
Perineum:
  • Anal triangle: anal canal and ischioanal fossa
  • Male urogenital triangle: scrotum, Testis and epididymis, penis and urethra
  • Female urogenital triangle: vulva
Surface and imaging anatomy:
  • Quadrants/nine regions
  • Planes: subcostal, transpyloric, transtubercular
  • Dermatomes
  • Abdominal incisions
  • Rectal and vaginal examinations
  • Imaging appearances of abdomen/gastrointestinal/biliary/urinary tracts,
  • Arteriography
  • CT/MRI/Ultrasound
 
Station 1
A 66-year-old man is struck in the chest by a winch whilst attempting to repair his car. In the emergency department he is diagnosed with multiple right-sided rib fractures.
Image (a) below shows the inferior aspect of a right rib (demonstrating normal anatomy):
zoom view
31.1 Identify the bony landmarks labelled B, C and D.
1.1 B Costal groove
C Head
D Tubercle
1.2What does A articulate with?
1.2 The costal cartilages.
1.3 What runs in the groove indicated by B?
1.3 The neurovascular bundle accompanying the ribs runs in the subcostal groove in-between the internal and innermost intercostal muscles (the order of Veins, Arteries and Nerves from superior to inferior can be remembered by the mnemonic VAN).
1.4 What is the arterial supply of the intercostal muscles of the second and tenth intercostal spaces?
1.4 The muscles of rib spaces 1–9 are supplied by the posterior and anterior intercostal arteries, whereas those of rib spaces 10 and 11 have only posterior arteries. The posterior intercostal arteries of the first two rib spaces come from the superior intercostal branch of the costocervical trunk, whereas the posterior intercostal arteries of the bottom nine come directly off the aorta (there are only 11 rib spaces). The anterior intercostal arteries are branches of the internal thoracic arteries and its branches.
1.5 Where do the intercostal veins drain?
1.5 The anterior intercostal veins drain into the internal thoracic and musculophrenic veins. The drainage of the posterior intercostal veins is more complicated. These drain into the azygos, hemiazygos or accessory hemiazygos veins with the following exceptions: the 1st posterior vein (the supreme intercostal vein) drains into the ipsilateral brachiocephalic or vertebral veins; the left 2nd, 3rd and 4th veins join to form a superior intercostal vein, which drains into the left brachiocephalic vein.
1.6 What is meant by the term ‘flail chest’?
1.6 The term ‘flail chest’ describes a scenario in which a section of chest wall is disconnected from its surrounding bony skeleton by multiple rib fractures. This can occur unilaterally (where the ribs often fracture both at the angle and near the costochondral junction), or bilaterally (where the sternum itself can be flail).
1.7 What type of joint is the first costochondral joint?
Image (b) below shows the anterior aspect of the sternum:
zoom view
1.7 A costochondral joint is a primary cartilaginous joint, with the costal hyaline cartilage connecting directly with the rib without any intervening fibrous tissue. As with all primary cartilaginous joints virtually no movement occurs at the costochondral joint.
1.8 Identify the parts labelled A, B, C, and D.
1.8 A Manubrium
B Body
C Xiphisternum
D Jugular notch
1.9 Name the structure that articulates with the sternum at E.
1.9 1st rib
4
 
Station 2
A 54-year old woman attends the preoperative clinic in preparation for abdominal surgery. History taking reveals that she has had haemoptysis on and off for a couple of weeks. On clinical examination there is dullness to percussion over her left lower chest.
The images below are of the anterior (a) and posterior (b) aspects of the chest:
zoom view
2.1 Which lung lobes are auscultated at sites labelled A to E?
2.1 See Figure 1.1.
A Upper lobe
B Middle lobe
C Upper lobe
D Upper lobe
E Lower lobe
2.2 Describe the surface marking of the pleural edges in terms of their relations to the thoracic skeleton.
2.2 The pleural edge extends from the junction between the middle and medial thirds of the clavicle, to an apex about 2.5 cm above the medial end of the clavicle, and then down to the sternoclavicular joint.
zoom view
Fig. 1.1: (a) Anterior thoracic wall and lobes of the lung. (b) Posterior thoracic wall and lobes of the lung.
It then meets the pleura of the contralateral side in the midline at the level of the 2nd costal cartilage. The right pleural edge extends down to the level of the 6th costal cartilage. It then turns laterally crossing the 8th rib in the midclavicular line and the 10th rib in the midaxillary line; it then meets the 12th rib at the lateral border of erector spinae. On the left side the heart, at the level of the 4th costal cartilage, reflects the pleura laterally but otherwise the pleural reflexion follows a similar course to that on the right side.
2.3 Describe the surface marking of the lung edges in terms of their relations to the thoracic skeleton.
2.3 The apex of the lung follows the pleura. The lower border of the lung is parallel to the line of pleural reflexion but two ribs above: thus the lower border of the lung crosses the 6th rib in midclavicular line, the 8th rib in the midaxillary line, and the 10th rib adjacent to the vertebral column posteriorly.
2.4 What are the surface markings for:
2.4a the oblique fissure of the lungs?
2.4a The oblique fissure divides the upper and lower lobes. For the most part, it corresponds to the line of the 5th rib. It may be indicated on the surface as a line running obliquely downwards and outwards from just lateral to the spine of the 3rd thoracic vertebra to the 6th costal cartilage 4 cm from the midline. With the shoulders abducted fully this line corresponds to the medial border of the scapula.
2.4b the transverse fissure of the lungs?
2.4 b The horizontal (transverse) fissure divides the middle from the upper lobe of the right lung. It follows a line along the 4th costal cartilage to meet the oblique fissure where it crosses the 5th rib near the midaxillary line.
2.5 Which costal cartilage attaches to the sternum at the sternal angle?
2.5 The 2nd costal cartilage articulates with the lateral aspect of the manubriosternal junction.
2.6 At which vertebral level is the suprasternal notch?
2.6 The suprasternal notch lies at the level of the T2/T3 intervertebral disc.
2.7 At which vertebral level is the xiphisternal joint?
2.7 The xiphisternal joint lies at the level of T9.
2.8 What are the boundaries of the superior mediastinum?
2.8 The superior mediastinum is bounded anteriorly by the manubrium, laterally by the pleurae, posteriorly by the T1–T4 vertebral bodies, superiorly by the superior thoracic aperture, and inferiorly by the plane of Louis (transverse thoracic plane at the T4/T5 intervertebral disc).
2.9 What are the contents of the superior mediastinum?
2.9 The superior mediastinum contains the great vessels (aortic arch, brachiocephalic artery and veins, left common carotid and subclavian arteries, superior vena cava), trachea, oesophagus, remains of the thymus, thoracic duct, right and left vagi, left recurrent laryngeal nerve and right and left phrenic nerves, and very importantly, lymph nodes.
5
 
Station 3
A 22-year-old man is brought to the emergency department after being hit by a car. He is intubated by the paramedics at the scene of the accident. On arrival in the emergency department it is noted that he has extensive bruising over his lower chest. Review your knowledge of intrathoracic anatomy using the following image.
This is a contrast-enhanced axial computed tomography (CT) scan of a normal thorax:
zoom view
3.1 Identify the structures labelled A, B, C.
3.1 A Ascending aorta
B Descending aorta
C Pulmonary trunk
3.2 Classify the divisions of the mediastinum.
3.2 The mediastinum is the space in the thoracic cavity between the right and left pleural sacs. It is conventionally divided into a superior mediastinum and inferior mediastinum by an imaginary plane plotted perpendicular to the sternum at the plane of Louis (Figure 1.2). The inferior mediastinum is further subdivided into anterior, middle, and posterior mediastina by the fibrous pericardium.
zoom view
Fig. 1.2: Divisions of the mediastinum.
3.3 Through which division of the mediastinum is this slice taken?
3.3 It shows the inferior mediastinum. This requires knowledge of the contents of each division of the mediastinum.
  • The superior mediastinum contains the great vessels (aortic arch, brachiocephalic artery and veins, left common carotid and subclavian arteries, superior vena cava), lymph nodes, trachea, oesophagus, remains of the thymus, thoracic duct, vagi, left recurrent laryngeal nerve and phrenic nerves.
  • The anterior mediastinum contains the remains of the thymus and branches of the right and left internal thoracic arteries.
  • The middle mediastinum contains the heart inside the pericardium, the ascending aorta, the superior vena cavae, the bifurcation of the trachea, the pulmonary arteries and veins, the phrenic nerves and pericardiophrenic vessels.
  • The posterior mediastinum contains the descending thoracic aorta and its branches, the azygos/hemiazygos/accessory hemiazygos veins, the right and left vagus nerves, the right and left splanchnic nerves, the oesophagus, and the thoracic duct. The ganglionated thoracic sympathetic chains may also be regarded as contents of the posterior mediastinum.
3.4 What are the boundaries of this division of the mediastinum?
3.4 This, the inferior mediastinum, is bounded anteriorly by the body of the sternum (mesosternum), laterally by the pleurae, posteriorly by the T5–T12 vertebral bodies, inferiorly by the diaphragm, and superiorly by the plane of Louis.
3.5 What is the innervation of the parietal and visceral pleura?
3.5 The parietal pleura are sensitive to pain, temperature, touch and pressure. Its innervation depends on region: intercostal nerves supply the costal pleura; the phrenic nerve supplies the mediastinal pleura; and the phrenic nerve and lower six intercostal nerves supply the diaphragmatic pleura. The visceral pleura are sensitive to stretch and receive their sensory innervation from the autonomic pulmonary plexus (formed from branches of the thoracic sympathetic trunk and vagus nerve).
3.6 Name some of the important functions of the thoracic sympathetic chain, and state which spinal cord segments contribute to the sympathetic chain?
3.6 The thoracic sympathetic chain has three main branches. It supplies sympathetic fibres to the skin, postganglionic fibres from T1–T5 to the thoracic viscera, and mostly preganglionic fibres from T5–T12 to supply the abdominal viscera (in the form of the greater splanchnic, lesser splanchnic and least splanchnic nerves). The chain receives preganglionic white ramus communicans from each spinal nerve to a corresponding ganglion, and gives back a grey ramus containing postganglionic fibres.
3.7 Define thoracic outlet syndrome.
3.7 Thoracic outlet syndrome is caused by compression at the superior thoracic aperture of neurovascular structures passing above the first rib, either between the anterior and middle scalene muscles or in front of scalenus anterior. It can affect the brachial plexus (most commonly, lower trunk of the brachial plexus) or subclavian artery/vein. A rare cause of thoracic outlet syndrome is a cervical rib or a cervical band of fibrous tissue. The syndrome manifests most commonly in the hands with pain, weakness, and coldness. Note that the superior thoracic aperture is usually referred to by clinicians as the thoracic outlet but by anatomists as the thoracic inlet!
3.8 Define subclavian steal syndrome.
3.8 Subclavian steal occurs when blood flows in a retrograde direction in the vertebral artery in association with proximal ipsilateral subclavian artery stenosis or occlusion. This blood is ‘stolen’ from the circle of Willis via the ipsilateral vertebral artery. Patients with retrograde flow are usually asymptomatic but they may develop dizziness, vertigo, syncope, dysarthria, and visual symptoms. There is usually a drop in blood pressure in the ipsilateral arm distal to the stenosis.
 
Station 4
A 55-year-old male banker experiences a crushing type of retrosternal chest pain of sudden onset whilst climbing the stairs in his office. An echocardiogram shows severe aortic stenosis.
6The image below is of an axial cardiac CT taken at the level of the aortic root (demonstrating normal anatomy):
zoom view
4.1 Identify the structures labelled A to E.
4.1 A Right coronary artery
B Left atrium
C Descending thoracic aorta
D Left anterior descending artery (anterior interventricular artery)
E Left circumflex artery
4.2 What is the origin of the right coronary artery? Name the branches of the right coronary artery.
4.2 The right coronary artery (Figure 1.3) originates from the anterior aortic sinus of the ascending aorta. It has the following branches:
  • The anterior ventricular branches supply the anterior surface of the right ventricle.
    zoom view
    Fig. 1.3: Arterial supply of the heart.
  • The marginal artery is a branch of the anterior ventricular and runs towards the apex.
  • The posterior ventricular branches supply the diaphragmatic surfaces of the right ventricle.
  • The posterior interventricular artery supplies the right and left ventricles and runs in the posterior interventricular groove.
  • The atrial branches supply the right atrium.
4.3 What is the origin of the left coronary artery? Name the branches of the left coronary artery.
4.3 The left coronary artery originates from the left posterior aortic sinus of the ascending aorta. Its branches are:
  • The anterior interventricular artery (also known as left anterior descending artery). This is the major branch of the left coronary artery and supplies the anterior aspect of both ventricles and the anterior half of the interventricular septum before proceeding to anastomose with the posterior interventricular branch of the right coronary artery.
  • The circumflex artery is the continuation of the left coronary artery after the anterior interventricular is given off. It winds around the left margin of the heart in the atrioventricular groove.
  • The left marginal artery is a branch of the circumflex.
  • The anterior/posterior ventricular arteries are branches of the circumflex.
  • The atrial branches are also branches of the circumflex.
4.4 Which coronary artery most commonly supplies:
4.4a the sinoatrial node?
4.4a The sinoatrial node is supplied by the right coronary artery in about two-thirds of people, and by the left in approximately one-third.
4.4b the atrioventricular node?
4.4b The atrioventricular node is supplied by the posterior interventricular branch of the right coronary artery in over 90% of individuals and less commonly by the left coronary artery via its circumflex branch.
4.5 Describe the venous drainage of the heart.
4.5 The coronary veins drain in to the coronary sinus, which is located in the posterior atrioventricular groove. The coronary sinus drains into the right atrium. The major veins it receives are the great cardiac vein (in the anterior interventricular groove), the middle cardiac vein (in the posterior interventricular groove), the small cardiac vein (running along the lower border of the heart), and the cardiac oblique vein (on the posterior surface of the left atrium). The anterior cardiac veins drain the anterior surface of the heart and empty anteriorly into the right atrium directly. There are numerous much smaller veins emptying directly into the chambers that they overlie.
4.6 Describe the conducting system of the heart.
4.6 The sinoatrial node is located in the wall of the right atrium, to the right of the opening of the superior vena cava. It generates rhythmic electrical impulses that radiate out throughout the atrial muscle, causing contraction. The atrioventricular node is in the ventricular end of the atrial septum and conducts the atrial impulse to the ventricles, via the atrioventricular bundle of His. The time that it takes for this conduction (about one tenth of a second) allows the atria to empty their blood into the ventricles. The bundle of His divides into two branches, one for each ventricle. The right bundle travels down on the right side of the interventricular septum to reach the anterior wall of the right ventricle and becomes continuous with the Purkinje plexus of the right ventricle. The left bundle divides into anterior and posterior branches, and these fibres are continuous with the Purkinje plexus of the left ventricle. These fibres induce contraction of the ventricles.
4.7 Describe the efferent nerve supply to the heart.
4.7 The parasympathetic supply of the heart is the vagus nerve. The heart's sympathetic supply is the cervical and upper thoracic sympathetic trunk. The cardiac plexuses are located below the arch of the aorta and transmit all of the heart's autonomic fibres. Sympathetic stimulation increases the force and rate of contraction and dilates the coronary arteries. Parasympathetic stimulation decreases the force and rate of contraction and constricts the coronary arteries.
4.8 Why is the pain of ischaemic heart disease referred to the chest wall?
4.8 Afferent fibres from the heart run with sympathetic fibres and enter the spinal cord through the posterior roots of T1–T4. The pain of ischaemia is referred to the skin areas of the skin supplied by the corresponding spinal nerves, i.e. the upper four intercostal nerves and the intercostobrachial nerve. This territory is the left chest wall and the upper part of the left arm.
 
Station 5
A 29-year-old woman is involved in a head-on collision with another car. She is brought to the emergency department where it is noted that she is restless, tachypnoeic and tachycardic. Clinical examination reveals she has a large right-sided tension pneumothorax and you are asked to perform emergency decompression.
7This image displays the anterior aspect of the female chest:
zoom view
5.1 Where would you insert a cannula for emergency decompression of tension pneumothorax? What layers does this needle pass through?
5.1 Tension pneumothorax requires emergency decompression with a 14–16 gauge needle in the 2nd intercostal space in the midclavicular line. The needle passes through skin, superficial fascia and fat, pectoralis major, external intercostal, internal intercostal, innermost intercostal, and parietal pleura.
5.2 What is the ‘safe triangle’ of chest drain insertion?
5.2 The safe triangle is made up of the lateral border of the pectoralis major, the anterior border of latissimus dorsi, and the upper border of the 6th rib (about the level of the nipple), with the apex slightly below the axilla.
5.3 The point marked B is located within which dermatome?
5.3 T6 or T7 (Figure 1.4)
zoom view
Fig. 1.4: Trunk dermatomes.
5.4 What is the direction of relaxed skin tension lines at point A?
5.4 At this point the lines are not horizontal, but are actually directed towards the axillary skin crease.
5.5 What is the surface marking for:
5.5a the entry point of the needle for subclavian vein catheterisation?
5.5a The target is 1 cm inferior to the junction of the middle and distal third of the clavicle. The tip should be directed towards the sternal notch.
5.5b the entry point of the needle for internal jugular vein catheterisation?
5.5b With the patient's head turned away from the insertion site, the target is the apex of the triangle formed by the sternal and clavicular heads of the sternocleidomastoid muscle. The needle should be inserted at a 30° angle to the skin directed towards the ipsilateral nipple.
5.5c a posterolateral thoracotomy incision for exposure of upper thoracic structures?
5.5c The site is somewhat dependent on what access is necessary, but the incision usually lies at the level of the 5th rib for upper thoracic structures (for exposure of lower structures the incision is at the 6th or 7th rib). The incision is started at a point midway between the medial border of the scapula and the thoracic spine. The incision curves about 3 cm below the inferior angle of the scapula and turns to run parallel with the rib.
5.6 At which vertebral level is:
5.6a the sternal angle (plane of Louis)?
5.6a The intervertebral disc between T4 and T5.
5.6b the bifurcation of the trachea?
5.6b The normal level is the plane of Louis, but in full inspiration the level is the T6 vertebra.
5.7 What are the surface markings for auscultation of the aortic and pulmonary valves?
5.7 The aortic valve is auscultated at the 2nd intercostal space, right upper sternal border (Figure 1.5). The pulmonary valve is heard at the 2nd intercostal space, left upper sternal border. The mitral valve is heard at the 5th intercostal space, left midclavicular line. The tricuspid valve is heard at the 4th intercostal space, lower left sternal border. Note that these sites do not actually overlie the valves themselves.
5.8 What are the surface markings for the borders of the heart?
5.8 The left border of the heart is from the 2nd costal cartilage, left of the sternum, to the 5th left intercostal space, midclavicular line (Figure 1.5). The right border of the heart is from the 3rd right costal cartilage, right of the sternum, to the 6th right costal cartilage, right of the sternum.
zoom view
Fig. 1.5: Borders and auscultation areas of the heart. A, aortic valve; P, pulmonary valve; T, tricuspid valve; M, mitral valve.
Table 1.1   Surface anatomy landmarks of the thorax
Landmark
Location and significance
Midaxillary line
Vertical line intersecting a point midway between the anterior and posterior axillary folds
Midclavicular line
Vertical line passing through the midshaft of the clavicle
Nipple
Superficial to the 4th intercostal space in the male and prepuberal female Usually within the T4 dermatome
Sternal angle
Junction of the manubrium and body of the sternum Attachment of the 2nd costal cartilage rib to the sternum, the T4/T5 intervertebral disc, the inferior boundary of the superior mediastinum
Suprasternal notch
Curved superior border of the manubrium
 
Station 6
Two days post-nephrectomy, a 59-year-old man is noted by the ward staff to have become suddenly very breathless. An emergency chest radiograph is requested and while you are waiting for the radiographs you wish to view a normal chest film to revise your knowledge of thoracic anatomy.
8This is an anteroposterior plain radiograph of the chest (demonstrating normal anatomy):
zoom view
6.1 Identify the structures labelled A to D.
6.1 A Right atrium
B Arch of aorta
C Left lung hilum
D Left ventricle
6.2 Which fissures do lines 1–3 indicate?
6.2 1 Right horizontal or transverse
2 Right oblique
3 Left oblique
6.3 At what vertebral level do the bronchi enter the lungs?
6.3 The right main bronchus is about 2.5 cm long and enters the hilum of the lung at T5. It gives off an upper lobe branch before reaching the hilum. The left main bronchus is about 5 cm, and passes downwards and laterally below the arch of the aorta, anterior to the oesophagus and descending aorta. It enters the hilum of the lung at T6.
6.4 Name the lobes of the right and left lungs.
6.4 The right lung is divided by the oblique and horizontal fissures in to upper, middle, and lower lobes. The left lung is divided by an oblique fissure into upper and lower lobes. The lingula (Latin: ‘little tongue’) of the left upper lobe is composed of two bronchopulmonary segments that are analogous to the right middle lobe.
6.5 Define the term ‘bronchopulmonary segment’.
6.5 A bronchopulmonary segment is a discrete anatomical and functional unit of the lung that can be removed without disturbing the function of the other segments. They are pyramid shaped with their apices at the hilum. Each is served by its own tertiary bronchi, vein, artery, and lymph and has its own autonomic nerve supply.
6.6 How many bronchopulmonary segments are there in each lung?
6.6 Each lung has 10 bronchopulmonary segments. It would be unlikely that you would need to recite them in the exam, but they are listed here for reference purposes (Table 1.2):
Table 1.2   Bronchopulmonary segments of the lung
Right
Left
Upper lobe
Upper lobe
1. Apical
1. Apical
2. Posterior
2. Posterior
3. Anterior
3. Anterior
Middle lobe
Lingula
4. Lateral
4. Superior
5. Medial
5. Inferior
Lower lobe
Lower lobe
6. Superior (apical)
6. Superior (apical)
7. Medial basal (cardiac)
7. Medial basal
8. Anterior basal
8. Anterior basal
9. Lateral basal
9. Lateral basal
10. Posterior basal
10. Posterior basal
6.7 What is the blood supply to the lungs?
6.7 The bronchial arteries supply oxygenated blood to the bronchial and connective tissue of the lungs. The left superior and inferior bronchial arteries arise from the thoracic aorta, whereas the single right bronchial artery has a variable origin (either the aorta, the left superior bronchial artery, or the right intercostal arteries). The bronchial veins drain into the azygous and hemiazygos veins. The pulmonary arteries supply deoxygenated blood to the alveoli via their terminal branches, and the superior and inferior pulmonary veins drain oxygenated blood to the left atrium.
6.8 What is the lymphatic drainage of the lungs?
6.8 The superficial lymphatic plexus lies under the visceral pleura and drains the surface of the lungs towards the hilum, whereas the deep plexus drains along the blood vessels towards the hilum. Lymph passes from the bronchopulmonary lymph nodes at the hilum to the tracheobronchial nodes at the bifurcation of the trachea, and thence to bronchomediastinal lymph trunks.
6.9 Describe the nerve supply to the lungs? What effect does stimulation of the sympathetic and parasympathetic system have on the lungs?
6.9 The pulmonary plexus at the hilum of the lung receives afferent autonomic nerve fibres from the mucous membranes of the bronchioles and alveoli stretch receptors, and serves efferent fibres to the bronchial musculature. Sympathetic fibres cause bronchodilatation and vasoconstriction, whereas parasympathetic fibres cause bronchoconstriction, vasodilatation, and glandular secretion.
6.10 In which bronchus (right or left) are inadvertently aspirated foreign bodies most likely to lodge and why?
6.10 Foreign objects are aspirated more commonly in the right bronchus because it is wider and has a steeper angle than the left.
 
Station 7
A 72-year-old man is hit by a car. He is noted to be in haemodynamic shock when reviewed in the emergency department. There is suspicion of intrathoracic bleeding and the man is taken to theatre.
9The image below is a dissection of the thorax and mediastinum viewed from the right:
zoom view
7.1 Identify the structures labelled A to G.
7.1 A A right intercostal artery and vein
B Right sympathetic chain
C Right phrenic nerve
D Superior vena cava
E Right principal bronchus
F Right pulmonary vein
G Pericardial sac (over right atrium)
7.2 What are the contents of the pulmonary hilum?
7.2 The pulmonary hilum contains: the pulmonary artery, the pulmonary vein, the main bronchus, the bronchial arteries and veins, lymph nodes, and autonomic nerves.
7.3 How many pulmonary veins drain each lung?
7.3 Two veins drain each lung (so there are four in total).
7.4 How many embryonic pharyngeal arches are there and which numbered arch disappears without giving rise to any specific structures?
7.4 There are six but the fifth exists only transiently, and no human structures are derived from it.
7.5 From which embryonic pharyngeal arch are the internal carotid arteries derived?
7.5 The 3rd arch.
7.6 From which embryonic pharyngeal arch is the right subclavian artery derived?
7.6 The 4th arch.
 
Station 8
During the preoperative assessment of a 65-year-old man due to have general anaesthesia, a systolic murmur is detected. The man undergoes an echocardiogram which reveals severe tricuspid regurgitation. You discuss the patient with the cardiothoracic surgeons.
10This is a prosection of the heart, displaying the inside of the right ventricle (demonstrating normal anatomy):
zoom view
8.1 Identify the structures labelled A to G.
8.1 A Ascending aorta
B Auricle of right atrium
C Pectinate muscles on right ventricular wall
D Chordae tendineae
E Pulmonary trunk
F Auricle of left atrium
G Anterior interventricular branch of left coronary artery
8.2 Identify the structure labelled H? What is its function?
8.2 H Papillary muscles. These attach to the cusps of the atrioventricular valves (in this case the tricuspid valve) to prevent prolapse.
8.3 Name the layers of pericardium.
8.3 The pericardium is divided in to fibrous and serous layers, the latter of which is subdivided into parietal and visceral layers. The fibrous pericardium is a tough layer fused with the central tendon of the diaphragm and the outer coats of the great vessels. It is attached to the sternum via the sternopericardial ligaments. The parietal pericardium lines the inner surface of the fibrous pericardium and is reflected around the great vessels to become continuous with the visceral pericardium that lines the heart.
8.4 Where is the pericardial space and what does it normally contain? What layers, from skin downwards, does a needle pass through to perform pericardiocentesis?
8.4 The pericardial space exists between the parietal and visceral layers, and is filled with about 50 mL of pericardial fluid.
In the subcostal approach the needle is positioned left of the xiphoid process with the needle angulated upwards at 45° to the skin. The needle passes through skin, superficial/deep fascia, the anterior layer of the rectus sheath, rectus abdominis, the posterior layer of the rectus sheath, diaphragm, endothoracic fascia, fibrous pericardium, and the parietal layer of serous pericardium.
In the parasternal approach the needle is placed at the 5th intercostal space near the left sternal margin. The needle passes through skin, superficial/deep fascia, pectoralis major muscle, intercostal muscles, transversus thoracis muscle, endothoracic fascia, fibrous pericardium, and the parietal layer of serous pericardium.
8.5 What structures are at risk of damage during pericardiocentesis?
8.5 In the subcostal approach, the main risk is of puncturing the liver if the needle is angulated too inferiorly. In the parasternal approach, the main risk is of puncturing the lungs. In both approaches, there is the risk of damage to the coronary arteries and atrial/ventricular walls.
8.6 What is the function and composition of the foramen ovale? What percentage of adults has an anatomically patent foramen ovale?
8.6 In the fetus, the foramen ovale allows oxygenated blood from the umbilical vein (via the inferior vena cava) to flow from the right to left atrium. It is composed of the septum primum and septum secundum. These are forced together at birth due to pressure changes resulting from expansion of the lungs, and usually fuse at about 3 months. In 10% of people fusion is incomplete.
8.7 What structures do the ductus arteriosus connect in the fetus?
8.7 The ductus arteriosus is a vascular shunt in the fetus connecting the pulmonary artery to the descending thoracic aorta, allowing blood from the right ventricle to bypass the lungs (which are non-functioning at this stage).
8.8 Where is the commonest site for coarctation of the aorta?
8.8 Aortic coarctation is associated with a patent ductus arteriosus, and occurs in the area where the ductus arteriosus inserts. Narrowing can be preductal, ductal, or postductal. It usually occurs distal to the origin of the left subclavian artery.
8.9 What do the bulbus cordis and truncus arteriosus give rise to in the adult?
8.9 The bulbus cordis and the primitive ventricle give rise to the ventricles of the heart. The cranial end of bulbus cordis and the truncus arteriosus give rise to the aorta and pulmonary trunk.
 
Station 9
A 45-year-old man is involved in a high-speed road collision. He is intubated at the scene of the accident by the paramedic team, and bilateral chest drains are inserted. On arrival in the emergency department the man remains very hypoxic and the chest drains are continuing to drain large volumes of blood. You assist with a thoracotomy performed in the resuscitation room to identify the source of bleeding.
11The following image is an axial dissection of the thorax at the level of T9 (demonstrating normal anatomy), viewed from below:
zoom view
9.1 Identify the structures labelled A to E.
9.1 A Right oblique fissure of the right lung
B Left ventricle
C Right latissimus dorsi muscle
D Descending thoracic aorta
E Left trapezius
9.2 List the successive elements of the airway, starting with the trachea and leading on to the alveolus.
9.2 The trachea splits into the left and right principle bronchi and thence to lobar (secondary bronchi), and segmentary (tertiary) bronchi. After entering the bronchopulmonary segment the bronchi undergoes successive branching into smaller and smaller tubes until they give rise to bronchioles. These are less than 1 mm in diameter and contain no cartilage. Bronchioles divide into terminal bronchioles, which give rise to respiratory bronchioles in their walls. These structures end in alveolar ducts that lead to alveolar sacs, across which gas exchange takes place with the surrounding capillaries.
9.3 State the origin and termination of the azygos vein.
9.3 The azygos vein forms about the level of the right renal vein from either a posterior tributary of the inferior vena cava or from the junction of the right ascending lumbar and right subcostal veins. It traverses the aortic opening of the diaphragm and lies to the right of the vertebra, behind the oesophagus. The vein ends by running anteriorly over the hilum of the right lung to enter the superior vena cava at T4. The azygos vein has tributaries of: the lower eight right posterior intercostal veins, the right superior intercostal vein, bronchial and oesophageal veins, and the accessory azygos/hemiazygos veins.
9.4 What tributaries do the hemiazygos and accessory veins receive, and where do they drain to?
9.4 The hemiazygos vein drains the four lower left posterior intercostal veins. It arises from the left ascending lumbar, the left subcostal, and often the left renal veins.
The accessory hemiazygos vein drains the 5–8th left posterior intercostal veins, and has tributaries from the bronchial and mid-oesophageal veins.
9.5 Describe the arrangement of muscles in the intercostal spaces.
9.5 There are three layers of muscle in the intercostal space (Figure 1.6). The external intercostal muscle forms the outermost layer; its fibres are directed forwards and downwards from the inferior border of the rib above to the superior border of the rib below. The internal intercostal muscle is the intermediate layer; its fibres are directed downwards and backwards from the subcostal groove of the rib above to the upper border of the rib below. The deepest layer is the innermost intercostal muscle (really composed of a group of three muscles). These muscles cross more than one intercostal space.
9.6 Which muscles contract to produce inspiration?
9.6 The external intercostal muscles aid in forced and passive inspiration. The internal intercostal muscles aid in forced expiration. Passive expiration is achieved by relaxation of the muscles and the elastic recoil of the lungs.
9.7 Name the accessory muscles of respiration.
9.7 The accessory muscles of respiration help to increase the thoracic capacity in deep inspiration. They are the sternocleidomastoid, scalenus anterior and medius, serratus anterior and pectoralis major and minor.
zoom view
Fig. 1.6: The layers of the chest wall.
 
Station 10
A 52-year-old man presents to the surgical clinic with progressive dysphagia for both liquids and solids over the last few months. He confesses to have lost a significant amount of weight recently and admits to being a heavy drinker and smoker.
He undergoes the following investigation (demonstrating normal anatomy):
12
zoom view
10.1 What type of radiological investigation is this?
10.1 Barium swallow
10.2 Name the parts of the gastrointestinal tract indicated by A, B and C.
10.2 A Lower oesophagus
B Stomach
C Also stomach
10.3 Comment on the narrowing at point D.
10.3 D This narrowing represents the site of the lower oesophageal sphincter.
10.4 Oesophogastroduodenoscopy (OGD) is performed. State in centimetres the distance from the incisor teeth:
10.4a at which the oesophagus commences.
10.4a 17 cm
10.4b at which the oesophagus is crossed by the left bronchus.
10.4b 28 cm
10.4c at which the oesophagus terminates.
10.4c 43 cm
10.5 Name the layers an endoscope must pass through to perforate the oesophagus.
10.5 To perforate the oesophagus one must pass through first mucosa, submucosa, a muscular layer (the composition of which depends on level), and an outer connective tissue layer (areolar tissue). The thoracic oesophagus has no serosa.
10.6 What type of muscle fibre lies in the oesophageal wall, and what cell type lines the oesophageal mucosa?
10.6 The oesophagus has outer longitudinal and internal circular muscular layers. The muscle fibres of the upper two-thirds of the oesophagus are striated (and hence under voluntary control), and the lower one-third is smooth. In health, it is lined by squamous epithelium.
10.7 What is the blood supply of the oesophagus?
10.7 The blood supply to the oesophagus, like most long tubes, is segmental. The upper third is supplied by the inferior thyroid artery and vein, the middle third by descending aortic branches and veins to the azygos, and the lower third by the left gastric artery and vein (portal system). Note that there is anastomosis between the portal and systemic systems; in portal hypertension these veins distend in to oesophageal varices that can cause life-threatening haemorrhage.
10.8 What is meant by the term ‘Barrett's oesophagus’?
10.8 Barrett's oesophagus is metaplasia of the squamous epithelium of the lower oesophagus into columnar epithelium. It is thought to be an adaption to chronic acid exposure from gastro-oesophageal reflux. There is a strong association with adenocarcinoma of the oesophagus.
10.9 What is the lymphatic drainage of the oesophagus?
10.9 Lymph drainage follows arterial supply. The upper third of the oesophagus drains into the deep cervical nodes, the middle third into the superior and posterior mediastinal nodes, and the lower third in to coeliac nodes.
10.10 What is ‘Virchow's node’ and why does it commonly present on the left side?
10.10 Virchow's node is an enlarged lymph node in the left supraclavicular fossa (the associated sign is called Troisier's sign). It is associated with gastric and other intra-abdominal cancer. The lymph node is on the left side because the majority of lymph drains via the thoracic duct in to the left subclavian vein. Metastases block the thoracic duct causing reflux in to the surrounding nodes.
10.11 Where is the cisterna chyli and what is its function?
10.11 The cisterna chyli is a dilated sac at the lower end of the thoracic duct that is the common pathway of drainage of lymph and chyle from the abdomen and lower limbs. It is usually positioned between the abdominal aorta and right crus of the diaphragm. Its position and existence are, however, inconsistent.
10.12 What is the embryological origin of the trachea?
10.12 The trachea develops from the floor of the foregut. Initially the laryngotracheal groove appears, which later becomes a tube. Buds appear on either side of the tube and develop into the lungs. The shared origin of the trachea and oesophagus explains the association of tracheoesophageal fistula with oesophageal atresia.
13
 
Station 11
A 63-year-old woman presents acutely with generalised abdominal pain and constipation. On examination the abdomen is distended and a mass is felt in the right lower quadrant.
This image demonstrates some features of the anterior aspect of the abdomen:
zoom view
11.1 With your knowledge of anatomy, give a differential diagnosis of a mass located in the region indicated by circle B.
11.1 Skin/soft tissue: sebaceous cyst, lipoma
Muscle: sarcoma, psoas abscess
Bowel: appendix abscess/mass, Crohn's, carcinoma, tuberculosis
Gynaecological: ovarian tumour, fibroids
Urological: pelvic kidney, bladder diverticulum
Vascular: aneurysm of the external or common iliac artery, enlarged iliac lymph node
11.2 Define McBurney's point.
11.2 McBurney's point (Figure 1.7) is the typical location of the appendix, and is located at a point two thirds from the umbilicus to the anterior superior iliac spine.
11.3 In which dermatome is the umbilicus located?
11.3 T10 (see Figure 1.4).
zoom view
Fig. 1.7: Abdominal incisions.
11.4 Name the sequence of layers you would pass through whilst incising through lines A and C to enter the abdomen?
11.4 A Skin, subcutaneous fat, Scarpa's fascia, external oblique muscle, internal oblique muscle, transversalis fascia, extraperitoneal fat, parietal peritoneum.
C Skin, subcutaneous fat, Scarpa's fascia, linea alba, transversalis fascia, extraperitoneal fat, parietal peritoneum.
11.5 What is the surface marking of the transpyloric plane and what structures are present at this level?
11.5 The transpyloric plane (Figure 1.8) is located halfway between the suprasternal notch and the pubic symphysis, at the level of the L1 vertebral body. At this level lie the following structures: the pylorus of stomach, fundus of gallbladder, pancreatic neck, duodenojejunal flexure (and first part of duodenum), spinal cord termination, line of attachment of transverse mesocolon, left renal hilum, origin of the superior mesenteric artery, origin of portal vein.
11.6 At what vertebral level is the subcostal plane and what structures are present at this level?
11.6 The subcostal plane (Figure 1.8) is the line parallel to the lowest part of the thoracic cage. The inferior mesenteric artery and L3 vertebra are present at this level.
11.7 At what vertebral level does the aorta bifurcate? What is the surface marking for this point?
11.7 The aorta bifurcates at the L4 vertebral level, which is usually about the level of the umbilicus. This is also the level of the supracristal plane (Figure 1.8), which is a line joining the most superior parts of the iliac crests.
11.8 What are the surface markings for:
11.8a the inferior border of the liver?
11.8a The inferior border of the liver extends from the tip of the right 10th rib to the left 5th intercostal space medial to the midclavicular line. The superior border is at the level of the 5th intercostal space.
zoom view
Fig. 1.8: Lines and planes of the abdomen.
11.8b the spleen?
11.8b The spleen lies under the 9–11th ribs on the left side. The long axis of the spleen lies along the 10th rib, the posterior pole being just to the left of the vertebral column, and the anterior pole is in the midaxillary line.
11.8c the fundus of the gallbladder?
11.8c The fundus of the gallbladder is at the point at which the rectus abdominis intersects the costal margin, at the tip of the 9th costal cartilage.
11.9 Describe the location of these abdominal incisions:
11.9 See Figure 1.7 and Table 1.3.
Table 1.3   Abdominal surface anatomy landmarks
Landmark
Location
Significance
Arcuate line (Douglas’ line)
Approximately one-third of the distance from the umbilicus to the pubic symphysis
Lower limit of the posterior sheath
Deep inguinal ring
Midway between anterior superior iliac spine and pubic tubercle
Opening in the transversalis fascia for the vas deferens and gonadal vessels (or round ligament in the female)
Linea alba
Midline aponeurotic band extending from xiphoid process to the pubic symphysis
Formed by the combined abdominal muscle aponeuroses.
McBurney's point
Two-thirds from the umbilicus to the anterior superior iliac spine
Typical location of the appendix
Mid-inguinal point
Midway between the anterior superior iliac spine and the pubic symphysis
Location of the femoral artery
Semilunar line
The lateral edge of the rectus abdominis muscle
Formed by the combined aponeuroses of the abdominal wall muscles at the lateral margin of the rectus sheath.
Subcostal plane
Line parallel to the lowest part of the thoracic cage
Origin of inferior mesenteric artery
Supracristal plane
Horizontal plane at the upper margin of the iliac crests
L4 Bifurcation of the aorta
Transpyloric line
Half the distance between the jugular notch and the pubic crest
  • Pylorus of stomach
  • Fundus of gallbladder
  • Pancreatic neck
  • Duodenojejunal flexure (and first part of duodenum)
  • Spinal cord termination
  • Line of attachment of transverse mesocolon
  • Left renal hilum
  • Origin of the superior mesenteric artery
  • Origin of portal vein
Umbilicus
Within T10, approximately at level of L4 vertebra
11.9a Gridiron
11.9a Gridiron: A 2.5–5 cm oblique incision at McBurney's point, perpendicular to a line running from the anterior superior iliac spine to the umbilicus. Used for appendicectomy.
1411.9b Kocher
11.9b Kocher: an oblique incision below and parallel to the right costal margin (starting below the xiphoid process). Used for access to biliary structures.
11.9c Mercedes Benz
11.9c Mercedes Benz: bilateral low Kocher's incisions with an upper midline limb up and through the xiphisternum. For access to upper abdominal viscera. Also known as a roof-top incision.
11.9d Pfannenstiel
11.9d Pfannenstiel: A transverse lower abdominal incision centered above the pubic symphysis, slightly upturned at the ends (a ‘smile’ incision).
 
Station 12
A 49-year-old woman presents with epigastric pain and weight loss over a period of 4 months. At the last clinic attendance an abdominal CT scan was requested. Before you review her scans you wish to familiarise yourself with the features of a normal abdominal scan.
This is an axial CT scan of the upper abdomen (a) (demonstrating normal anatomy):
zoom view
12.1 Identify the structures labelled A to G.
12.1 A Inferior vena cava
B Erector spinae muscle
C Portal vein
D Right crus of diaphragm
E Superior mesenteric artery
F Fundus of the stomach
G Aorta (abdominal)
12.2 At what vertebral level is the above CT slice taken?
12.2 At the level of the superior mesenteric artery, below the level of the spleen.
12.3 What is C and what are its tributaries?
12.3 The portal vein (C) drains the gastrointestinal tract and associated viscera. It is formed from the splenic and superior mesenteric veins as they unite behind the neck of the pancreas. The splenic vein receives the short gastric, left gastroepiploic, inferior mesenteric, and pancreatic veins. The superior mesenteric vein receives the jejunal, ilial, ileocolic, right colic, middle colic, inferior pancreaticoduodenal and right gastroepiploic veins. There are three other direct tributaries of the portal vein: the left gastric, right gastric, and cystic veins.
12.4 What proportion of blood does C supply to the liver?
The image on the next page (b) is another axial CT scan of the same patient:
12.4 The portal vein supplies about 70% of the blood to the liver. The remaining 30% is oxygenated blood from the hepatic arteries.
12.5 Identify the structures labelled A to E.
12.5 A Gallbladder
B Biliary tract
C Superior mesenteric artery
D Left lobe of the liver
E Tail of the pancreas
12.6 What are the functions of organ E?
12.6 The pancreas (E) has exocrine and endocrine functions. The pancreatic islets (Islets of Langerhans) produce insulin and glucagon. The pancreas also secretes enzymes capable of hydrolysing proteins, fats, and carbohydrates.
12.7 Name the different parts of organ E.
12.7 The pancreas is divided in to a head, uncinate process, neck, body, and tail. The head lies within the concavity of the duodenum, with its uncinate process extending to the left behind the superior mesenteric vessels. The neck is positioned anterior to the portal vein and superior mesenteric artery origins. The body runs upwards and to the left and the tail abuts the hilum of the spleen.
12.8 Define the term ‘pseudocyst’? Describe its pathogenesis.
15
zoom view
12.8 The lesser sac separates the stomach from the pancreas. If fluid leaks from the pancreas during acute pancreatitis this can become trapped within the lesser sac forming a pseudocyst.
 
Station 13
A 23-year-old woman undergoes laparoscopy for investigation of abdominal pain. You assist the consultant who is performing the operation. On the fourth postoperative day you examine her wounds and note that there is an abscess developing in the umbilical wound.
This is a dissection of the anterior abdominal wall:
zoom view
1613.1 Identify the structure labelled A? What gives rise to its colour?
13.1 A Linea alba (‘white line’). The whiteness indicates that it is a relatively avascular structure and hence ideal for incision without bleeding.
13.2 Identify the structures labelled B to G.
13.2 B Tendinous insertion
C External oblique muscle
D Rectus abdominis
E Superficial epigastric vessels
F Umbilicus
G Posterior layer of rectus sheath
13.3 Name in sequence the layer of the abdominal wall traversed by the umbilical port.
13.3 Skin, subcutaneous fat, Scarpa's fascia, umbilical cicatrix pillar, extraperitoneal fat, parietal peritoneum.
13.4 In which direction do the fibres of the internal oblique, external oblique, and transversus abdominis fibres run at the level of the umbilicus?
13.4 External oblique fibres run inferiorly and anteriorly. Internal oblique fibres run perpendicular to the external oblique muscle, directed superiorly and anteriorly. Transversus abdominis fibres run transversely.
13.5 What are the contents of the rectus sheath?
13.5 The rectus sheath contains the large rectus abdominis muscle (extending from the pubic symphysis to the xiphisternum/lower costal cartilages), the pyramidalis muscle, the superior and inferior epigastric vessels, ventral primary rami of T7–T12, and lymphatics.
13.6 What is the surface marking of the arcuate line and what is its significance?
13.6 The arcuate line (Douglas’ line) demarcates the lower limit of the posterior sheath. It is located about one-third of the distance from the umbilicus to the pubic crest. Above the level of this line, the internal oblique aponeurosis splits to envelope the rectus abdominis muscle, and the transversus abdominis aponeurosis runs under the rectus abdominis. Below the arcuate line, the internal oblique and transversus abdominis aponeuroses merge and pass superficial to the rectus muscle. Hence, below the arcuate line the only layers deep to the rectus abdominis are the transversalis fascia, extraperitoneal fat, and parietal peritoneum.
13.7 What is the distal limit of Scarpa's fascia?
13.7 Scarpa's fascia extends in to the thigh and fuses with the fascia lata at the flexure of the skin crease of the hip joint (about 1 cm below the inguinal ligament).
13.8 What is the continuation of Scarpa's fascia in the perineum called?
13.8 It fuses with Colles’ fascia in the perineum.
13.9 Where is the median umbilical ligament located and what does it contain?
13.9 The median umbilical ligament extends from the bladder to the umbilicus, on the deep surface of the anterior abdominal wall. It can be seen easily during laparoscopy by pointing the laparoscope towards the anterior abdominal wall in the median plane. It contains the urachus, which is the remnant of the allantois, a canal that drains the urinary bladder of the fetus that joins and runs through the umbilical cord. If the allantois fails to close then urine continues to leak through the umbilicus after birth.
13.10 Where are the medial umbilical ligaments located and what do they contain?
13.10 The medial umbilical ligaments are lateral to the median umbilical ligament on the deep surface of the anterior abdominal wall. They contain the remnant of the fetal umbilical arteries.
 
Station 14
A 22-year-old woman presenting with right iliac fossa pain undergoes laparoscopy for suspected appendicitis. You assist the consultant performing the operation.
This is a dissection of the appendix and lower abdominal structures in a normal subject:
zoom view
14.1 Identify the structures labelled A, B, D and E.
14.1 A Caecum
B Appendix
D Ileum
E Mesentery of small bowel
14.2 Identify the structure labelled C. What structures run in this tissue?
14.2 C Mesoappendix. The most important structures are the appendicular artery and vein, which may bleed if not ligated carefully. There are also autonomic nerves, lymphatic vessels, and sometimes a lymph node.
14.3 Explain on the basis of anatomical principles why infection of the appendix may result in necrosis whilst infection of the gallbladder usually does not.
14.3 A small non-anastomosing single artery, the appendicular artery, supplies the appendix (Figure 1.9). When the appendix becomes inflamed, oedema of the wall compresses the artery causing thrombosis. This leads to necrosis and perforation of the blind ending tip of the appendix. In contrast, in addition to the cystic artery, the gallbladder has collateral supply from the liver bed, ensuring that adequate blood supply is preserved.
14.4 Name four common positions assumed by the appendix.
14.4 Common positions include: retrocolic/retrocaecal, pelvic/subcaecal, retroileal/preileal. The order of frequency is disputed but the commonest two are probably pelvic and retrocaecal.
zoom view
Fig. 1.9: Appendix and arterial supply.
This variability of the appendix position can make diagnosis sometimes difficult and removal technically difficult.
1714.5 Using embryological principles explain the changing nature and location of pain in appendicitis.
14.5 Visceral pain from the appendix is triggered by distension of the lumen or muscle spasm. Afferent pain fibres travel to the T10 spinal level, and a midline periumbilical pain is felt. As the appendix becomes more inflamed it can cause localised inflammation of the peritoneum, and pain is referred to the right iliac fossa.
14.6 Which nerves may be damaged when performing an open appendicectomy?
14.6 The ilioinguinal and iliohypogastric nerves. To avoid these nerves incision should not be closer than 3 cm from the anterior superior iliac spine.
14.7 What are taeniae coli and where do they converge?
14.7 The teniae coli are three bands of smooth muscle running longitudinally along the caecum, ascending, transverse and descending and sigmoid colon (Figure 1.9). They contract to form haustra, which are sacculations of the large bowel that can be seen on radiographs. The teniae coli converge at the vermiform appendix and the rectum.
14.8 What are appendices epiploicae?
14.8 Appendices epiploicae are small fat-filled peritoneal pouches along the teniae coli. They can sometimes become inflamed (epiploic appendagitis) mimicking appendicitis and other intra-abdominal conditions.
 
Station 15
A 63-year-old woman gives a history of urinary incontinence a couple of weeks following an abdominoperineal resection for rectal cancer. At the last clinic attendance an abdominal CT was booked. Before you review her scans you wish to familiarise yourself with the features of a normal female subject.
Image (a) is a contrast-enhanced axial CT through the pelvis (demonstrating normal anatomy):
zoom view
15.1 Identify the structures labelled A to E.
15.1 A Right external iliac artery
B Right external iliac vein
C Urinary bladder
D Uterus
E Rectum
15.2 At what vertebral level does the structure labelled A divide?
15.2 L5/S1.
15.3 What bony landmark in the pelvis defines this division?
15.3 The sacroiliac joints.
15.4 Does the ureter pass anterior or posterior to this division?
Image (b) on the next page is an axial CT scan through the pelvis of the same patient:
15.4 The ureter passes anteriorly over the bifurcation of the iliac arteries.
15.5 Identify the structures labelled A to D
15.5 A Sartorius
B Superior pubic ramus
C Obturator internus
D Ischium
15.6 Name the parts of the levator ani.
15.6 The levator ani originates from the body of the pubis, the ischial spine, and the fascia of obturator internus. It inserts in to the perineal body, the anococcygeal body, and the walls of the pelvic organs below the bladder (the prostate, vagina, rectum and anal canal). As well as the functions listed in Table 1.4 it also increases intra-abdominal pressure during defecation, micturition, and parturition. The parts are outlined in Table 1.4.
15.7 What are the boundaries of the pelvic outlet?
15.7 The pelvic outlet is bounded posteriorly by the coccyx, laterally by the ischial tuberosities, and anteriorly by symphysis pubis (see Figure 1.11).
Table 1.4   The parts and functions of levator ani
Part
Name
Functional anatomy
Anterior
  • Levator prostatae
  • Sphincter vaginae
Forms a sling around the prostate/vagina and inserts in to the perineal body, helping to stabilise it
Intermediate
  • Puborectalis
  • Pubococcygeus
Forms a sling around the junction of the rectum/anal canal
Posterior
Iliococcygeus
Inserted into the anococcygeal body and the coccyx
15.8 What attaches to the perineal body?
18
zoom view
15.8 The perineal body is a pyramidal fibromuscular mass of tissue at the junction of the urogenital triangle and the anal triangle. It has attachments to: the external anal sphincter, bulbospongiosus muscle, superficial and deep transverse perineal muscles, anterior fibres of levator ani, and the external urinary sphincter.
 
Station 16
A 79-year-old man presents to his general practitioner with a 3-month history of weight loss and change in bowel habit. He is referred to the colorectal clinic where he undergoes investigations to rule out colorectal malignancy.
This is a contrast study of the large bowel (demonstrating normal anatomy):
zoom view
1916.1 How does the small bowel differ in appearance from the large bowel on an abdominal plain film?
16.1 The large bowel is peripheral and less coiled. It has haustra (which on an abdominal radiograph do not traverse the entire diameter of the colon). The lumen of large bowel is greater than the small bowel.
16.2 What type of radiological study is shown? Comment on the diameter of the bowel.
16.2 Double contrast barium enema. The bowel has been inflated by air pumped through the rectum.
16.3 What name is given to the lines labelled B?
16.3 B Haustra
16.4 Name the segments of the colon labelled A and C. What is the blood supply to these structures?
16.4 A Caecum/ascending colon. This is supplied by the colic branch of the ileocolic artery, and the right colic artery (both branches of the superior mesenteric artery).
C Descending colon. This is supplied by the left colic artery (a branch of the inferior mesenteric artery).
The blood supply to the rest of the large bowel is illustrated in Figures 1.13 and 1.18 (see pp. 69 and 77). The proximal two-thirds of the transverse colon is perfused by the middle colic artery (superior mesenteric), and the latter one-third by the inferior mesenteric. The sigmoid arteries supply the sigmoid colon. The rectum is supplied by the superior rectal artery (inferior mesenteric), middle rectal artery (internal iliac), and inferior rectal artery (internal iliac).
16.5 What is the marginal artery of Drummond?
16.5 There is a continuous vascular arcade throughout the length of the gastrointestinal tract, due to anastomosis of branches of the superior and inferior mesenteric arteries along the marginal artery of Drummond.
16.6 What are the topographical relations of structure D?
16.6 D Rectum. The rectovesical fascia of Denonvilliers separates the rectum from anterior structures and is dissected in rectal dissection for carcinoma. Anteriorly in the upper two-thirds are coils of small intestine that lie in the space between the rectum and bladder in men, or rectum and uterus in women (the pouch of Douglas). In the lower two-thirds anteriorly are the prostate, bladder, vas deferens, and seminal vesicles in males, and vagina in the female. Posteriorly are the sacrum, coccyx, median sacral and rectal vessels, sympathetic trunk, pelvic splanchnic nerves, and piriformis. Laterally lies levator ani, coccygeus and obturator internus muscles, fat, lymph nodes, ischioanal fossa, and the lateral ligaments of the rectum.
16.7 How does the peritoneum relate to structure D?
16.7 D The upper third of the rectum has peritoneum on its anterior and lateral surfaces, the middle third has peritoneum on its anterior surface only, and the lower third is beneath the rectal floor and has no peritoneal attachments.
16.8 What parts of the large bowel are particularly susceptible to injury in blunt abdominal trauma?
16.8 Injuries occur at the junctions of where mobile parts of the colon (the transverse and sigmoid) join the fixed parts (ascending and descending).
16.9 What muscles make up the posterior abdominal wall?
16.9 From medial to lateral are the psoas major, quadratus lumborum (above the iliac crest), or iliacus (below the iliac crest), transversus abdominis and internal oblique (Figure 1.10). The posterior part of the diaphragm also contributes to the upper posterior wall of the abdomen.
zoom view
Fig. 1.10: Posterior abdominal wall.
 
Station 17
A 52-year-old man presents with a swelling in the left groin and is diagnosed with an inguinal hernia. You are assisting your consultant in theatre during the inguinal hernia repair, and he asks you to identify some of the anatomical structures.
This is a dissection of the left groin (demonstrating normal anatomy):
zoom view
2017.1 Identify the structures labelled A to D.
17.1 A Spermatic cord
B Long saphenous vein
C Common femoral artery
D Common femoral vein
17.2 What are the contents of A?
17.2 The most well-known mnemonic for the contents of the spermatic cord (A) is the ‘rule of threes’ (Table 1.5).
Table 1.5   The ‘rule of threes’ for contents of the spermatic cord
Layers of fascia
  • External spermatic
  • Cremasteric
  • Internal spermatic
Arteries
  • Testicular
  • Cremasteric
  • Artery of the vas
Veins
  • Pampiniform plexus
  • Cremasteric
  • Vein of the vas
Nerves
  • Nerve to the cremaster
  • Sympathetic fibres (T10–T11)
  • Ilioinguinal nerve (this is actually on, not in, the cord)
Other structures
  • Vas deferens
  • Lymphatics
  • Processes vaginalis (pathologically, in patients with an indirect inguinal hernia)
17.3 What is the inguinal ligament? What are the medial and lateral attachments of the inguinal ligament?
17.3 The inguinal ligament is formed from the rolled over aponeurosis of the external oblique. It runs from the pubic tubercle to the anterior superior iliac spine.
17.4 What is the surface marking for the deep inguinal ring? Intraoperatively, what anatomical structure may be found that helps define this point?
17.4 The deep inguinal ring is an opening in the fascia transversalis at the mid-point of the inguinal ligament. Medially run the inferior epigastric vessels.
17.5 How may an inguinal hernia be clinically distinguished from a femoral hernia?
17.5 The opening of an inguinal hernia is above and medial to the pubic tubercle whereas a femoral hernia is below and lateral.
17.6 What are the boundaries of the inguinal canal?
17.6 The boundaries of the inguinal canal:
  • anterior: external oblique aponeurosis, reinforced at its lateral one-third by the origin of the internal oblique
  • posterior: conjoint tendon medially (the fused insertion of the internal oblique and transversus abdominis), transversals fascia laterally
  • roof: arching fibres of the internal oblique and transversus abdominis
  • floor: the inguinal ligament, and the lacunar ligament medially.
17.7 What is the embryological origin of the cremasteric fascia?
17.7 As the processus vaginalis descends into the scrotum during development it brings with it layers of the abdominal wall. The external spermatic fascia is derived from the external oblique aponeurosis. The cremasteric fascia is derived from the internal oblique. The internal spermatic fascia is derived from the fascia transversalis.
17.8 Describe the significance and boundaries of Hesselbach's triangle.
17.8 Direct inguinal hernias pass through Hesselbach's triangle, a defect in the transversalis fascia, whereas indirect hernias must traverse the deep inguinal ring. The boundaries of Hesselbach's triangle are:
  • medially – the lateral border of rectus abdominis
  • superolaterally – the inferior epigastric vessels
  • inferolaterally – the inguinal ligament.
17.9 Why is one cautioned against taking deep bites when suturing the inferior border of a mesh to the inguinal ligament?
17.9 Because the femoral vessels run just under this ligament, and may bleed profusely! The femoral vein is more difficult to control than the femoral artery.
 
Station 18
A 52-year-old man presents to the colorectal clinic with rectal bleeding. On proctoscopy you identify first degree haemorrhoids and proceed to perform banding.
The image below is a normal axial dissection of the male pelvis viewed from below:
zoom view
18.1 Identify the structures labelled A to G
18.1 A Right superficial femoral artery
B Right vas deferens and spermatic cord
C Right obturator internus
D Alcock's canal, internal pudendal vessels, pudendal nerve
E Left sciatic nerve
F Membranous urethra
G Anus
18.2 Regarding the structure labelled G:
21
18.2a what communication exists between the portal and systemic venous circulation in this region?
18.2a There is communication between the superior rectal vein (portal system), and the inferior rectal veins (draining to the internal iliac vein via the internal pudendal veins, systemic circulation).
18.2b Name other regions in the body where similar portosystemic communications exist.
18.2b The other communications are shown in Table 1.6.
18.3 Describe the boundaries of the anal triangle of the perineum.
18.3 This is formed by the two ischial tuberosities and the coccyx (Figure 1.11). The anterior border is the posterior border of the perineal membrane, and the sacrotuberous ligaments form the two sides.
18.4 What are the topographical relations of the prostate?
18.4 Anterior: the pubic symphysis (separated by extraperitoneal fat), and the prostatic plexus of veins.
Posterior: the rectum separated by the fascia of Denonvilliers.
Superior: the bladder.
Inferior: the external sphincter of the bladder.
Table 1.6   Portal-systemic communications
Lower oesophagus
Between the oesophageal branch of the left gastric vein (portal system), and the oesophageal vein (azygos system)
Abdominal wall
Between portal branches in the liver and the veins passing to the abdominal wall (forming a caput medusae)
Bare area of liver
Between portal veins in the liver and veins of the diaphragm (across the bare area)
Retroperitoneum
Between the portal tributaries in the mesentery and retroperitoneal veins
zoom view
Fig. 1.11: Anal and urogenital triangles.
18.5 What are the contents and boundaries of the ischioanal fossae?
18.5 The ischioanal fossae (or ischiorectal fossae, an old term) are wedge shaped spaces on either side of the anal canal. Their boundaries are:
  • laterally: obturator internus muscle and fascia
  • medially: levator ani and pelvic fascia, external anal sphincter
  • anteriorly: the urogenital perineum
  • posteriorly: sacrotuberous ligament and gluteus maximus
  • inferiorly: skin and subcutaneous fat
  • superiorly: levator ani
The space contains fat (which is particularly prone to infection and abscess formation) and the inferior rectal nerve and vessels. The lateral walls contain Alcock's canal, which has in it the pudendal nerve and vessels.
18.6 Where is the commonest site for an anal fissure?
18.6 Anal fissures occur most commonly in the posterior midline. Fissures develop in the anal valves (the lower ends of the anal columns) as hard faecal matter catches during defecation. This area may be susceptible due to a lack of support from the superficial part of the external sphincter.
18.7 Anatomically speaking, what is a haemorrhoid?
18.7 A haemorrhoid is fold of mucosa and submucosa containing a varicosed tributary of the superior rectal vein and a terminal branch of the superior rectal artery.
18.8 What is Goodsall's rule?
18.8 Goodsall's rule states that the external opening of a fistula situated behind the transverse anal line will open in to the anal canal in the posterior midline, but a fistula that opens anterior to this line is associated with a direct tract.
18.9 What are the lengths of the normal adult anal canal and rectum?
18.9 The anal canal is approximately 4 cm long and the rectum is about 13 cm long.
18.10 What are the topographical relations of the anal canal?
18.10 The topographical relations of the anal canal are:
  • posteriorly: the anococcygeal body
  • laterally: the ischiorectal fossae
  • anteriorly in men: the perineal body, the urogenital diaphragm, and the membranous part of the urethra
  • anteriorly in women: the lower part of the vagina.
 
Station 19
A 45-year-old man known to have ulcerative colitis presents with acute abdominal pain. An abdominal radiograph is requested and reveals toxic megacolon. You assist your consultant in performing a subtotal colectomy.
This image is an axial dissection of the abdomen at the level L2 (demonstrating normal anatomy) viewed from below:
zoom view
19.1 Identify the parts of the bowel labelled A, B and C.
19.1 A Hepatic flexure or ascending colon
B Transverse colon
C Small bowel
19.2 What muscle is present at D?
19.2D Left psoas major muscle
2219.3 Identify the structures labelled E, F, and G.
19.3 E Spleen
F Linea alba
G Rectus abdominis muscle
19.4 What are the posterior topographical relations of the ascending colon?
19.4 Posterior to the ascending colon lies:
  • musculoskeletal: iliac crest, iliacus, quadratus lumborum, transversus abdominis, and the right psoas.
  • organs: lower pole of the right kidney.
  • nerves: iliohypogastric and ilioinguinal nerves.
19.5 What are the posterior topographical relations of the descending colon?
19.5 Posterior to the descending colon lies:
  • musculoskeletal: iliac crest, iliacus, quadratus lumborum, transversus abdominis, and the left psoas.
  • organs: lateral border of left kidney
  • nerves: iliohypogastric, ilioinguinal, femoral nerves and the lateral cutaneous nerve of the thigh.
19.6 Which circumferential positions in the colonic wall are susceptible to acquired diverticular disease?
19.6 Diverticula are herniations of the mucosa through the circular muscle at points where the blood vessels pierce the muscle (natural points of weakness).
19.7 What does the transverse mesocolon attach to on the posterior abdominal wall?
19.7 The transverse mesocolon attaches the transverse colon to the posterior wall of the abdomen and the pancreas. It is continuous with the two posterior layers of the greater omentum (Figure 1.15).
19.8 What structures run within the transverse mesocolon?
19.8 The transverse mesocolon contains the transverse colon (in its free edge), the middle colic vessels and their branches, lymphatics, autonomic nerves, and extraperitoneal fatty tissue.
19.9 Of what clinical significance is the ileocaecal valve in a patient with large bowel obstruction?
19.9 An incompetent ileocaecal valve allows decompression of the large intestine in patients with large bowel obstruction and thereby reduces the risk of perforation.
19.10 Which part of the bowel most commonly undergoes volvulus?
19.10 The sigmoid colon has a long mesentery and may rotate upon it, causing an obstructed, often massively distended, loop of bowel. Volvulus of the caecum and transverse colon may also occur less commonly.
 
Station 20
A 75-year-old man with a history of hypertension and hypercholesterolaemia presents with post-prandial abdominal pain. Ultrasound and OGD do not reveal any abnormalities. Chronic mesenteric ischaemia is suspected and an angiogram is requested.
Image (a) is a digital subtraction angiogram of the coeliac plexus and its branches (demonstrating normal anatomy):
zoom view
20.1 Identify the arteries labelled A to F.
20.1 See Figure 1.12.
A Proper hepatic artery
B Common hepatic artery
C Left gastric artery
D Gastroduodenal artery
E Coeliac trunk
F Splenic artery
20.2 At what vertebral level does the coeliac artery (coeliac axis) leave the aorta?
20.2 Upper part of L1 (not T12, although many textbooks claim this).
20.3 What organs does D supply?
20.3 The gastroduodenal artery (D) supplies the stomach, the duodenum, and, indirectly, the pancreatic head and neck (via the anterior and posterior superior pancreaticoduodenal arteries).
20.4 Within what peritoneal structure does A run, and what topographical relationship does it have to the bile duct and portal vein?
Image (b) on the next page is a digital subtraction angiogram of the superior mesenteric artery and its branches (demonstrating normal anatomy).
23
zoom view
20.4 The hepatic artery (A) runs in the free border of the lesser omentum, anterior to the portal vein and left of the bile duct.
20.5 Identify the arteries labelled A to F.
20.5 See Figure 1.13.
A Right colic artery
B Ileocolic artery
C Appendicular artery
D Superior mesenteric artery
zoom view
Fig. 1.12: The coeliac axis.
zoom view
Fig. 1.13: Branches of the superior mesenteric artery.
E Main stem of jejunal arteries
F Main stem of ileal arteries
20.6 At what vertebral level does the superior mesenteric artery leave the aorta?
20.6 L1
20.7 At what vertebral level does the inferior mesenteric artery leave the aorta?
20.7 L3
20.8 Over what structures does the superior mesenteric artery pass before entering the root of the small intestinal mesentery.
20.8 After leaving the aorta, the superior mesenteric artery passes over the left renal vein, beneath the splenic vein and neck of the pancreas. It then passes over the uncinate process of the pancreas and the junction of the third and fourth parts of the duodenum, before entering the mesentery of the small and large bowel to give off its terminal branches.
20.9 Which part of the large intestine is particularly susceptible to ischaemia?
20.9 The distal third of the transverse colon/splenic flexure is termed a ‘watershed’ area, as there is a change in blood supply from the superior mesenteric to the inferior mesenteric artery. Watershed areas are vulnerable to ischaemia as they do not have good collateral supply.
 
Station 21
A 72-year-old man with a known abdominal aortic aneurysm is admitted with acute, severe abdominal and back pain. A diagnosis of leaking aneurysm is made and he is taken to theatre for an emergency operation.
This is a dissection of the retroperitoneal region of a normal subject:
zoom view
2421.1 Identify the structures labelled A to H.
21.1 A Right renal vein
B Inferior vena cava
C Testicular vein (right)
D Left suprarenal organ
E Superior mesenteric artery
F Left renal vein
G Abdominal aorta
H Left ureter
21.2 What is the ligament of Treitz?
21.2 The ligament of Treitz connects the duodenojejunal junction to the diaphragm (this ‘ligament’ actually contains muscular fibres that on contraction widen the angle of the duodenojejunal flexure assisting movement of intestinal contents). The ligament is commonly cut to access the aorta.
21.3 What are the lateral branches of the aorta?
21.3 The branches of the aorta are outlined in Table 1.7.
Table 1.7   Branches of the abdominal aorta
Three anterior visceral branches
  • Coeliac
  • Superior mesenteric
  • Inferior mesenteric
Three lateral visceral branches
  • Suprarenal
  • Renal
  • Testicular or ovarian
Five lateral abdominal wall branches
  • Inferior phrenic
  • Four lumbar branches
Three terminal branches
  • Two common iliac branches
  • Median sacral
21.4 What is the arterial supply of the adrenal glands?
21.4 The adrenal glands are supplied by the superior adrenal artery (from the inferior phrenic), middle adrenal artery (abdominal aorta), and inferior adrenal artery (renal artery).
21.5 What is the venous drainage of the adrenal glands?
21.5 The adrenal vein (the right adrenal vein drains into the inferior vena cava, the left adrenal vein drains in to the left renal vein).
21.6 At operation the inferior mesenteric artery can often be ligated without serious consequence. What is the anatomical explanation for this?
21.6 Due to collateral supply via the marginal artery of Drummond. It is sometimes ligated in operations such as open aortic aneurysm repair.
21.7 Regarding structure B:
21.7a at what vertebral level does it commence?
21.7a Commences at L5 behind the common iliac arteries.
21.7b as it ascends, what are its successive anterior topographical relations?
21.7b It is initially related anteriorly to the small intestine, the third part of the duodenum, the head of the pancreas, and the first part of the duodenum. It passes behind the epiploic foramen (in front of which is the portal vein, common bile duct and hepatic artery); it then ascends in a groove in the liver before traversing the diaphragm.
21.7c name the tributaries.
21.7c This can be remembered by the mnemonic: I L ike To Rise So High: Iliac, Lumbar, Testicular, Renal, Suprarenal, Hepatic.
 
Station 22
An obese 55-year-old woman presents with long standing intermittent, colicky right upper quadrant pain. An abdominal ultrasound scan proves inconclusive and she is referred for magnetic resonance cholangiopancreatography (MRCP). You are due to review her in the outpatient clinic and wish to familiarise yourself with normal biliary anatomy as seen on a MRCP.
The image below is a MRCP demonstrating the normal anatomy of the biliary system:
zoom view
22.1 Identify the structures labelled A to E.
22.1 A Gallbladder
B Right hepatic duct
C Left hepatic duct
D Right renal pelvis
E Common bile duct
2522.2 What route does bile take to enter the intestinal tract? In gallstone ileus, what route does a gallstone usually travel to enter the intestinal tract?
22.2 Bile is stored in the gallbladder. It passes in to the cystic duct, which joins the common hepatic duct to form the common bile duct (Figure 1.14). This travels in the free edge of the lesser omentum (with the hepatic artery and portal vein). The duct is joined by the main pancreatic duct (of Wirsung) at the ampulla of Vater, which enters the second part of the duodenum past the sphincter of Oddi.
Gallstone ileus is the condition of a gallstone causing mechanical intestinal obstruction (hence the condition is not really ileus at all). Instead of travelling through bile ducts, gallstones usually erode through the wall of the gallbladder over a period of time. They often get lodged in the distal ileum.
22.3 What is the narrowest part of the extrahepatic biliary system?
22.3 Its opening in to the second part of the duodenum.
22.4 What is the function and composition of bile?
22.4 Bile is produced by hepatocytes in the liver. It is composed mainly of water (85%), bile salts, mucous, pigments, fats, inorganic salts, and cholesterol. Bile acts as a surfactant, emulsifying fats. The increased surface area allows for more efficient action of enzymes such as pancreatic lipase. Its other functions include: being the route of excretion for the haemoglobin breakdown product bilirubin, and neutralising excess stomach acid before it enters the ileum.
22.5 Which hormone stimulates the release of bile? What is the trigger for this hormone and where is the hormone synthesised.
22.5 Cholecystokinin is a peptide hormone that stimulates the contraction of the gallbladder and the relaxation of the sphincter of Oddi.
zoom view
Fig. 1.14: The biliary system.
It is synthesised by the ‘I-cells’ of the mucosal epithelium of the small intestine, and secreted in response to Chyme entering the duodenum. Its other functions include: increasing the production of bile in the liver; stimulation of the release of digestive enzymes in the pancreas; causing relaxation of the stomach musculature.
22.6 What epithelium lines the extrahepatic biliary ducts?
22.6 Extra-hepatic ducts are lined by tall columnar cells interspersed with mucous glands.
22.7 What is Pringle's manoeuvre?
22.7 Pringle's manoeuvre temporarily prevents blood from entering the liver by compressing the hepatic artery and portal vein. Intraoperatively it can be performed by placing a finger within the foramen of Winslow and another on its anterior wall and squeezing.
22.8 What is Mirizzi's syndrome?
22.8 Mirizzi's syndrome is a cause of obstructive jaundice caused by one or more gallstones becoming impacted in Hartmann's pouch. The biliary obstruction can be caused by either external compression of the common hepatic duct by the gallstone, or fistulisation of the gallstone in to the common hepatic duct.
 
Station 23
A 23-year-old female bank official presents as an emergency after being shot during a bank raid. There is a bullet wound in her right loin with extensive bleeding from a superficial vessel. She is taken to theatre for exploration of the wound.
This photograph demonstrates some features of the surface anatomy of the anterior aspect of the abdomen:
zoom view
23.1 What anatomical lines are indicated by C and D?
23.1 See Figure 1.8.
C Linea semilunaris
D Arcuate line
23.2 What plane is indicated by dotted yellow line B? What anatomical structures can be found at this level?
23.2 B The supracristal plane (Table 1.3). This is a transverse plane through the uppermost part of the iliac crest, at the level of the L4 vertebra. It usually passes close to the umbilicus. The plane divides the lower and upper quadrants of the abdomen. At this level the abdominal aorta bifurcates.
23.3 Which vessels in the anterior abdominal wall, indicated by point A, may have been punctured by the bullet?
23.3 The inferior epigastric artery is a branch of the external iliac artery, and enters the rectus sheath anterior to the arcuate line by piercing the transversalis fascia. It runs in the sheath posterior to the rectus abdominis muscle and supplies the anterior abdominal wall. It ends by anastomosing with the superior epigastric branch of the internal thoracic artery. The inferior epigastric vein follows a similar course and drains in to the external iliac vein.
23.4 What are the origins of these vessels?
23.4 The external iliac artery and vein.
23.5 Define the linea semilunaris.
23.5 The linea semilunaris (Figure 1.8 and Table 1.3) is a tendinous line lateral to the rectus abdominis, extending from the cartilage of the ninth rib to the pubic tubercle. It demarcates the lateral fusion of the anterior and posterior rectus sheath layers.
23.6 What is the innervation and action of the internal oblique?
23.6 The internal oblique is supplied by the lower six thoracic nerves, the iliohypogastric nerve, and the ilioinguinal nerve (also true for the external oblique and the transversus abdominis). It assists in flexion and rotation of the trunk.
23.7 List the functions of the muscles of the abdominal wall.
23.7 The muscles of the anterior and lateral walls have a number of functions. They assist during forced expiration by pulling down the ribs and sternum. During inspiration they aid the diaphragm by relaxing. They protect the abdominal contents from trauma. They can increase abdominal pressure during micturition, defecation, vomiting and parturition by contracting simultaneously with the diaphragm with a closed glottis.
26
 
Station 24
A 70-year-old man presents in shock with a rigid abdomen. He is taken to theatre and found to have a perforated duodenal ulcer, which is repaired with an omental patch.
This is a prosection showing the stomach and other upper abdominal viscera in a normal subject:
zoom view
24.1 Identify the structure labelled 1? What are its parts? Which of these parts are mobile?
24.1 1 The duodenum
The first part of the duodenum begins at the pylorus and runs up and backwards (at the transpyloric plane). The second part of the duodenum runs vertically downward in front of the hilum of the right kidney. The bile duct and pancreatic duct enter the duodenum at the ampulla of Vater, with the accessory pancreatic duct nearby. The third part of the duodenum runs horizontally to the left on the subcostal plane, following the lower margin of the head of the pancreas. The fourth part of the duodenum runs up and to the left and ends at the duodenojejunal flexure, which is indicated by the suspensory ligament of Treitz.
Only the first few centimetres of the first part of the duodenum are intraperitoneal (mobile), the rest of the duodenum is retroperitoneal.
24.2 Identify the structure labelled 2? What are its branches?
24.2 2 Coeliac axis
The branches of the coeliac axis are the left gastric artery, the splenic artery, and the common hepatic artery (Figure 1.12).
24.3 Identify the structure labelled 3? Describe its blood supply.
24.3 3 Pancreas
The splenic and superior/inferior pancreaticoduodenal arteries supply the pancreas. The veins are named after the arteries and drain in to the portal system.
24.4 Identify the structures labelled A to D.
24.4 A Common hepatic artery
B Right lobe of the liver
C The spleen
D The splenic artery
24.5 What is the name of the fold of peritoneum that hangs from the greater curvature of the stomach? What blood vessels run in this tissue?
24.5 The greater omentum is a fold of parietal peritoneum that is suspended from the greater curvature of the stomach (Figure 1.15). Its anterior fold hangs over the small intestines before being reflecting back up on itself to reach the transverse colon, and then to the posterior abdominal wall. The right and left gastroepiploic vessels run in and supply the greater omentum. It also carries lymphatics (to the stomach) and autonomic nerves.
24.6 What is the epiploic foramen (foramen of Winslow)? Define its boundaries.
24.6 The epiploic foramen (foramen of Winslow) is the entrance to the lesser sac:
  • anteriorly: border of lesser omentum carrying the bile duct, hepatic artery, and portal vein
  • posteriorly: inferior vena cava
  • superiorly: caudate process of the liver
  • inferiorly: first part of the duodenum.
24.7 What are the topographical relations of the second part of the duodenum?
24.7 The topographical relations of the second part of the duodenum:
  • anteriorly: gallbladder, right lobe of the liver, transverse colon, and small intestine
    zoom view
    Fig. 1.15: The compartments of the abdomen.
  • posteriorly: right renal pelvis
  • superiorly: head of the pancreas, bile ducts draining in to the duodenum
  • inferiorly: ascending colon, right colic flexure, and right lobe of the liver.
24.8 What is the definition and contents of the supracolic compartment?
24.8 The supracolic compartment is the division of the abdomen above the transverse mesocolon, and the infracolic compartment is the division below this level. The supracolic compartment contains the oesophagus, stomach, first part of duodenum, lesser omentum, spleen, liver, and gallbladder.
24.9 What are the contents of the splenorenal (lienorenal) ligament?
24.9 The splenic vessels and the tail of the pancreas.
24.10 What vessels are carried within the gastrosplenic ligament?
24.10 The short gastric and left gastroepiploic vessels.
24.11 Where is the root of the small bowel mesentery attached?
24.11 The left L2 transverse process to the right sacroiliac joint.
27
 
Station 25
A 45-year-old woman with recurrent bouts of epigastric pain is noted to have gallstones on ultrasound scanning. She attends for laparoscopic cholecystectomy. You are assisting the consultant who is performing the procedure.
This is a prosection displaying the inferior surface of the liver (demonstrating normal anatomy):
zoom view
25.1 Identify structures labelled A to J.
25.1 See Figure 1.16.
A Gallbladder
B Right lobe of liver
C Bile duct
D Inferior vena cava
E Quadrate lobe of liver
F Ligamentum teres hepatis and falciform ligament
zoom view
Fig. 1.16: The posterior surface of the liver.
G Left lobe of liver
H Common hepatic artery
I Portal vein
J Caudate lobe of liver
25.2 State the blood supply, lymphatic drainage, and nerve supply of structure A?
25.2 The cystic artery is the main blood supply to the gallbladder (A). This is usually a branch of the right hepatic, but there are several anatomic variants. There are also small vessels that run from the gallbladder to the liver in the gallbladder bed. Lymph drains via a cystic lymph node near the neck of the gallbladder to the hepatic then coeliac nodes. The gallbladder receives sympathetic and parasympathetic supply via the coeliac plexus.
25.3 What epithelium lines structure A?
25.3 The gallbladder (A) is lined by tall columnar epithelium. This epithelium does not secrete mucous.
25.4 What are the boundaries of Calot's triangle and what does the triangle contain?
25.4 Calot's triangle is now conventionally defined as the cystic duct, the common hepatic duct, and the inferior surface of the liver (Figure 1.17) (although the original description in 1891 described the triangle as formed by the cystic duct, the bile duct and the cystic artery). The cystic artery is constantly found in this triangle. Visualising the ducts and arteries is essential before removing the gallbladder to ensure that they are not inadvertently injured. Common bile duct injury, especially those injuries unrecognised at time of surgery, can be disastrous.
25.5 What is the upper limit of the normal diameter of the common bile duct on ultrasound in a 20-year-old?
25.5 The upper limit of the common bile duct diameter on ultrasound in adults is conventionally about 7 mm. In the elderly, and after cholecystectomy, the diameter increases.
25.6 What is the embryological origin of the ligamentum teres hepatis?
25.6 The ligamentum teres hepatis is the remnant of the left fetal umbilical vein.
zoom view
Fig. 1.17: Calot's triangle.
25.7 What is the embryological origin of the liver?
25.7 The liver and biliary tree appear in the third/fourth week as hepatic diverticula from the ventral wall of the distal foregut endoderm.
 
Station 26
A 64-year-old man is diagnosed with rectal adenocarcinoma and is admitted for an anterior resection of the rectum. You are assisting your consultant who is performing the procedure.
28This is a sagittal prosection of the left hemipelvis of a normal adult male:
zoom view
26.1 Identify the structures labelled A to F.
26.1 A Rectum
B Internal iliac artery
C Ductus deferens
D Bladder
E Pubic symphysis
F Penis
26.2 Name the segments of the gastrointestinal tract that are immobile.
26.2 Immobile implies retroperitoneal. This includes most of the duodenum, the ascending and descending colon, and the distal two thirds of the rectum.
26.3 What are the branches of the inferior mesenteric artery?
26.3 Branches of the inferior mesenteric are: the left colic artery, branches to the sigmoid, and the superior rectal artery (the continuation of the inferior mesenteric artery) (Figure 1.18).
26.4 What epithelium lines the anal canal?
26.4 Longitudinal folds of simple columnar epithelium line the upper two-thirds. The lower one-third is lined by stratified squamous epithelium, blending with the skin. The dentate line divides these areas.
26.5 Where in the adult gut are the junctions between the embryological foregut and midgut, and the embryological midgut and hindgut?
26.5 The foregut runs from the mouth to the duodenum, as far as the entry of the bile duct (D2). The midgut ends two-thirds of the way along the transverse colon. The hindgut ends two-thirds of the way along the anal canal at the dentate line.
26.6 What is the superior boundary of the left paracolic gutter?
26.6 The paracolic gutters are peritoneal recesses on the posterior abdominal wall, lying lateral respectively to the ascending and descending colon. Their significance is that substances such as bile or pus can travel along their length and settle at sites remote from their origin. The left paracolic gutter is limited superiorly by the phrenicocolic ligament, and inferiorly by the attachment of the lateral limb of the sigmoid mesocolon at the pelvic brim. The right paracolic gutter is superiorly continuous with the hepatorenal pouch (Morrison's pouch), and inferiorly with the pelvis. The right paracolic gutter is continuous with the lesser sac.
zoom view
Fig. 1.18: Branches of the inferior mesenteric artery.
26.7 What is the most dependent part of the peritoneal cavity in the supine position?
26.7 The hepatorenal pouch (Morrison's pouch) is the most dependent part of the abdomen and is a common site for accumulation of fluid/pus/blood.
26.8 List the routes via which rectal adenocarcinoma spreads.
26.8 Rectal adenocarcinoma spreads via the following routes:
  • local spread: direct invasion of other structures in the pelvis
  • lymph node spread: regional and then distal
  • blood-borne distal spread: to the liver, lungs, and bone
  • peritoneal spread to other abdominal organs.
 
Station 27
A 45-year-old male builder sustains a heavy blow to the thorax and abdomen by a reversing truck. On arrival at the emergency department, the trauma series of radiographs reveals a fractured pelvis and diaphragmatic rupture.
The image on the next page is a dissection of the superior surface of the normal diaphragm, showing the structures that traverse it.
27.1 What structure passes through the opening labelled A? At what vertebral level does this structure traverse the diaphragm and what accompanies it through this opening?
29
zoom view
27.1 A Inferior vena cava. Enters the diaphragm at T8, accompanied by the right phrenic nerve.
27.2 What structure passes through the opening labelled B?
27.2 B Oesophageal hiatus
27.3 Identify the structure labelled C. At what vertebral level does it traverse the diaphragm and what accompanies it through this opening?
27.3 C Aorta. Traverses the diaphragm at T12, accompanied by the thoracic duct and azygous/hemiazygous veins.
27.4 Identify the structures labelled D to F
27.4 D Sternum
E Vertebral body
F Spinal cord
27.5 At what vertebral levels do the following traverse the diaphragm:
27.5a the vagus nerves?
27 The vagi accompany the oesophagus through the diaphragm at T10.
27.5b the right phrenic nerve?
27 T8 or T9.
27.5c the left gastric artery?
27 T10.
Table 1.8 summarises the structures traversing the diaphragm.
Table 1.8   Structures traversing the diaphragm
Vertebral level
Main structure transmitted
Additional structures transmitted
T8
Inferior vena cava opening
Right phrenic nerve
T10
Oesophageal opening
  • Vagi, branches of the left gastric vessels
  • Lymphatics
T12
Aortic opening
  • Thoracic duct
  • Azygos, and hemiazygos veins
T12 (crura)
Splanchnic nerves
27.6 Where, respectively, do the splanchnic nerves and sympathetic chain traverse the diaphragm?
27.6 The splanchnic nerve traverses the crura of the diaphragm, and the sympathetic chain passes behind the diaphragm deep to the medial arcuate ligament.
27.7 What nerves provide the motor innervation of the diaphragm?
27.7 The phrenic nerves (C3– C5; mnemonic: ‘C3, 4, 5 keeps the diaphragm alive’). These contain motor, sensory, and sympathetic nerve fibers. There is sometimes an accessory phrenic nerve (often a branch of the nerve to the subclavius).
27.8 Describe the course of these nerves.
27.8 The phrenic nerves originate at the C3–C5 vertebral levels. They run vertically downwards over the anterior scalene muscles deep to the prevertebral layer of deep cervical fascia. They enter the thorax by passing over the subclavian arteries. The right phrenic nerve passes along the right side of the brachiocephalic artery, posterior to the subclavian vein, and then crosses anterior to the root of the right lung, over the pericardium of the right atrium, and then leaves the thorax by passing through the caval opening in the diaphragm. The left phrenic nerve travels lateral to the left subclavian artery and passes in front of the root of the left lung and over the pericardium of the left ventricle to pierce the muscular diaphragm to supply the peritoneum on its under surface.
27.9 What four components contribute to the embryological origin of the diaphragm?
27.9 The central tendon is formed by the septum transversum. The peripheral rim comes from the body wall. There are also contributions from the oesophageal mesentery and the pleuroperitoneal membranes.
27.10 Name two common types of acquired diaphragmatic hernias.
27.10 The most common acquired hernias are termed ‘sliding’ and ‘rolling’. Sliding hernias consist of the projection of the upper part of the stomach through the diaphragm in to the chest when the patient lies or bends. It predisposes to gastroesophageal reflux due to incompetence of the lower oesophageal sphincter. A rolling hernia describes the fundus of the stomach rolling up through the diaphragm in front of the oesophagus. Patients with this condition do not experience reflux.
27.11 Name two common types of congenital diaphragmatic hernias.
27.11 Herniation may occur posteriorly through the foramen of Bochdalek. This is the most common form of congenital diaphragmatic hernia and is due to developmental failure of the posterolateral diaphragmatic foramina. It most commonly occurs on the left side. A hernia through the foramen of Morgagni is located anteromedially, between the costal and sternal origins of the diaphragm. Other forms of congenital hernia include a deficiency of the central tendon, or a large oesophageal hiatus.
 
Station 28
A 24-year-old man presents with severe colicky pain in the left renal angle radiating to the groin. He undergoes an intravenous urogram. Before reviewing the scan, you familiarise yourself with some of the normal features of this type of scan.
On the next page is a normal intravenous urogram.
28.1 At what vertebral levels do the kidneys lie?
28.1 T12–L3
28.2 Identify the structures labelled A, B and C.
30
zoom view
28.2 A Minor calyx of the left kidney
B Major calyx of the left kidney
C Left renal pelvis
28.3 What are the three natural narrowings in D?
28.3 D The right ureter. The three narrowest parts of the ureter are (i) the pelviureteric junction, (ii) where the ureter crosses the pelvic brim and (iii) the vesicoureteric junction.
28.4 What is the blood supply to D?
28.4 Like most long tubes the ureter has a segmental blood supply from vessels that it passes close to: the aorta, the renal artery, the testicular/ovarian artery, the internal iliac artery and the inferior vesical vessels.
28.5 What is the topographical relationship between the renal artery and the renal vein?
28.5 The renal vein is anterior to the renal artery, which is anterior to the renal pelvis.
28.6 Can you ligate the left renal vein without serious consequences and if so, why?
28.6 Yes, this is often cut during open aortic aneurysm repair (remember that the left renal vein reaches across the aorta to reach the inferior vena cava). This is possible due to sufficient collateral drainage via the adrenal and inferior phrenic veins.
28.7 Name the retroperitoneal structures of the abdomen.
28.7 Pancreas, kidneys, ureters, adrenals, aorta, para-aortic lymph nodes, lumbar sympathetic chain ascending/descending colon, the duodenum beyond the first few centimetres, inferior vena cava, rectum. The spleen is not retroperitoneal, a common incorrect answer given in the exam!
28.8 From what embryological structures are the kidneys and ureters derived?
28.8 The distal part of the pronephros develops in to the mesonephric duct. A diverticulum of the lower end of the mesonephric duct develops in to the metanephric duct. Tissue overlying the end of this duct develops in to the kidneys (metanephros), whilst the duct itself develops in to the collecting tubules, calyces, pelvis, and ureter.
28.9 What arrests the ascent of a horseshoe kidney from the pelvis during development?
28.9 The inferior mesenteric artery.
 
Station 29
A 37-year-old woman is referred to the surgical team with abdominal pain. However, soon after arriving in the emergency department she collapses, shocked. A urine test is positive for β-hCG and she is suspected to have a ruptured ectopic pregnancy.
31This is a sagittal view of a normal female left hemipelvis:
zoom view
29.1 Identify the structures labelled A, B, C.
29.1 A The rectum
B The pubic symphysis
C The bladder
29.2 Regarding structure D:
29.2a what is it?
29D The uterus
29.2b name the various parts of structure D.
29.2b See Figure 1.19.
Fundus: lies above the entrance of the fallopian tubes.
Body: the part that lies below the fallopian tubes.
Cervix: projects in to the vagina.
Cavity: hollow space within the uterus.
Internal os: communication in to the uterus.
External os: communication in to the vagina.
29.2c what is the blood supply to structure D?
29.2 The uterine artery (internal iliac artery), and the ovarian artery (abdominal aorta).
29.2d what are different positions in which it can lie?
29 The most common position is anteverted, where the long axis of the uterus is bent forwards. The uterus is also usually anteflexed (bent forward at the level of the internal os). It follows that the uterus can also be retroverted and retroflexed.
29.3 Regarding structure E:
29.3a what is it?
29E The cavity of the vagina.
29.3b what are its topographical relations?
29 The relations of the vagina are important to know for the purposes of vaginal examination. Anteriorly lies the bladder, urethra, and symphysis pubis. Posteriorly lies the Pouch of Douglas (in which fluid and bowel may be felt) and the rectum. Laterally are the levator ani muscles, pelvic fascia and the ureters. At the apex of the vagina is the cervix.
29.3c what is the blood supply?
29.3C the vagina is supplied by the vaginal, uterine, internal pudendal, and middle rectal arteries (all branches of the internal iliac artery). It is drained by the vaginal vein (internal iliac vein).
29.3d what epithelium lines it?
29.3d the vagina and vaginal cervix are lined by stratified squamous epithelium. The uterine cervix is lined by tall columnar cells (which secrete the cervical mucus plug). The uterus is lined by cuboidal ciliated cells forming tubular glands.
zoom view
Fig. 1.19: The uterus.
29.4 Name the parts of the fallopian tube.
29.4 See Figure 1.19. The infundibulum is the most lateral part and opens in to the peritoneal cavity via the ostium. This joins the wide ampulla, becoming the narrow isthmus before piercing the uterine wall.
29.5 Where are the Bartholin's glands and what is their function?
29.5 These are two glands located in the labium magus. They secrete mucus to provide vaginal lubrication. They can become obstructed, forming Bartholin's cysts, which are prone to infection.
29.6 What are the broad ligaments of the uterus?
29.6 The broad ligaments are folds of peritoneum that connect the lateral sides of the uterus to the pelvic sidewalls. The fallopian tubes lie in the free edge of the broad ligaments and open into the cornu of the uterus. The ligaments also carry the ovary (attached by the mesovarium to the posterior aspect of the uterus), the round ligament, the ovarian ligament, the uterine vessels and their branches, lymphatics and nerves.
29.7 What is the round ligament of the uterus?
29.7 The round ligaments maintain anteversion of the uterus during pregnancy. They are attached to the uterine horns (where the uterus and the fallopian tubes meet) and travel in the anterior layer of the broad ligament to leave the pelvis via the internal inguinal ring. They then pass through the inguinal canal to attach to the labium majora.
29.8 Name three other ligaments that support the uterus and vagina.
29.8 The cardinal (or cervical) ligaments pass laterally from the cervix and upper vagina to the sidewalls of the pelvis. The uterosacral ligaments pass backwards from the posterolateral cervix and from the lateral vaginal fornices to attach to the periosteum in front of the sacroiliac joints and the lateral part of the sacrum. The pubocervical fascia extends from the cardinal ligament to the pubis, either side of the bladder (acting as a sling).
32
 
Station 30
A 63-year-old man is diagnosed with bladder cancer and undergoes a radical cystectomy and ileal conduit formation. You are assisting the consultant who is performing the operation.
This is a dissection of the male pelvis (demonstrating normal anatomy):
zoom view
30.1 Identify the structures labelled A to H.
30.1 A Right psoas major muscle
B Right external iliac artery
C Left iliacus muscle
D Left ureter
E Superior hypogastric plexus
F Left internal iliac artery
G Left external iliac vein
H Left common femoral artery
30.2 What are the boundaries of the urogenital triangle? What are its contents in the male?
30.2 The anterior vertex is the pubic symphysis and the two other vertices are the ischiopubic rami of the pelvic bone. Its contents in males are the penis and scrotum. In females, the triangle contains the external genitalia, the urethra, and the vagina (Figure 1.11).
30.3 What is the blood supply of the bladder?
30.3 The bladder is supplied by the superior and inferior vesical arteries (internal iliac artery). It drains to the vesical venous plexus (iliac vein).
30.4 Describe the innervation of the bladder.
30.4 The bladder is innervated by sympathetic and parasympathetic fibres. The sympathetic fibres originate at L1–2. These fibres inhibit contraction of the detrusor and stimulate tonic contraction of the internal urethral sphincter. The parasympathetic preganglionic fibres originate at S2–4 as the pelvic splanchnic nerves, and synapse with postganglionic neurones in the bladder wall. Most of the afferent impulses (initiated by stretch) travel through these fibres. The parasympathetic fibres stimulate contraction of the detrusor and inhibit the internal urethral sphincter. In the continent individual there are fibres originating in the cerebral cortex that inhibit the micturition reflex until it is required.
30.5 What layers, from skin inwards, does a needle pass through when aspirating a hydrocele?
30.5 A mnemonic for the layers of scrotum is: Some Damn Englishman Called It Testes: Skin, Dartos, External spermatic fascia, Cremaster, Internal spermatic fascia, Tunica vaginalis, Testis.
30.6 What is the lymphatic drainage of the testes?
30.6 The lymph drainage of the testis and epididymis is via the spermatic cord and ends at the para-aortic lymph nodes at L1. The scrotum, in contrast, drains to the superficial inguinal lymph nodes.
30.7 What is the origin of the left testicular artery?
30.7 The testicular arteries both branch off from the aorta at L2, they then travel in the inguinal canal to reach the testes. Note that it is the left testicular vein not artery that drains in to the left renal vein, whereas the right joins the inferior vena cava.
30.8 Name the parts of the male urethra. What is its approximate length in the adult male?
30.8 The male urethra is approximately 20 cm long. The prostatic urethra is the area surrounded by prostate. It is about 3 cm and has a central elevated area called the urethral crest, with a depressed area either side termed the prostatic sinuses (into which prostatic ducts empty). The crest has a short tract – the verumontanum – into which opens the prostatic utricle. The ejaculatory ducts open either side of the utricle. The membranous urethra is about 2 cm and traverses the external sphincter urethrae and perineal membrane. The spongy urethra is the area within the corpus spongiosum of the penis.
30.9 Is ejaculation mediated by the sympathetic or parasympathetic nervous system?
30.9 Ejaculation is a sympathetic process, whereas erection is parasympathetic. A useful mnemonic is Point = Parasympathetic, Shoot = Sympathetic.
30.10 Which epithelium lines the prostatic urethra?
30.10 Transitional cell epithelium.
 
Station 31
A 75-year-old woman presents to the surgical clinic with rectal prolapse that occurs during defecation. You assess her using your knowledge of the anatomy of the pelvis and rectum.
The image on the next page is a sagittal prosection of the left hemipelvis of a normal subject.
33
zoom view
31.1 Identify the structures labelled A to I.
31.1 A Piriformis
B Coccygeus and sacrospinous ligament
C Obturator internus
D Internal pudendal artery
E Left common iliac
F L5 vertebral body
G Obturator nerve
H Anterior trunk of internal iliac
I External iliac artery
31.2 What is the origin and insertion of the muscle labelled A?
31.2 The piriformis (A) originates from the anterior surface of the lateral mass of the sacrum. Its tendon traverses the greater sciatic foramen to insert in to the upper border of the greater trochanter. It is innervated by the nerve to the piriformis (L5–S2) and is an external rotator of the hip.
31.3 What are the origin, insertion, action and nerve supply of C?
31.3 The obturator internus (C) originates from the inner surface of the anterolateral wall of the pelvis and the obturator membrane. It inserts in to the greater trochanter. The muscle is innervated by the nerve to obturator internus (sacral plexus), and is a lateral rotator of the femur.
31.4 From which nerve roots does the lumbar plexus arise?
31.4 L1–L4.
31.5 What branches of the lumbar plexus emerge from the lateral border of the psoas?
31.5 All of the branches of the lumbar plexus arise from the lateral border of the psoas (iliohypogastric nerve, ilioinguinal nerve, lateral cutaneous nerve of the thigh, femoral nerve) except for the genitofemoral nerve (anterior aspect), and obturator nerve (medial border).
31.6 What are the root values for the femoral nerve?
31.6 L2–L4.
31.7 What are the root values for the sacral plexus?
31.7 L4–S4.
31.8 Describe the course and relations of the right and left lumbar sympathetic chains.
31.8 The lumbar sympathetic chain is a continuation of the thoracic chain as it passes under the medial arcuate ligament of the diaphragm and travels on the lumbar vertebral bodies. On the left, it runs posterolateral to the aorta, on the right underneath the inferior vena cava. They converge on the coccyx at a structure known as the ganglion impar.
31.9 What are the indications for lumbar sympathectomy?
31.9 Lumbar sympathectomy is performed for patients with non-reconstructible arterial disease or vasospastic conditions of the lower limbs. It involves excision of a variable number of the L1–L2 ganglia to denervate the sympathetic supply to the leg and hence increase its blood supply.
31.10 From where does the parasympathetic supply of the abdomen originate?
31.10 The vagus nerve is the main parasympathetic nerve of the abdominal organs. It supplies the gastrointestinal tract as far as the proximal transverse colon. There is also parasympathetic supply from S2–S4 in the form of the pelvic splanchnic nerves. These supply the distal transverse colon as well as the rectum, internal anal sphincter, bladder wall, internal vesicle sphincter, penis and clitoris.
 
Station 32
A 45-year-old male sushi chef presents with weight loss and epigastric pain. He has a family history of stomach cancer and is worried that this might be the diagnosis. You request urgent outpatient radiological investigations.
The image on the next page is a contrast study of a normal stomach:
32.1 What type of study is this?
32.1 The image displays a barium meal.
32.2 Identify the parts of the stomach labelled A, B, C, D, F and G.
32.2 A Lesser curve
B Pylorus
C Antrum
D Fundus
F Greater curve
G Body
32.3 What do the vertical lines at point E indicate?
32.3 E Stomach rugae. These are longitudinal folds in the mucous membrane of the stomach that flatten out when the stomach distends.
32.4 Describe the arterial supply of the stomach.
32.4 All of the arteries that supply the stomach are derived from the coeliac axis (Figure 1.12). The left gastric is the only direct branch of the axis, and passes upwards and to the left to reach the oesophagus (which it also supplies) before descending along the lesser curvature. The right gastric arises from the common hepatic artery and runs up the lesser curvature. The short gastric arises from the splenic artery at the hilum of the spleen and travels in the gastrosplenic ligament to supply the upper greater curvature. The left gastroepiploic also originates from the splenic artery and travels in the greater omentum to supply the greater curvature. The right gastroepiploic is a branch of the gastroduodenal artery (which in turn comes off the hepatic artery). It supplies the inferior greater curvature.
32.5 Describe the innervation of the stomach.
zoom view
32.5 Sympathetic fibres arise from the coeliac plexus. They carry afferent pain fibres, cause reduction in secretory and motor function, and cause constriction of the pylorus. The parasympathetic fibres arise from the vagus nerves. Parasympathetic fibres are secretomotor to the stomach and cause relaxation of the pylorus. The left vagus nerve forms the anterior vagal trunk and enters the abdomen on the anterior surface of the oesophagus. It gives off branches to the anterior stomach wall, the liver, and the pylorus of the stomach. The posterior vagal trunk enters the abdomen on the posterior surface of the oesophagus and supplies the posterior wall of the stomach. The posterior trunk also gives off branches to the coeliac and superior mesenteric plexuses to supply the pancreas and the colon as far as the splenic flexure.
3432.6 What are the consequences of highly selective vagotomy?
32.6 Highly selective vagotomy is division of those branches of the anterior and posterior vagus nerves that supply the acid-secreting body of the stomach. The nerve of Latarjet is preserved, maintaining function of the pyloric antrum.
32.7 What are the anterior and posterior relations of the stomach?
32.7 The anterior relations of the stomach are: the anterior abdominal wall, the left costal margin, the left pleura and lung, the diaphragm, and the left lobe of the liver. The posterior relations are: the lesser sac, the spleen and splenic artery, the pancreas, the left suprarenal gland, the left kidney, and transverse colon and mesocolon.
32.8 What anatomical landmarks demarcate the duodenum from the stomach?
32.8 The junction of pylorus of the stomach from the duodenum is marked by an external constriction and the constant vein of Mayo.
32.9 Describe the cardiac sphincter of the stomach.
32.9 The cardiac sphincter is a physiological rather than anatomical sphincter. Tonic constriction of the circular layer of smooth muscle at this level prevents gastric contents from regurgitating upwards. It relaxes ahead of peristaltic waves caused by the swallowing of food. There are mucosal folds at the junction which act as valves, and the right crus of the diaphragm also exerts external pressure.
32.10 Describe the lymphatic drainage of the stomach.
32.10 The lymph drainage follows the arterial supply. The superior two-thirds of the stomach drain along the left and right gastric vessels. The right greater curvature of the stomach drains along the right gastroepiploic arteries to the subpyloric nodes. The left part of the greater curve drains alongside the short gastric and splenic vessels to the suprapancreatic nodes. All lymph eventually passes to the coeliac nodes.
 
Station 33
A 35-year-old man presents to the hospital with left upper quadrant pain following a game of rugby, where he thinks he may have been elbowed in the stomach. On questioning he admits to having had a sore throat for the past week. You assess him and request a CT scan as you think he may have ruptured his spleen.
The images below are contrast-enhanced axial CT slices taken at the level of the spleen in two different normal subjects. Both images demonstrate normal anatomy. Image (a) has been acquired during the venous phase and image (b) has been acquired during the arterial phase.
zoom view
3533.1 Identify the structures labelled A to E.
33.1 See Figure 1.20 demonstrating the vessels within the splenic hilum.
A Portal vein origin (confluence of the splenic and superior mesenteric veins)
B Splenic vein
C Spleen
D Origin of the coeliac axis
E Splenic artery
Note how the density of the splenic artery mimics that of the aorta in an arterial phase scan. This knowledge helps you identify that the vessel here labeled ‘E’ is an artery and not a vein.
33.2 Which pathogen causes upper respiratory tract infection and also predisposes the individual to splenic rupture?
33.2 The Epstein–Barr virus causes glandular fever and is associated with splenomegaly.
33.3 In what direction does a spleen enlarge and why?
33.3 The diaphragm ensures that the spleen enlarges downwards, but the left colic flexure and phrenicocolic ligament direct the spleen medially. The notched anterior border of the spleen is palpable as it projects below the costal margin.
33.4 What organs are in direct contact with the spleen?
33.4 Posteriorly: the diaphragm (behind which is the pleura, left lung and 9–11th ribs).
Anteriorly: the stomach, the tail of the pancreas.
Inferiorly: the splenic flexure of colon.
Medially: the left kidney.
33.5 Is the spleen intraperitoneal or retroperitoneal?
33.5 The spleen is intraperitoneal (and hence mobile).
33.6 What splenic ligaments must be cut during splenectomy?
33.6 Splenorenal (or lienorenal), gastrosplenic, splenocolic, and splenophrenic ligaments (or we also accept: ‘all of them’).
33.7 What are the branches of the splenic artery?
33.7 The splenic artery originates from the coeliac axis (see Figure 1.12).
zoom view
Fig. 1.20: The spleen.
It runs a tortuous course along the upper border of the pancreas giving off multiple branches to the pancreas (the largest of which is the arteria pancreatica magna), the short gastric artery, the left gastroepiploic artery, and the posterior gastric artery.
33.8 Describe the functions of the spleen.
33.8 In the foetus the spleen has haematopoietic properties up until the 5th month of gestation. In the adult it has immune functions (via humoral and cell-mediated pathways) and filters red blood cells.
33.9 What embryological structure does the spleen develop from?
33.9 The spleen develops as multiple thickenings of mesenchyme in the dorsal mesentery. In most people these masses fuse, although approximately 10% of people have more than one spleen.
33.10 What is the innervation of the adrenal glands?
33.10 The medulla of the adrenal gland receives preganglionic sympathetic fibres from the greater splanchnic nerve. It can be considered a specialised sympathetic ganglion, except that it releases its adrenergic secretions directly into the bloodstream. The cortex is regulated by hormones from the pituitary and hypothalamus, well as the renin–angiotensin system.
33.11 Name the different macroscopic regions of the adrenal gland and their respective functions.
33.11 The adrenal glands have an outer yellow cortex, and a dark brown inner medulla. The medulla secretes adrenaline and noradrenaline in response to sympathetic stimulation. The cortex produces corticosteroid hormones and is further divided in to a zona glomerulosa (producing mineralocorticoids), zona fasciculata (producing cortisol), and zona reticularis (producing androgens).
 
Station 34
A 47-year-old man undergoes a liver biopsy for investigation of jaundice. Twenty-four hours later he presents to the emergency department in shock, complaining of right upper quadrant pain. You suspect that he may be bleeding from his biopsy site.
The following image is an axial dissection obtained at the level of the hepatic hilum (demonstrating normal anatomy):
zoom view
3634.1 Identify the structures labelled A to E.
34.1 A Common hepatic artery
B Common hepatic duct
C Portal vein
D Inferior vena cava
E Tail of the pancreas
34.2 What anatomical layer, related to the liver, may help contain hepatic bleeding?
34.2 The liver has a connective tissue layer, Glisson's capsule, which covers its surface and invests its blood vessels. Bleeding from the liver can be contained within this capsule, although this may rupture and blood can leak in to the peritoneal cavity.
34.3 Describe the functional divisions of the liver.
34.3 The gross liver can be divided into left and right segments by the attachments of the falciform ligament, ligamentum teres, and ligamentum venosum. However, it is functionally divided by a plane that passes through the gallbladder and the inferior vena cava fossae. These functional lobes have separate blood supply and biliary drainage, and can thus be resected separately. In the Couinaud or ‘French’ system, these functional lobes are divided into a total of eight sub-segments.
34.4 Describe the innervation of the liver.
34.4 The liver receives parasympathetic and sympathetic supply from the coeliac plexus. The anterior vagal trunk also gives off a branch to the liver.
34.5 What are the contents and attachments of the falciform ligament?
34.5 The falciform ligament is a two-layered fold of peritoneum that contains the ligamentum teres (the remnant of the umbilical vein, see Figure 1.16). It attaches the umbilicus to the anterior surface of the liver before splitting in to two layers on its posterior surface. The right layer forms the upper coronary ligament and the left the upper triangular ligament.
34.6 What is the ligamentum venosum?
34.6 The ligamentum venosum is the remnant of the foetal ductus venosus, which shunts blood from the umbilical vein to the inferior vena cava. It adheres to the left branch of the portal vein and travels in a fissure on the visceral surface of the liver to attach superiorly to the inferior vena cava.
34.7 How are the portal vein, hepatic artery, and bile duct related to each other in the free edge of the lesser omentum?
34.7 The bile duct lies anterior and to the right, the hepatic artery lies anterior and to the left, and the portal vein lies posteriorly.
 
Station 35
A 57-year-old male builder is struck by a falling metal girder on a building site. In the emergency department he is complaining of severe pain in the left pelvis, and there is significant bruising in this area.
This is a plain anteroposterior radiograph of the pelvis (demonstrating normal anatomy):
zoom view
35.1 Identify the structures labelled A, B, D, E and F.
35.1 A Wing of the ileum
B Iliopectineal line (with the ischial spine behind it)
D Ala of sacrum
E Coccyx
F Superior pubic ramus
35.2 Name the foramen indicated by C. What structures pass through here?
35.2 C Obturator foramen. The obturator nerve and vessels pass through this space.
35.3 What type of joint is the sacroiliac joint?
35.3 The sacroiliac joint is a synovial plane joint.
35.4 What type of joint is the sacrococcygeal joint?
35.4 The sacrococcygeal joint is a secondary cartilaginous joint.
35.5 What are the boundaries of the pelvic inlet?
35.5 The pelvic inlet (or brim) is bounded anteriorly by the symphysis pubis, laterally by the iliopectineal lines, and posteriorly by the sacral promontory.
35.6 Define the false pelvis.
35.6 The false pelvis is a space within the abdomen bounded posteriorly by the lumbar vertebrae, laterally by the iliac fossae and iliacus muscles, inferiorly by the pelvic inlet, and anteriorly by the anterior abdominal wall.
3735.7 What are contents of the pudendal (Alcock's) canal?
35.7 The pudendal (Alcock's) canal is a fascial space in the lateral wall of the ischioanal fossa containing the pudendal nerve and internal pudendal vessels.
35.8 What lies medial to the pudendal canal, and what lies laterally?
35.8 Medial is the ischioanal fossa, and laterally are the obturator internus and ischial tuberosity.
35.9 What are the clinical consequences of superior hypogastric plexus damage?
35.9 The superior hypogastric plexus is a continuation of the aortic plexus with contributions from the third and fourth lumbar sympathetic ganglia. It lies on the promontory of the sacrum and may be damaged during operations in the pelvis, e.g. open aortic aneurysm repair. Injury results in erectile dysfunction in males and bladder dysfunction in females.
35.10 What are the branches of the pudendal nerve?
35.10 The pudendal nerve is a branch of the sacral plexus. It leaves the pelvis via the greater sciatic foramen, and enters the perineum through the lesser sciatic foramen. Its branches are:
  • inferior rectal nerve: supplies the external anal sphincter, the mucous membrane of the lower half of the anal canal, and the perianal skin.
  • dorsal nerve of the penis/clitoris
  • perineal nerve: has a superficial branch that supplies the skin of the posterior scrotum/labia majora, and a deep branch supplying the muscles of the urogenital triangle.
 
Station 36
A 78-year-old man presents to the urology clinic with a swollen left testicle. On palpation the testicle feels firm, non-tender and enlarged.
Test your knowledge of the regional axial anatomy on the following axial cadaveric dissection done at the level of the testes and penis (note, in this specimen this subject has only one testis).
zoom view
36.1 Identify the structures labelled A to D.
36.1 See Figure 1.21.
A Corpora cavernosa of the penis
B Corporum spongiosum of the penis
C Right testis
D Epididymis
36.2 Trace the pathway of semen from the seminferous tubules to the terminal urethra.
36.2 The seminiferous tubules are located in the lobules of the testis. Each testis has 200–300 lobules and each lobule contains one to three coiled tubules. The tubules drain in to a plexus termed the rete testis, and thence in to efferent ductules. This pierces the tunica albuginea at the upper testis and passes into the head of the epididymis. The efferent ductules coalesce upon a single much coiled tube, which forms the body and tail of the epididymis. This is a 6 metre long tube that allows for storage and maturation of spermatozoa. The tube continues from the tail as the vas deferens, which travels through the inguinal canal in the spermatic cord. It emerges from the deep inguinal ring and then travels downwards and backwards on the lateral wall of the pelvis (intersecting the ureter at the ischial spine), before running medially and downwards on the posterior bladder. The final part of the vas forms an ampulla before combining with the duct of the seminal vesicle to form the ejaculatory duct. The two ejaculatory ducts pierce the posterior surface of the prostate to open in to the prostatic urethra either side of the prostatic utricle.
36.3 What is the blood supply to the testis?
36.3 The testis and epididymis are supplied by the testicular arteries, which come off the aorta at L2 and pass through the inguinal canal. The venous drainage is the testicular veins, via the pampiniform plexus. The right vein drains in to the inferior vena cava, and the left to the left renal vein.
zoom view
Fig. 1.21: The testis.
36.4 What are the functions of the Leydig and Sertoli cells of the testis?
36.4 The Sertoli cells form the epithelium of the seminiferous tubules. Their function is to nurture developing sperm cells through spermatogenesis. The cell is activated by follicle-stimulating hormone (FSH) and in turn secretes a number of hormones and proteins. Leydig cells are interstitial cells that produce androgens in response to luteinizing hormone (LH).
36.5 List the possible locations for an undescended or incompletely descended testis.
36.5 The testes develop on the posterior abdominal wall and descend during the latter stages of pregnancy, explaining the distant origin of their vascular and nervous supply. If descent is incomplete they may be found at any point along this path: the abdomen, the inguinal canal, superficial ring, or high in the scrotum.
36.6 What is a varicocele? On which side are they most common?
36.6 A varicocele is a dilatation of the pampiniform plexus. The majority of varicoceles occur on the left side and this is probably due to the testicular vein on the left side entering the left renal vein rather than the inferior vena cava.
36.7 What is a hydrocele? How may peritoneal fluid enter the scrotum in some individuals?
36.7 A hydrocele is fluid within the tunica vaginalis, and may be associated with a patent processus vaginalis. The processus vaginalis is an embryological outpouching of peritoneum in to the scrotum, and surrounds the front and sides of the testis. The processus normally closes soon after birth. A persistent patent processus vaginalis allows for fluid and peritoneal contents to travel in to the scrotum. Persistency is more common on the right side. Hydroceles can also be caused by inflammation of the testis without communication to the peritoneal cavity.
36.8 What is the clinical significance and embryological origin of the appendix testis (hydatid of Morgagni)?
36.8 Both the testis and epididymis have appendages. The former is derived from the paramesonephric (Müllerian) ducts, and the latter from the mesonephric tubules. The testicular appendage is called the hydatid of Morgagni, and is present in most individuals. As these appendages exist on stalked bodies, they may both undergo torsion. This, in itself, is not a problem, but the clinical presentation may mimic torsion of the testes. Clinically a ‘blue dot’ is visible through the scrotal skin.
36.9 What is Fournier's gangrene?
36.9 Fournier's gangrene is a necrotising infection of the perineum and associated structures. The condition is associated with diabetes and immunosuppression, and a mixture of both aerobic and anaerobic organisms is usually responsible. It can be rapidly spreading and requires urgent admission, antibiotics and debridement.
38
 
Station 37
A 72-year-old man presents with frequency of micturition and nocturia. On rectal examination the left lobe of his prostate is enlarged and hard. He has a prostate specific antigen of 20 ng/mL. You suspect prostate cancer and request an MRI.
This is a sagittal MRI of a normal male pelvis:
zoom view
37.1 Identify the structures labelled A to E.
37.1 A Bladder
B Pubic symphysis
C Penis
D Prostate
E Rectum
37.2 What does ‘BPH’ stand for?
37.2 BPH stands for benign prostatic hyperplasia (not hypertrophy). Hyperplasia is the abnormal but benign proliferation of cells of the same type. It is often a response to a specific external stimulus. Hypertrophy, in contrast, is a benign increase in the size of the cells.
37.3 Which zone of the prostate becomes enlarged in BPH?
37.3 The transition zone is affected in benign prostatic hyperplasia, compressing the surrounding peripheral zone. The zones of the prostate are outlined in Table 1.9.
37.4 In which zone of the prostate does cancer most commonly occur?
37.4 Prostate carcinoma usually occurs in the peripheral zone.
37.5 What is the arterial supply to the prostate?
37.5 The prostate is supplied by prostatic branches of the inferior vesical arteries (both ultimately originating from the internal iliac artery).
37.6 By what route may prostatic metastases travel to the spine?
37.6 Cancer can spread via the prostatic venous drainage. The veins of the prostate form a venous plexus in front of the vertebral bodies, outside of the prostatic capsule, before draining in to the internal iliac veins. There are connections between the prostatic venous plexus and the vertebral veins. The veins in the plexus do not have valves, and therefore during periods of raised abdominal pressure (e.g. coughing or straining) the direction of flow may be directed in to the vertebrals, allowing seeding to the vertebral bodies.
37.7 What is the seminal colliculus (verumontanum)?
37.7 The seminal colliculus or verumontanum is an elevation of the posterior wall of the prostatic urethra in the middle of the urethral crest. At its margins open the prostatic utricle and the ejaculatory and prostatic ducts. During transurethral resection of the prostate the surgeon works above this level to avoid damage to the urethral sphincter.
37.8 What is the prostatic utricle? What is its embryological origin?
37.8 The prostatic utricle is a blind ending pouch on the posterior wall of the prostatic urethra at the apex of the urethral crest, on the seminal colliculus (verumontanum). It is derived from the paramesonephric (Müllerian) duct, which in the female becomes the fallopian tubes, uterus and upper vagina.
37.9 Describe the structure of the body of the penis. In which part does the urethra run?
37.9 The body of the penis comprises two dorsal corpora cavernosa and a ventral corpus spongiosum. The corpus spongiosum expands distally to form the glans penis. The penis is enclosed in Buck's fascia and has a foreskin that is connected to the glans penis by the frenulum. The urethra travels within the corpus spongiosum.
37.10 What is the arterial supply to the penis?
37.10 The arterial supply is the internal pudendal artery (internal iliac). The corpus spongiosum is supplied by the artery of the bulb, the corpora cavernosa are supplied by the deep arteries of the penis, and the sheath of the corpora cavernosa are supplied by the dorsal artery of the penis.
Table 1.9   The zones of the prostate
Central
Wedge shaped region that surrounds the ejaculatory ducts (extends from bladder base to the veru)
< 5% of prostatic cancer
Peripheral
Posterolateral part of the prostate, surrounding the central zone
70% of prostatic cancer
Transition
Surrounds the prostatic urethra proximal to the veru
  • 20% of prostatic cancer
  • Benign prostatic hyperplasia arises here
Anterior fibromuscular zone (or stroma)
Anterior region composed of fibrous and muscular tissue
Not affected by cancer
The veins of the penis drain to the internal pudendal vein.
 
Station 38
A 25-year-old woman suffers from acute right iliac fossa pain and undergoes laparoscopy. The appendix is seen to be normal. However there is a large haemorrhagic ovarian cyst on the right side.
39The image below is an axial dissection through a normal adult female pelvis:
zoom view
38.1 Identify the structures labelled A to E.
38.1 A Right ureter
B Bladder
C Uterus
D Rectum
E Levator ani
38.2 What is the blood supply to the ovaries?
38.2 The ovary is supplied by the ovarian artery, which branches from the abdominal aorta at L1. The corresponding veins drain in to the inferior vena cava on the right, and the left renal vein on the left. This is identical to the blood supply of the testicles.
38.3 What is the lymphatic drainage of the ovaries?
38.3 The lymph drainage follows the arterial supply into paraaortic nodes at the L1 level.
38.4 In which structure do the vessels and nerves of the ovary run?
38.4 The neurovascular and lymphatic structures travel via the suspensory ligament (also known as the infundibulopelvic ligament) of the ovary and thence through the mesovarium, to enter the hilum of the ovary.
38.5 Which ligaments are connected to the ovary?
38.5 The ovary is attached to the following ligaments:
  • the mesovarium: attaches the ovary to the broad ligament of the uterus
  • the round ligament of the ovary: connects the ovary to the lateral margin of the uterus.
  • the suspensory ligament of the ovary: connects the mesovarium to the lateral wall of the pelvis.
38.6 What is the name of the layer which envelopes the ovary?
38.6 The ovary (and testis) is surrounded by the tunica albuginea.
38.7 Which tumour marker rises in ovarian cancer?
38.7 Ca125.
 
Station 39
A 47-year-old man presents with pain in the left upper medial thigh 6 months after undergoing an inguinal hernia repair. The consultant suspects that the ilioinguinal nerve was damaged during the operation.
The prosection on the next page displays abdominal contents and branches of the lumbar plexus in a normal subject.
39.1 Identify structures A to D.
39.1 A Left ureter
B Psoas major
C Superior hypogastric plexus
D Left hypogastric nerve
39.2 Where is the lumbar plexus located?
39.2 The lumbar plexus is formed in the psoas muscle from the anterior rami of L1–L4.
39.3 What does the genitofemoral nerve supply?
39.3 The genitofemoral nerve (L1, L2) supplies the cremaster muscle of the scrotum and scrotal skin (genital branch) and the skin of the anterior thigh (femoral branch) (Table 1.10).
Table 1.10   The nerves of the lumbar plexus
Nerve
Motor supply
Sensory supply
Iliohypogastric (T12–L1)
  • External oblique
  • Internal oblique
  • Transversus abdominis
  • Lower abdominal wall
  • Buttock
Ilioinguinal (L1)
  • External oblique
  • Internal oblique
  • Transversus abdominis
  • Upper medial thigh
  • Root of penis and scrotum
  • Mons pubis
  • Labia majora
Genitofemoral (L1–L2)
Cremaster muscle
Anterior thigh
Lateral cutaneous nerve of the thigh (L2–L3)
None
Anterior and lateral thigh
Obturator (L2–L4)
  • Obturator externus
  • Gracilis, adductor brevis
  • Adductor longus
  • Pectineus
  • Adductor magnus (adductor part)
Medial thigh
Femoral (L2–L4)
  • Iliacus, sartorius
  • Pectineus
  • Quadriceps femoris
  • Anterior thigh
  • Medial leg and foot (saphenous branch)
Muscular branches
  • Psoas
  • Quadratus lumborum
  • Iliacus
None
39.4 What does the ilioinguinal nerve supply?
39.4 The ilioinguinal nerve supplies the external oblique, the internal oblique, transversus abdominis, the skin of the upper medial thigh, the base of the penis and scrotum, the mons pubis and the labia majora.
4039.5 Describe the cremasteric reflex.
39.5 The cremasteric reflex can be elicited by lightly stroking the medial thigh. The afferent limb of the reflex is the femoral branch of the genitofemoral nerve. This causes contraction of the cremaster via the efferent limb, which is the genital branch of the genitofemoral nerve. The function of the cremasteric reflex is to raise the testis for warmth and protection. It may be absent in testicular torsion.
39.6 Where does the sympathetic trunk enter the abdomen?
zoom view
39.6 The sympathetic trunk enters the abdomen behind the medial arcuate ligament.
41
 
Answers
Station 1
1.1 Identify the bony landmarks labelled B, C and D.
1.1 B Costal groove
C Head
D Tubercle
1.2What does A articulate with?
1.2 The costal cartilages.
1.3 What runs in the groove indicated by B?
1.3 The neurovascular bundle accompanying the ribs runs in the subcostal groove in-between the internal and innermost intercostal muscles (the order of Veins, Arteries and Nerves from superior to inferior can be remembered by the mnemonic VAN).
1.4 What is the arterial supply of the intercostal muscles of the second and tenth intercostal spaces?
1.4 The muscles of rib spaces 1–9 are supplied by the posterior and anterior intercostal arteries, whereas those of rib spaces 10 and 11 have only posterior arteries. The posterior intercostal arteries of the first two rib spaces come from the superior intercostal branch of the costocervical trunk, whereas the posterior intercostal arteries of the bottom nine come directly off the aorta (there are only 11 rib spaces). The anterior intercostal arteries are branches of the internal thoracic arteries and its branches.
1.5 Where do the intercostal veins drain?
1.5 The anterior intercostal veins drain into the internal thoracic and musculophrenic veins. The drainage of the posterior intercostal veins is more complicated. These drain into the azygos, hemiazygos or accessory hemiazygos veins with the following exceptions: the 1st posterior vein (the supreme intercostal vein) drains into the ipsilateral brachiocephalic or vertebral veins; the left 2nd, 3rd and 4th veins join to form a superior intercostal vein, which drains into the left brachiocephalic vein.
1.6 What is meant by the term ‘flail chest’?
1.6 The term ‘flail chest’ describes a scenario in which a section of chest wall is disconnected from its surrounding bony skeleton by multiple rib fractures. This can occur unilaterally (where the ribs often fracture both at the angle and near the costochondral junction), or bilaterally (where the sternum itself can be flail).
1.7 What type of joint is the first costochondral joint?
Image (b) below shows the anterior aspect of the sternum:
zoom view
1.7 A costochondral joint is a primary cartilaginous joint, with the costal hyaline cartilage connecting directly with the rib without any intervening fibrous tissue. As with all primary cartilaginous joints virtually no movement occurs at the costochondral joint.
1.8 Identify the parts labelled A, B, C, and D.
1.8 A Manubrium
B Body
C Xiphisternum
D Jugular notch
1.9 Name the structure that articulates with the sternum at E.
1.9 1st rib
42
Station 2
2.1 Which lung lobes are auscultated at sites labelled A to E?
2.1 See Figure 1.1.
A Upper lobe
B Middle lobe
C Upper lobe
D Upper lobe
E Lower lobe
2.2 Describe the surface marking of the pleural edges in terms of their relations to the thoracic skeleton.
2.2 The pleural edge extends from the junction between the middle and medial thirds of the clavicle, to an apex about 2.5 cm above the medial end of the clavicle, and then down to the sternoclavicular joint.
zoom view
Fig. 1.1: (a) Anterior thoracic wall and lobes of the lung. (b) Posterior thoracic wall and lobes of the lung.
43It then meets the pleura of the contralateral side in the midline at the level of the 2nd costal cartilage. The right pleural edge extends down to the level of the 6th costal cartilage. It then turns laterally crossing the 8th rib in the midclavicular line and the 10th rib in the midaxillary line; it then meets the 12th rib at the lateral border of erector spinae. On the left side the heart, at the level of the 4th costal cartilage, reflects the pleura laterally but otherwise the pleural reflexion follows a similar course to that on the right side.
2.3 Describe the surface marking of the lung edges in terms of their relations to the thoracic skeleton.
2.3 The apex of the lung follows the pleura. The lower border of the lung is parallel to the line of pleural reflexion but two ribs above: thus the lower border of the lung crosses the 6th rib in midclavicular line, the 8th rib in the midaxillary line, and the 10th rib adjacent to the vertebral column posteriorly.
2.4 What are the surface markings for:
2.4a the oblique fissure of the lungs?
2.4a The oblique fissure divides the upper and lower lobes. For the most part, it corresponds to the line of the 5th rib. It may be indicated on the surface as a line running obliquely downwards and outwards from just lateral to the spine of the 3rd thoracic vertebra to the 6th costal cartilage 4 cm from the midline. With the shoulders abducted fully this line corresponds to the medial border of the scapula.
2.4b the transverse fissure of the lungs?
2.4 b The horizontal (transverse) fissure divides the middle from the upper lobe of the right lung. It follows a line along the 4th costal cartilage to meet the oblique fissure where it crosses the 5th rib near the midaxillary line.
2.5 Which costal cartilage attaches to the sternum at the sternal angle?
2.5 The 2nd costal cartilage articulates with the lateral aspect of the manubriosternal junction.
2.6 At which vertebral level is the suprasternal notch?
2.6 The suprasternal notch lies at the level of the T2/T3 intervertebral disc.
2.7 At which vertebral level is the xiphisternal joint?
2.7 The xiphisternal joint lies at the level of T9.
2.8 What are the boundaries of the superior mediastinum?
2.8 The superior mediastinum is bounded anteriorly by the manubrium, laterally by the pleurae, posteriorly by the T1–T4 vertebral bodies, superiorly by the superior thoracic aperture, and inferiorly by the plane of Louis (transverse thoracic plane at the T4/T5 intervertebral disc).
2.9 What are the contents of the superior mediastinum?
2.9 The superior mediastinum contains the great vessels (aortic arch, brachiocephalic artery and veins, left common carotid and subclavian arteries, superior vena cava), trachea, oesophagus, remains of the thymus, thoracic duct, right and left vagi, left recurrent laryngeal nerve and right and left phrenic nerves, and very importantly, lymph nodes.
Station 3
3.1 Identify the structures labelled A, B, C.
3.1 A Ascending aorta
B Descending aorta
C Pulmonary trunk
3.2 Classify the divisions of the mediastinum.
3.2 The mediastinum is the space in the thoracic cavity between the right and left pleural sacs. It is conventionally divided into a superior mediastinum and inferior mediastinum by an imaginary plane plotted perpendicular to the sternum at the plane of Louis (Figure 1.2). The inferior mediastinum is further subdivided into anterior, middle, and posterior mediastina by the fibrous pericardium.
44
zoom view
Fig. 1.2: Divisions of the mediastinum.
3.3 Through which division of the mediastinum is this slice taken?
3.3 It shows the inferior mediastinum. This requires knowledge of the contents of each division of the mediastinum.
  • The superior mediastinum contains the great vessels (aortic arch, brachiocephalic artery and veins, left common carotid and subclavian arteries, superior vena cava), lymph nodes, trachea, oesophagus, remains of the thymus, thoracic duct, vagi, left recurrent laryngeal nerve and phrenic nerves.
  • The anterior mediastinum contains the remains of the thymus and branches of the right and left internal thoracic arteries.
  • The middle mediastinum contains the heart inside the pericardium, the ascending aorta, the superior vena cavae, the bifurcation of the trachea, the pulmonary arteries and veins, the phrenic nerves and pericardiophrenic vessels.
  • The posterior mediastinum contains the descending thoracic aorta and its branches, the azygos/hemiazygos/accessory hemiazygos veins, the right and left vagus nerves, the right and left splanchnic nerves, the oesophagus, and the thoracic duct. The ganglionated thoracic sympathetic chains may also be regarded as contents of the posterior mediastinum.
3.4 What are the boundaries of this division of the mediastinum?
3.4 This, the inferior mediastinum, is bounded anteriorly by the body of the sternum (mesosternum), laterally by the pleurae, posteriorly by the T5–T12 vertebral bodies, inferiorly by the diaphragm, and superiorly by the plane of Louis.
3.5 What is the innervation of the parietal and visceral pleura?
3.5 The parietal pleura are sensitive to pain, temperature, touch and pressure. Its innervation depends on region: intercostal nerves supply the costal pleura; the phrenic nerve supplies the mediastinal pleura; and the phrenic nerve and lower six intercostal nerves supply the diaphragmatic pleura. The visceral pleura are sensitive to stretch and receive their sensory innervation from the autonomic pulmonary plexus (formed from branches of the thoracic sympathetic trunk and vagus nerve).
3.6 Name some of the important functions of the thoracic sympathetic chain, and state which spinal cord segments contribute to the sympathetic chain?
3.6 The thoracic sympathetic chain has three main branches. It supplies sympathetic fibres to the skin, postganglionic fibres from T1–T5 to the thoracic viscera, and mostly preganglionic fibres from T5–T12 to supply the abdominal viscera (in the 45form of the greater splanchnic, lesser splanchnic and least splanchnic nerves). The chain receives preganglionic white ramus communicans from each spinal nerve to a corresponding ganglion, and gives back a grey ramus containing postganglionic fibres.
3.7 Define thoracic outlet syndrome.
3.7 Thoracic outlet syndrome is caused by compression at the superior thoracic aperture of neurovascular structures passing above the first rib, either between the anterior and middle scalene muscles or in front of scalenus anterior. It can affect the brachial plexus (most commonly, lower trunk of the brachial plexus) or subclavian artery/vein. A rare cause of thoracic outlet syndrome is a cervical rib or a cervical band of fibrous tissue. The syndrome manifests most commonly in the hands with pain, weakness, and coldness. Note that the superior thoracic aperture is usually referred to by clinicians as the thoracic outlet but by anatomists as the thoracic inlet!
3.8 Define subclavian steal syndrome.
3.8 Subclavian steal occurs when blood flows in a retrograde direction in the vertebral artery in association with proximal ipsilateral subclavian artery stenosis or occlusion. This blood is ‘stolen’ from the circle of Willis via the ipsilateral vertebral artery. Patients with retrograde flow are usually asymptomatic but they may develop dizziness, vertigo, syncope, dysarthria, and visual symptoms. There is usually a drop in blood pressure in the ipsilateral arm distal to the stenosis.
Station 4
4.1 Identify the structures labelled A to E.
4.1 A Right coronary artery
B Left atrium
C Descending thoracic aorta
D Left anterior descending artery (anterior interventricular artery)
E Left circumflex artery
4.2 What is the origin of the right coronary artery? Name the branches of the right coronary artery.
4.2 The right coronary artery (Figure 1.3) originates from the anterior aortic sinus of the ascending aorta. It has the following branches:
  • The anterior ventricular branches supply the anterior surface of the right ventricle.
    zoom view
    Fig. 1.3: Arterial supply of the heart.
    46
  • The marginal artery is a branch of the anterior ventricular and runs towards the apex.
  • The posterior ventricular branches supply the diaphragmatic surfaces of the right ventricle.
  • The posterior interventricular artery supplies the right and left ventricles and runs in the posterior interventricular groove.
  • The atrial branches supply the right atrium.
4.3 What is the origin of the left coronary artery? Name the branches of the left coronary artery.
4.3 The left coronary artery originates from the left posterior aortic sinus of the ascending aorta. Its branches are:
  • The anterior interventricular artery (also known as left anterior descending artery). This is the major branch of the left coronary artery and supplies the anterior aspect of both ventricles and the anterior half of the interventricular septum before proceeding to anastomose with the posterior interventricular branch of the right coronary artery.
  • The circumflex artery is the continuation of the left coronary artery after the anterior interventricular is given off. It winds around the left margin of the heart in the atrioventricular groove.
  • The left marginal artery is a branch of the circumflex.
  • The anterior/posterior ventricular arteries are branches of the circumflex.
  • The atrial branches are also branches of the circumflex.
4.4 Which coronary artery most commonly supplies:
4.4a the sinoatrial node?
4.4a The sinoatrial node is supplied by the right coronary artery in about two-thirds of people, and by the left in approximately one-third.
4.4b the atrioventricular node?
4.4b The atrioventricular node is supplied by the posterior interventricular branch of the right coronary artery in over 90% of individuals and less commonly by the left coronary artery via its circumflex branch.
4.5 Describe the venous drainage of the heart.
4.5 The coronary veins drain in to the coronary sinus, which is located in the posterior atrioventricular groove. The coronary sinus drains into the right atrium. The major veins it receives are the great cardiac vein (in the anterior interventricular groove), the middle cardiac vein (in the posterior interventricular groove), the small cardiac vein (running along the lower border of the heart), and the cardiac oblique vein (on the posterior surface of the left atrium). The anterior cardiac veins drain the anterior surface of the heart and empty anteriorly into the right atrium directly. There are numerous much smaller veins emptying directly into the chambers that they overlie.
4.6 Describe the conducting system of the heart.
4.6 The sinoatrial node is located in the wall of the right atrium, to the right of the opening of the superior vena cava. It generates rhythmic electrical impulses that radiate out throughout the atrial muscle, causing contraction. The atrioventricular node is in the ventricular end of the atrial septum and conducts the atrial impulse to the ventricles, via the atrioventricular bundle of His. The time that it takes for this conduction (about one tenth of a second) allows the atria to empty their blood into the ventricles. The bundle of His divides into two branches, one for each ventricle. The right bundle travels down on the right side of the interventricular septum to reach the anterior wall of the right ventricle and becomes continuous with the Purkinje plexus of the right ventricle. The left bundle divides into anterior and posterior branches, and these fibres are continuous with the Purkinje plexus of the left ventricle. These fibres induce contraction of the ventricles.
474.7 Describe the efferent nerve supply to the heart.
4.7 The parasympathetic supply of the heart is the vagus nerve. The heart's sympathetic supply is the cervical and upper thoracic sympathetic trunk. The cardiac plexuses are located below the arch of the aorta and transmit all of the heart's autonomic fibres. Sympathetic stimulation increases the force and rate of contraction and dilates the coronary arteries. Parasympathetic stimulation decreases the force and rate of contraction and constricts the coronary arteries.
4.8 Why is the pain of ischaemic heart disease referred to the chest wall?
4.8 Afferent fibres from the heart run with sympathetic fibres and enter the spinal cord through the posterior roots of T1–T4. The pain of ischaemia is referred to the skin areas of the skin supplied by the corresponding spinal nerves, i.e. the upper four intercostal nerves and the intercostobrachial nerve. This territory is the left chest wall and the upper part of the left arm.
Station 5
5.1 Where would you insert a cannula for emergency decompression of tension pneumothorax? What layers does this needle pass through?
5.1 Tension pneumothorax requires emergency decompression with a 14–16 gauge needle in the 2nd intercostal space in the midclavicular line. The needle passes through skin, superficial fascia and fat, pectoralis major, external intercostal, internal intercostal, innermost intercostal, and parietal pleura.
5.2 What is the ‘safe triangle’ of chest drain insertion?
5.2 The safe triangle is made up of the lateral border of the pectoralis major, the anterior border of latissimus dorsi, and the upper border of the 6th rib (about the level of the nipple), with the apex slightly below the axilla.
5.3 The point marked B is located within which dermatome?
5.3 T6 or T7 (Figure 1.4)
zoom view
Fig. 1.4: Trunk dermatomes.
5.4 What is the direction of relaxed skin tension lines at point A?
485.4 At this point the lines are not horizontal, but are actually directed towards the axillary skin crease.
5.5 What is the surface marking for:
5.5a the entry point of the needle for subclavian vein catheterisation?
5.5a The target is 1 cm inferior to the junction of the middle and distal third of the clavicle. The tip should be directed towards the sternal notch.
5.5b the entry point of the needle for internal jugular vein catheterisation?
5.5b With the patient's head turned away from the insertion site, the target is the apex of the triangle formed by the sternal and clavicular heads of the sternocleidomastoid muscle. The needle should be inserted at a 30° angle to the skin directed towards the ipsilateral nipple.
5.5c a posterolateral thoracotomy incision for exposure of upper thoracic structures?
5.5c The site is somewhat dependent on what access is necessary, but the incision usually lies at the level of the 5th rib for upper thoracic structures (for exposure of lower structures the incision is at the 6th or 7th rib). The incision is started at a point midway between the medial border of the scapula and the thoracic spine. The incision curves about 3 cm below the inferior angle of the scapula and turns to run parallel with the rib.
5.6 At which vertebral level is:
5.6a the sternal angle (plane of Louis)?
5.6a The intervertebral disc between T4 and T5.
5.6b the bifurcation of the trachea?
5.6b The normal level is the plane of Louis, but in full inspiration the level is the T6 vertebra.
5.7 What are the surface markings for auscultation of the aortic and pulmonary valves?
5.7 The aortic valve is auscultated at the 2nd intercostal space, right upper sternal border (Figure 1.5). The pulmonary valve is heard at the 2nd intercostal space, left upper sternal border. The mitral valve is heard at the 5th intercostal space, left midclavicular line. The tricuspid valve is heard at the 4th intercostal space, lower left sternal border. Note that these sites do not actually overlie the valves themselves.
5.8 What are the surface markings for the borders of the heart?
5.8 The left border of the heart is from the 2nd costal cartilage, left of the sternum, to the 5th left intercostal space, midclavicular line (Figure 1.5). The right border of the heart is from the 3rd right costal cartilage, right of the sternum, to the 6th right costal cartilage, right of the sternum.
zoom view
Fig. 1.5: Borders and auscultation areas of the heart. A, aortic valve; P, pulmonary valve; T, tricuspid valve; M, mitral valve.
49
Table 1.1   Surface anatomy landmarks of the thorax
Landmark
Location and significance
Midaxillary line
Vertical line intersecting a point midway between the anterior and posterior axillary folds
Midclavicular line
Vertical line passing through the midshaft of the clavicle
Nipple
Superficial to the 4th intercostal space in the male and prepuberal female Usually within the T4 dermatome
Sternal angle
Junction of the manubrium and body of the sternum Attachment of the 2nd costal cartilage rib to the sternum, the T4/T5 intervertebral disc, the inferior boundary of the superior mediastinum
Suprasternal notch
Curved superior border of the manubrium
Station 6
6.1 Identify the structures labelled A to D.
6.1 A Right atrium
B Arch of aorta
C Left lung hilum
D Left ventricle
6.2 Which fissures do lines 1–3 indicate?
6.2 1 Right horizontal or transverse
2 Right oblique
3 Left oblique
6.3 At what vertebral level do the bronchi enter the lungs?
6.3 The right main bronchus is about 2.5 cm long and enters the hilum of the lung at T5. It gives off an upper lobe branch before reaching the hilum. The left main bronchus is about 5 cm, and passes downwards and laterally below the arch of the aorta, anterior to the oesophagus and descending aorta. It enters the hilum of the lung at T6.
6.4 Name the lobes of the right and left lungs.
6.4 The right lung is divided by the oblique and horizontal fissures in to upper, middle, and lower lobes. The left lung is divided by an oblique fissure into upper and lower lobes. The lingula (Latin: ‘little tongue’) of the left upper lobe is composed of two bronchopulmonary segments that are analogous to the right middle lobe.
6.5 Define the term ‘bronchopulmonary segment’.
6.5 A bronchopulmonary segment is a discrete anatomical and functional unit of the lung that can be removed without disturbing the function of the other segments. They are pyramid shaped with their apices at the hilum. Each is served by its own tertiary bronchi, vein, artery, and lymph and has its own autonomic nerve supply.
6.6 How many bronchopulmonary segments are there in each lung?
6.6 Each lung has 10 bronchopulmonary segments. It would be unlikely that you would need to recite them in the exam, but they are listed here for reference purposes (Table 1.2):
50
Table 1.2   Bronchopulmonary segments of the lung
Right
Left
Upper lobe
Upper lobe
1. Apical
1. Apical
2. Posterior
2. Posterior
3. Anterior
3. Anterior
Middle lobe
Lingula
4. Lateral
4. Superior
5. Medial
5. Inferior
Lower lobe
Lower lobe
6. Superior (apical)
6. Superior (apical)
7. Medial basal (cardiac)
7. Medial basal
8. Anterior basal
8. Anterior basal
9. Lateral basal
9. Lateral basal
10. Posterior basal
10. Posterior basal
6.7 What is the blood supply to the lungs?
6.7 The bronchial arteries supply oxygenated blood to the bronchial and connective tissue of the lungs. The left superior and inferior bronchial arteries arise from the thoracic aorta, whereas the single right bronchial artery has a variable origin (either the aorta, the left superior bronchial artery, or the right intercostal arteries). The bronchial veins drain into the azygous and hemiazygos veins. The pulmonary arteries supply deoxygenated blood to the alveoli via their terminal branches, and the superior and inferior pulmonary veins drain oxygenated blood to the left atrium.
6.8 What is the lymphatic drainage of the lungs?
6.8 The superficial lymphatic plexus lies under the visceral pleura and drains the surface of the lungs towards the hilum, whereas the deep plexus drains along the blood vessels towards the hilum. Lymph passes from the bronchopulmonary lymph nodes at the hilum to the tracheobronchial nodes at the bifurcation of the trachea, and thence to bronchomediastinal lymph trunks.
6.9 Describe the nerve supply to the lungs? What effect does stimulation of the sympathetic and parasympathetic system have on the lungs?
6.9 The pulmonary plexus at the hilum of the lung receives afferent autonomic nerve fibres from the mucous membranes of the bronchioles and alveoli stretch receptors, and serves efferent fibres to the bronchial musculature. Sympathetic fibres cause bronchodilatation and vasoconstriction, whereas parasympathetic fibres cause bronchoconstriction, vasodilatation, and glandular secretion.
6.10 In which bronchus (right or left) are inadvertently aspirated foreign bodies most likely to lodge and why?
6.10 Foreign objects are aspirated more commonly in the right bronchus because it is wider and has a steeper angle than the left.
Station 7
7.1 Identify the structures labelled A to G.
7.1 A A right intercostal artery and vein
51B Right sympathetic chain
C Right phrenic nerve
D Superior vena cava
E Right principal bronchus
F Right pulmonary vein
G Pericardial sac (over right atrium)
7.2 What are the contents of the pulmonary hilum?
7.2 The pulmonary hilum contains: the pulmonary artery, the pulmonary vein, the main bronchus, the bronchial arteries and veins, lymph nodes, and autonomic nerves.
7.3 How many pulmonary veins drain each lung?
7.3 Two veins drain each lung (so there are four in total).
7.4 How many embryonic pharyngeal arches are there and which numbered arch disappears without giving rise to any specific structures?
7.4 There are six but the fifth exists only transiently, and no human structures are derived from it.
7.5 From which embryonic pharyngeal arch are the internal carotid arteries derived?
7.5 The 3rd arch.
7.6 From which embryonic pharyngeal arch is the right subclavian artery derived?
7.6 The 4th arch.
Station 8
8.1 Identify the structures labelled A to G.
8.1 A Ascending aorta
B Auricle of right atrium
C Pectinate muscles on right ventricular wall
D Chordae tendineae
E Pulmonary trunk
F Auricle of left atrium
G Anterior interventricular branch of left coronary artery
8.2 Identify the structure labelled H? What is its function?
8.2 H Papillary muscles. These attach to the cusps of the atrioventricular valves (in this case the tricuspid valve) to prevent prolapse.
8.3 Name the layers of pericardium.
8.3 The pericardium is divided in to fibrous and serous layers, the latter of which is subdivided into parietal and visceral layers. The fibrous pericardium is a tough layer fused with the central tendon of the diaphragm and the outer coats of the great vessels. It is attached to the sternum via the sternopericardial ligaments. The parietal pericardium lines the inner surface of the fibrous pericardium and is reflected around the great vessels to become continuous with the visceral pericardium that lines the heart.
8.4 Where is the pericardial space and what does it normally contain? What layers, from skin downwards, does a needle pass through to perform pericardiocentesis?
8.4 The pericardial space exists between the parietal and visceral layers, and is filled with about 50 mL of pericardial fluid.
In the subcostal approach the needle is positioned left of the xiphoid process with the needle angulated upwards at 45° to the skin. The needle passes through skin, 52superficial/deep fascia, the anterior layer of the rectus sheath, rectus abdominis, the posterior layer of the rectus sheath, diaphragm, endothoracic fascia, fibrous pericardium, and the parietal layer of serous pericardium.
In the parasternal approach the needle is placed at the 5th intercostal space near the left sternal margin. The needle passes through skin, superficial/deep fascia, pectoralis major muscle, intercostal muscles, transversus thoracis muscle, endothoracic fascia, fibrous pericardium, and the parietal layer of serous pericardium.
8.5 What structures are at risk of damage during pericardiocentesis?
8.5 In the subcostal approach, the main risk is of puncturing the liver if the needle is angulated too inferiorly. In the parasternal approach, the main risk is of puncturing the lungs. In both approaches, there is the risk of damage to the coronary arteries and atrial/ventricular walls.
8.6 What is the function and composition of the foramen ovale? What percentage of adults has an anatomically patent foramen ovale?
8.6 In the fetus, the foramen ovale allows oxygenated blood from the umbilical vein (via the inferior vena cava) to flow from the right to left atrium. It is composed of the septum primum and septum secundum. These are forced together at birth due to pressure changes resulting from expansion of the lungs, and usually fuse at about 3 months. In 10% of people fusion is incomplete.
8.7 What structures do the ductus arteriosus connect in the fetus?
8.7 The ductus arteriosus is a vascular shunt in the fetus connecting the pulmonary artery to the descending thoracic aorta, allowing blood from the right ventricle to bypass the lungs (which are non-functioning at this stage).
8.8 Where is the commonest site for coarctation of the aorta?
8.8 Aortic coarctation is associated with a patent ductus arteriosus, and occurs in the area where the ductus arteriosus inserts. Narrowing can be preductal, ductal, or postductal. It usually occurs distal to the origin of the left subclavian artery.
8.9 What do the bulbus cordis and truncus arteriosus give rise to in the adult?
8.9 The bulbus cordis and the primitive ventricle give rise to the ventricles of the heart. The cranial end of bulbus cordis and the truncus arteriosus give rise to the aorta and pulmonary trunk.
Station 9
9.1 Identify the structures labelled A to E.
9.1 A Right oblique fissure of the right lung
B Left ventricle
C Right latissimus dorsi muscle
D Descending thoracic aorta
E Left trapezius
9.2 List the successive elements of the airway, starting with the trachea and leading on to the alveolus.
9.2 The trachea splits into the left and right principle bronchi and thence to lobar (secondary bronchi), and segmentary (tertiary) bronchi. After entering the bronchopulmonary segment the bronchi undergoes successive branching into smaller and smaller tubes until they give rise to bronchioles. These are less than 1 mm in diameter and contain no cartilage. Bronchioles divide into terminal bronchioles, which give rise to respiratory bronchioles in their walls. These structures end in alveolar ducts that lead to alveolar sacs, across which gas exchange takes place with the surrounding capillaries.
9.3 State the origin and termination of the azygos vein.
539.3 The azygos vein forms about the level of the right renal vein from either a posterior tributary of the inferior vena cava or from the junction of the right ascending lumbar and right subcostal veins. It traverses the aortic opening of the diaphragm and lies to the right of the vertebra, behind the oesophagus. The vein ends by running anteriorly over the hilum of the right lung to enter the superior vena cava at T4. The azygos vein has tributaries of: the lower eight right posterior intercostal veins, the right superior intercostal vein, bronchial and oesophageal veins, and the accessory azygos/hemiazygos veins.
9.4 What tributaries do the hemiazygos and accessory veins receive, and where do they drain to?
9.4 The hemiazygos vein drains the four lower left posterior intercostal veins. It arises from the left ascending lumbar, the left subcostal, and often the left renal veins.
The accessory hemiazygos vein drains the 5–8th left posterior intercostal veins, and has tributaries from the bronchial and mid-oesophageal veins.
9.5 Describe the arrangement of muscles in the intercostal spaces.
9.5 There are three layers of muscle in the intercostal space (Figure 1.6). The external intercostal muscle forms the outermost layer; its fibres are directed forwards and downwards from the inferior border of the rib above to the superior border of the rib below. The internal intercostal muscle is the intermediate layer; its fibres are directed downwards and backwards from the subcostal groove of the rib above to the upper border of the rib below. The deepest layer is the innermost intercostal muscle (really composed of a group of three muscles). These muscles cross more than one intercostal space.
9.6 Which muscles contract to produce inspiration?
9.6 The external intercostal muscles aid in forced and passive inspiration. The internal intercostal muscles aid in forced expiration. Passive expiration is achieved by relaxation of the muscles and the elastic recoil of the lungs.
9.7 Name the accessory muscles of respiration.
9.7 The accessory muscles of respiration help to increase the thoracic capacity in deep inspiration. They are the sternocleidomastoid, scalenus anterior and medius, serratus anterior and pectoralis major and minor.
zoom view
Fig. 1.6: The layers of the chest wall.
Station 10
10.1 What type of radiological investigation is this?
5410.1 Barium swallow
10.2 Name the parts of the gastrointestinal tract indicated by A, B and C.
10.2 A Lower oesophagus
B Stomach
C Also stomach
10.3 Comment on the narrowing at point D.
10.3 D This narrowing represents the site of the lower oesophageal sphincter.
10.4 Oesophogastroduodenoscopy (OGD) is performed. State in centimetres the distance from the incisor teeth:
10.4a at which the oesophagus commences.
10.4a 17 cm
10.4b at which the oesophagus is crossed by the left bronchus.
10.4b 28 cm
10.4c at which the oesophagus terminates.
10.4c 43 cm
10.5 Name the layers an endoscope must pass through to perforate the oesophagus.
10.5 To perforate the oesophagus one must pass through first mucosa, submucosa, a muscular layer (the composition of which depends on level), and an outer connective tissue layer (areolar tissue). The thoracic oesophagus has no serosa.
10.6 What type of muscle fibre lies in the oesophageal wall, and what cell type lines the oesophageal mucosa?
10.6 The oesophagus has outer longitudinal and internal circular muscular layers. The muscle fibres of the upper two-thirds of the oesophagus are striated (and hence under voluntary control), and the lower one-third is smooth. In health, it is lined by squamous epithelium.
10.7 What is the blood supply of the oesophagus?
10.7 The blood supply to the oesophagus, like most long tubes, is segmental. The upper third is supplied by the inferior thyroid artery and vein, the middle third by descending aortic branches and veins to the azygos, and the lower third by the left gastric artery and vein (portal system). Note that there is anastomosis between the portal and systemic systems; in portal hypertension these veins distend in to oesophageal varices that can cause life-threatening haemorrhage.
10.8 What is meant by the term ‘Barrett's oesophagus’?
10.8 Barrett's oesophagus is metaplasia of the squamous epithelium of the lower oesophagus into columnar epithelium. It is thought to be an adaption to chronic acid exposure from gastro-oesophageal reflux. There is a strong association with adenocarcinoma of the oesophagus.
10.9 What is the lymphatic drainage of the oesophagus?
10.9 Lymph drainage follows arterial supply. The upper third of the oesophagus drains into the deep cervical nodes, the middle third into the superior and posterior mediastinal nodes, and the lower third in to coeliac nodes.
10.10 What is ‘Virchow's node’ and why does it commonly present on the left side?
10.10 Virchow's node is an enlarged lymph node in the left supraclavicular fossa (the associated sign is called Troisier's sign). It is associated with gastric and other intra-abdominal cancer. The lymph node is on the left side because the majority of lymph drains via the thoracic duct in to the left subclavian vein. Metastases block the thoracic duct causing reflux in to the surrounding nodes.
10.11 Where is the cisterna chyli and what is its function?
10.11 The cisterna chyli is a dilated sac at the lower end of the thoracic duct that is the common pathway of drainage of lymph and chyle from the abdomen and lower limbs. It is usually positioned between the abdominal aorta and right crus of the diaphragm. Its position and existence are, however, inconsistent.
10.12 What is the embryological origin of the trachea?
5510.12 The trachea develops from the floor of the foregut. Initially the laryngotracheal groove appears, which later becomes a tube. Buds appear on either side of the tube and develop into the lungs. The shared origin of the trachea and oesophagus explains the association of tracheoesophageal fistula with oesophageal atresia.
Station 11
11.1 With your knowledge of anatomy, give a differential diagnosis of a mass located in the region indicated by circle B.
11.1 Skin/soft tissue: sebaceous cyst, lipoma
Muscle: sarcoma, psoas abscess
Bowel: appendix abscess/mass, Crohn's, carcinoma, tuberculosis
Gynaecological: ovarian tumour, fibroids
Urological: pelvic kidney, bladder diverticulum
Vascular: aneurysm of the external or common iliac artery, enlarged iliac lymph node
11.2 Define McBurney's point.
11.2 McBurney's point (Figure 1.7) is the typical location of the appendix, and is located at a point two thirds from the umbilicus to the anterior superior iliac spine.
11.3 In which dermatome is the umbilicus located?
11.3 T10 (see Figure 1.4).
zoom view
Fig. 1.7: Abdominal incisions.
11.4 Name the sequence of layers you would pass through whilst incising through lines A and C to enter the abdomen?
5611.4 A Skin, subcutaneous fat, Scarpa's fascia, external oblique muscle, internal oblique muscle, transversalis fascia, extraperitoneal fat, parietal peritoneum.
C Skin, subcutaneous fat, Scarpa's fascia, linea alba, transversalis fascia, extraperitoneal fat, parietal peritoneum.
11.5 What is the surface marking of the transpyloric plane and what structures are present at this level?
11.5 The transpyloric plane (Figure 1.8) is located halfway between the suprasternal notch and the pubic symphysis, at the level of the L1 vertebral body. At this level lie the following structures: the pylorus of stomach, fundus of gallbladder, pancreatic neck, duodenojejunal flexure (and first part of duodenum), spinal cord termination, line of attachment of transverse mesocolon, left renal hilum, origin of the superior mesenteric artery, origin of portal vein.
11.6 At what vertebral level is the subcostal plane and what structures are present at this level?
11.6 The subcostal plane (Figure 1.8) is the line parallel to the lowest part of the thoracic cage. The inferior mesenteric artery and L3 vertebra are present at this level.
11.7 At what vertebral level does the aorta bifurcate? What is the surface marking for this point?
11.7 The aorta bifurcates at the L4 vertebral level, which is usually about the level of the umbilicus. This is also the level of the supracristal plane (Figure 1.8), which is a line joining the most superior parts of the iliac crests.
11.8 What are the surface markings for:
11.8a the inferior border of the liver?
11.8a The inferior border of the liver extends from the tip of the right 10th rib to the left 5th intercostal space medial to the midclavicular line. The superior border is at the level of the 5th intercostal space.
zoom view
Fig. 1.8: Lines and planes of the abdomen.
5711.8b the spleen?
11.8b The spleen lies under the 9–11th ribs on the left side. The long axis of the spleen lies along the 10th rib, the posterior pole being just to the left of the vertebral column, and the anterior pole is in the midaxillary line.
11.8c the fundus of the gallbladder?
11.8c The fundus of the gallbladder is at the point at which the rectus abdominis intersects the costal margin, at the tip of the 9th costal cartilage.
11.9 Describe the location of these abdominal incisions:
11.9 See Figure 1.7 and Table 1.3.
Table 1.3   Abdominal surface anatomy landmarks
Landmark
Location
Significance
Arcuate line (Douglas’ line)
Approximately one-third of the distance from the umbilicus to the pubic symphysis
Lower limit of the posterior sheath
Deep inguinal ring
Midway between anterior superior iliac spine and pubic tubercle
Opening in the transversalis fascia for the vas deferens and gonadal vessels (or round ligament in the female)
Linea alba
Midline aponeurotic band extending from xiphoid process to the pubic symphysis
Formed by the combined abdominal muscle aponeuroses.
McBurney's point
Two-thirds from the umbilicus to the anterior superior iliac spine
Typical location of the appendix
Mid-inguinal point
Midway between the anterior superior iliac spine and the pubic symphysis
Location of the femoral artery
Semilunar line
The lateral edge of the rectus abdominis muscle
Formed by the combined aponeuroses of the abdominal wall muscles at the lateral margin of the rectus sheath.
Subcostal plane
Line parallel to the lowest part of the thoracic cage
Origin of inferior mesenteric artery
Supracristal plane
Horizontal plane at the upper margin of the iliac crests
L4 Bifurcation of the aorta
Transpyloric line
Half the distance between the jugular notch and the pubic crest
  • Pylorus of stomach
  • Fundus of gallbladder
  • Pancreatic neck
  • Duodenojejunal flexure (and first part of duodenum)
  • Spinal cord termination
  • Line of attachment of transverse mesocolon
  • Left renal hilum
  • Origin of the superior mesenteric artery
  • Origin of portal vein
Umbilicus
Within T10, approximately at level of L4 vertebra
11.9a Gridiron
5811.9a Gridiron: A 2.5–5 cm oblique incision at McBurney's point, perpendicular to a line running from the anterior superior iliac spine to the umbilicus. Used for appendicectomy.
11.9b Kocher
11.9b Kocher: an oblique incision below and parallel to the right costal margin (starting below the xiphoid process). Used for access to biliary structures.
11.9c Mercedes Benz
11.9c Mercedes Benz: bilateral low Kocher's incisions with an upper midline limb up and through the xiphisternum. For access to upper abdominal viscera. Also known as a roof-top incision.
11.9d Pfannenstiel
11.9d Pfannenstiel: A transverse lower abdominal incision centered above the pubic symphysis, slightly upturned at the ends (a ‘smile’ incision).
Station 12
12.1 Identify the structures labelled A to G.
12.1 A Inferior vena cava
B Erector spinae muscle
C Portal vein
D Right crus of diaphragm
E Superior mesenteric artery
F Fundus of the stomach
G Aorta (abdominal)
12.2 At what vertebral level is the above CT slice taken?
12.2 At the level of the superior mesenteric artery, below the level of the spleen.
12.3 What is C and what are its tributaries?
12.3 The portal vein (C) drains the gastrointestinal tract and associated viscera. It is formed from the splenic and superior mesenteric veins as they unite behind the neck of the pancreas. The splenic vein receives the short gastric, left gastroepiploic, inferior mesenteric, and pancreatic veins. The superior mesenteric vein receives the jejunal, ilial, ileocolic, right colic, middle colic, inferior pancreaticoduodenal and right gastroepiploic veins. There are three other direct tributaries of the portal vein: the left gastric, right gastric, and cystic veins.
12.4 What proportion of blood does C supply to the liver?
The image on the next page (b) is another axial CT scan of the same patient:
12.4 The portal vein supplies about 70% of the blood to the liver. The remaining 30% is oxygenated blood from the hepatic arteries.
12.5 Identify the structures labelled A to E.
12.5 A Gallbladder
B Biliary tract
C Superior mesenteric artery
D Left lobe of the liver
E Tail of the pancreas
12.6 What are the functions of organ E?
12.6 The pancreas (E) has exocrine and endocrine functions. The pancreatic islets (Islets of Langerhans) produce insulin and glucagon. The pancreas also secretes enzymes capable of hydrolysing proteins, fats, and carbohydrates.
12.7 Name the different parts of organ E.
5912.7 The pancreas is divided in to a head, uncinate process, neck, body, and tail. The head lies within the concavity of the duodenum, with its uncinate process extending to the left behind the superior mesenteric vessels. The neck is positioned anterior to the portal vein and superior mesenteric artery origins. The body runs upwards and to the left and the tail abuts the hilum of the spleen.
12.8 Define the term ‘pseudocyst’? Describe its pathogenesis.
zoom view
12.8 The lesser sac separates the stomach from the pancreas. If fluid leaks from the pancreas during acute pancreatitis this can become trapped within the lesser sac forming a pseudocyst.
Station 13
13.1 Identify the structure labelled A? What gives rise to its colour?
13.1 A Linea alba (‘white line’). The whiteness indicates that it is a relatively avascular structure and hence ideal for incision without bleeding.
13.2 Identify the structures labelled B to G.
13.2 B Tendinous insertion
C External oblique muscle
D Rectus abdominis
E Superficial epigastric vessels
F Umbilicus
G Posterior layer of rectus sheath
13.3 Name in sequence the layer of the abdominal wall traversed by the umbilical port.
13.3 Skin, subcutaneous fat, Scarpa's fascia, umbilical cicatrix pillar, extraperitoneal fat, parietal peritoneum.
13.4 In which direction do the fibres of the internal oblique, external oblique, and transversus abdominis fibres run at the level of the umbilicus?
13.4 External oblique fibres run inferiorly and anteriorly. Internal oblique fibres run perpendicular to the external oblique muscle, directed superiorly and anteriorly. Transversus abdominis fibres run transversely.
13.5 What are the contents of the rectus sheath?
13.5 The rectus sheath contains the large rectus abdominis muscle (extending from the pubic symphysis to the xiphisternum/lower costal cartilages), the pyramidalis muscle, the superior and inferior epigastric vessels, ventral primary rami of T7–T12, and lymphatics.
13.6 What is the surface marking of the arcuate line and what is its significance?
13.6 The arcuate line (Douglas’ line) demarcates the lower limit of the posterior sheath. It is located about one-third of the distance from the umbilicus to the pubic crest. Above the level of this line, the internal oblique aponeurosis splits to envelope the rectus abdominis muscle, and the transversus abdominis aponeurosis runs under the rectus abdominis. Below the arcuate line, the internal oblique and transversus abdominis aponeuroses merge and pass superficial to the rectus muscle. Hence, below the arcuate line the only layers deep to the rectus abdominis are the transversalis fascia, extraperitoneal fat, and parietal peritoneum.
13.7 What is the distal limit of Scarpa's fascia?
13.7 Scarpa's fascia extends in to the thigh and fuses with the fascia lata at the flexure of the skin crease of the hip joint (about 1 cm below the inguinal ligament).
13.8 What is the continuation of Scarpa's fascia in the perineum called?
13.8 It fuses with Colles’ fascia in the perineum.
6013.9 Where is the median umbilical ligament located and what does it contain?
13.9 The median umbilical ligament extends from the bladder to the umbilicus, on the deep surface of the anterior abdominal wall. It can be seen easily during laparoscopy by pointing the laparoscope towards the anterior abdominal wall in the median plane. It contains the urachus, which is the remnant of the allantois, a canal that drains the urinary bladder of the fetus that joins and runs through the umbilical cord. If the allantois fails to close then urine continues to leak through the umbilicus after birth.
13.10 Where are the medial umbilical ligaments located and what do they contain?
13.10 The medial umbilical ligaments are lateral to the median umbilical ligament on the deep surface of the anterior abdominal wall. They contain the remnant of the fetal umbilical arteries.
Station 14
14.1 Identify the structures labelled A, B, D and E.
14.1 A Caecum
B Appendix
D Ileum
E Mesentery of small bowel
14.2 Identify the structure labelled C. What structures run in this tissue?
14.2 C Mesoappendix. The most important structures are the appendicular artery and vein, which may bleed if not ligated carefully. There are also autonomic nerves, lymphatic vessels, and sometimes a lymph node.
14.3 Explain on the basis of anatomical principles why infection of the appendix may result in necrosis whilst infection of the gallbladder usually does not.
14.3 A small non-anastomosing single artery, the appendicular artery, supplies the appendix (Figure 1.9). When the appendix becomes inflamed, oedema of the wall compresses the artery causing thrombosis. This leads to necrosis and perforation of the blind ending tip of the appendix. In contrast, in addition to the cystic artery, the gallbladder has collateral supply from the liver bed, ensuring that adequate blood supply is preserved.
14.4 Name four common positions assumed by the appendix.
14.4 Common positions include: retrocolic/retrocaecal, pelvic/subcaecal, retroileal/preileal. The order of frequency is disputed but the commonest two are probably pelvic and retrocaecal.
zoom view
Fig. 1.9: Appendix and arterial supply.
61This variability of the appendix position can make diagnosis sometimes difficult and removal technically difficult.
14.5 Using embryological principles explain the changing nature and location of pain in appendicitis.
14.5 Visceral pain from the appendix is triggered by distension of the lumen or muscle spasm. Afferent pain fibres travel to the T10 spinal level, and a midline periumbilical pain is felt. As the appendix becomes more inflamed it can cause localised inflammation of the peritoneum, and pain is referred to the right iliac fossa.
14.6 Which nerves may be damaged when performing an open appendicectomy?
14.6 The ilioinguinal and iliohypogastric nerves. To avoid these nerves incision should not be closer than 3 cm from the anterior superior iliac spine.
14.7 What are taeniae coli and where do they converge?
14.7 The teniae coli are three bands of smooth muscle running longitudinally along the caecum, ascending, transverse and descending and sigmoid colon (Figure 1.9). They contract to form haustra, which are sacculations of the large bowel that can be seen on radiographs. The teniae coli converge at the vermiform appendix and the rectum.
14.8 What are appendices epiploicae?
14.8 Appendices epiploicae are small fat-filled peritoneal pouches along the teniae coli. They can sometimes become inflamed (epiploic appendagitis) mimicking appendicitis and other intra-abdominal conditions.
Station 15
15.1 Identify the structures labelled A to E.
15.1 A Right external iliac artery
B Right external iliac vein
C Urinary bladder
D Uterus
E Rectum
15.2 At what vertebral level does the structure labelled A divide?
15.2 L5/S1.
15.3 What bony landmark in the pelvis defines this division?
15.3 The sacroiliac joints.
15.4 Does the ureter pass anterior or posterior to this division?
Image (b) on the next page is an axial CT scan through the pelvis of the same patient:
15.4 The ureter passes anteriorly over the bifurcation of the iliac arteries.
15.5 Identify the structures labelled A to D
15.5 A Sartorius
B Superior pubic ramus
C Obturator internus
D Ischium
15.6 Name the parts of the levator ani.
15.6 The levator ani originates from the body of the pubis, the ischial spine, and the fascia of obturator internus. It inserts in to the perineal body, the anococcygeal body, and the walls of the pelvic organs below the bladder (the prostate, vagina, rectum and anal canal). As well as the functions listed in Table 1.4 it also increases intra-abdominal pressure during defecation, micturition, and parturition. The parts are outlined in Table 1.4.
15.7 What are the boundaries of the pelvic outlet?
15.7 The pelvic outlet is bounded posteriorly by the coccyx, laterally by the ischial tuberosities, and anteriorly by symphysis pubis (see Figure 1.11).
62
Table 1.4   The parts and functions of levator ani
Part
Name
Functional anatomy
Anterior
  • Levator prostatae
  • Sphincter vaginae
Forms a sling around the prostate/vagina and inserts in to the perineal body, helping to stabilise it
Intermediate
  • Puborectalis
  • Pubococcygeus
Forms a sling around the junction of the rectum/anal canal
Posterior
Iliococcygeus
Inserted into the anococcygeal body and the coccyx
15.8 What attaches to the perineal body?
zoom view
15.8 The perineal body is a pyramidal fibromuscular mass of tissue at the junction of the urogenital triangle and the anal triangle. It has attachments to: the external anal sphincter, bulbospongiosus muscle, superficial and deep transverse perineal muscles, anterior fibres of levator ani, and the external urinary sphincter.
Station 16
16.1 How does the small bowel differ in appearance from the large bowel on an abdominal plain film?
16.1 The large bowel is peripheral and less coiled. It has haustra (which on an abdominal radiograph do not traverse the entire diameter of the colon). The lumen of large bowel is greater than the small bowel.
16.2 What type of radiological study is shown? Comment on the diameter of the bowel.
16.2 Double contrast barium enema. The bowel has been inflated by air pumped through the rectum.
16.3 What name is given to the lines labelled B?
16.3 B Haustra
16.4 Name the segments of the colon labelled A and C. What is the blood supply to these structures?
16.4 A Caecum/ascending colon. This is supplied by the colic branch of the ileocolic artery, and the right colic artery (both branches of the superior mesenteric artery).
C Descending colon. This is supplied by the left colic artery (a branch of the inferior mesenteric artery).
The blood supply to the rest of the large bowel is illustrated in Figures 1.13 and 1.18 (see pp. 69 and 77). The proximal two-thirds of the transverse colon is perfused by the middle colic artery (superior mesenteric), and the latter one-third by the inferior mesenteric. The sigmoid arteries supply the sigmoid colon. The rectum is supplied by the superior rectal artery (inferior mesenteric), middle rectal artery (internal iliac), and inferior rectal artery (internal iliac).
16.5 What is the marginal artery of Drummond?
16.5 There is a continuous vascular arcade throughout the length of the gastrointestinal tract, due to anastomosis of branches of the superior and inferior mesenteric arteries along the marginal artery of Drummond.
16.6 What are the topographical relations of structure D?
6316.6 D Rectum. The rectovesical fascia of Denonvilliers separates the rectum from anterior structures and is dissected in rectal dissection for carcinoma. Anteriorly in the upper two-thirds are coils of small intestine that lie in the space between the rectum and bladder in men, or rectum and uterus in women (the pouch of Douglas). In the lower two-thirds anteriorly are the prostate, bladder, vas deferens, and seminal vesicles in males, and vagina in the female. Posteriorly are the sacrum, coccyx, median sacral and rectal vessels, sympathetic trunk, pelvic splanchnic nerves, and piriformis. Laterally lies levator ani, coccygeus and obturator internus muscles, fat, lymph nodes, ischioanal fossa, and the lateral ligaments of the rectum.
16.7 How does the peritoneum relate to structure D?
16.7 D The upper third of the rectum has peritoneum on its anterior and lateral surfaces, the middle third has peritoneum on its anterior surface only, and the lower third is beneath the rectal floor and has no peritoneal attachments.
16.8 What parts of the large bowel are particularly susceptible to injury in blunt abdominal trauma?
16.8 Injuries occur at the junctions of where mobile parts of the colon (the transverse and sigmoid) join the fixed parts (ascending and descending).
16.9 What muscles make up the posterior abdominal wall?
16.9 From medial to lateral are the psoas major, quadratus lumborum (above the iliac crest), or iliacus (below the iliac crest), transversus abdominis and internal oblique (Figure 1.10). The posterior part of the diaphragm also contributes to the upper posterior wall of the abdomen.
zoom view
Fig. 1.10: Posterior abdominal wall.
Station 17
17.1 Identify the structures labelled A to D.
17.1 A Spermatic cord
B Long saphenous vein
C Common femoral artery
D Common femoral vein
17.2 What are the contents of A?
17.2 The most well-known mnemonic for the contents of the spermatic cord (A) is the ‘rule of threes’ (Table 1.5).
64
Table 1.5   The ‘rule of threes’ for contents of the spermatic cord
Layers of fascia
  • External spermatic
  • Cremasteric
  • Internal spermatic
Arteries
  • Testicular
  • Cremasteric
  • Artery of the vas
Veins
  • Pampiniform plexus
  • Cremasteric
  • Vein of the vas
Nerves
  • Nerve to the cremaster
  • Sympathetic fibres (T10–T11)
  • Ilioinguinal nerve (this is actually on, not in, the cord)
Other structures
  • Vas deferens
  • Lymphatics
  • Processes vaginalis (pathologically, in patients with an indirect inguinal hernia)
17.3 What is the inguinal ligament? What are the medial and lateral attachments of the inguinal ligament?
17.3 The inguinal ligament is formed from the rolled over aponeurosis of the external oblique. It runs from the pubic tubercle to the anterior superior iliac spine.
17.4 What is the surface marking for the deep inguinal ring? Intraoperatively, what anatomical structure may be found that helps define this point?
17.4 The deep inguinal ring is an opening in the fascia transversalis at the mid-point of the inguinal ligament. Medially run the inferior epigastric vessels.
17.5 How may an inguinal hernia be clinically distinguished from a femoral hernia?
17.5 The opening of an inguinal hernia is above and medial to the pubic tubercle whereas a femoral hernia is below and lateral.
17.6 What are the boundaries of the inguinal canal?
17.6 The boundaries of the inguinal canal:
  • anterior: external oblique aponeurosis, reinforced at its lateral one-third by the origin of the internal oblique
  • posterior: conjoint tendon medially (the fused insertion of the internal oblique and transversus abdominis), transversals fascia laterally
  • roof: arching fibres of the internal oblique and transversus abdominis
  • floor: the inguinal ligament, and the lacunar ligament medially.
17.7 What is the embryological origin of the cremasteric fascia?
17.7 As the processus vaginalis descends into the scrotum during development it brings with it layers of the abdominal wall. The external spermatic fascia is derived from the external oblique aponeurosis. The cremasteric fascia is derived from the internal oblique. The internal spermatic fascia is derived from the fascia transversalis.
17.8 Describe the significance and boundaries of Hesselbach's triangle.
17.8 Direct inguinal hernias pass through Hesselbach's triangle, a defect in the transversalis fascia, whereas indirect hernias must traverse the deep inguinal ring. The boundaries of Hesselbach's triangle are:
65
  • medially – the lateral border of rectus abdominis
  • superolaterally – the inferior epigastric vessels
  • inferolaterally – the inguinal ligament.
17.9 Why is one cautioned against taking deep bites when suturing the inferior border of a mesh to the inguinal ligament?
17.9 Because the femoral vessels run just under this ligament, and may bleed profusely! The femoral vein is more difficult to control than the femoral artery.
Station 18
18.1 Identify the structures labelled A to G
18.1 A Right superficial femoral artery
B Right vas deferens and spermatic cord
C Right obturator internus
D Alcock's canal, internal pudendal vessels, pudendal nerve
E Left sciatic nerve
F Membranous urethra
G Anus
18.2 Regarding the structure labelled G:
18.2a what communication exists between the portal and systemic venous circulation in this region?
18.2a There is communication between the superior rectal vein (portal system), and the inferior rectal veins (draining to the internal iliac vein via the internal pudendal veins, systemic circulation).
18.2b Name other regions in the body where similar portosystemic communications exist.
18.2b The other communications are shown in Table 1.6.
18.3 Describe the boundaries of the anal triangle of the perineum.
18.3 This is formed by the two ischial tuberosities and the coccyx (Figure 1.11). The anterior border is the posterior border of the perineal membrane, and the sacrotuberous ligaments form the two sides.
18.4 What are the topographical relations of the prostate?
18.4 Anterior: the pubic symphysis (separated by extraperitoneal fat), and the prostatic plexus of veins.
Posterior: the rectum separated by the fascia of Denonvilliers.
Superior: the bladder.
Inferior: the external sphincter of the bladder.
Table 1.6   Portal-systemic communications
Lower oesophagus
Between the oesophageal branch of the left gastric vein (portal system), and the oesophageal vein (azygos system)
Abdominal wall
Between portal branches in the liver and the veins passing to the abdominal wall (forming a caput medusae)
Bare area of liver
Between portal veins in the liver and veins of the diaphragm (across the bare area)
Retroperitoneum
Between the portal tributaries in the mesentery and retroperitoneal veins
66
zoom view
Fig. 1.11: Anal and urogenital triangles.
18.5 What are the contents and boundaries of the ischioanal fossae?
18.5 The ischioanal fossae (or ischiorectal fossae, an old term) are wedge shaped spaces on either side of the anal canal. Their boundaries are:
  • laterally: obturator internus muscle and fascia
  • medially: levator ani and pelvic fascia, external anal sphincter
  • anteriorly: the urogenital perineum
  • posteriorly: sacrotuberous ligament and gluteus maximus
  • inferiorly: skin and subcutaneous fat
  • superiorly: levator ani
The space contains fat (which is particularly prone to infection and abscess formation) and the inferior rectal nerve and vessels. The lateral walls contain Alcock's canal, which has in it the pudendal nerve and vessels.
18.6 Where is the commonest site for an anal fissure?
18.6 Anal fissures occur most commonly in the posterior midline. Fissures develop in the anal valves (the lower ends of the anal columns) as hard faecal matter catches during defecation. This area may be susceptible due to a lack of support from the superficial part of the external sphincter.
18.7 Anatomically speaking, what is a haemorrhoid?
18.7 A haemorrhoid is fold of mucosa and submucosa containing a varicosed tributary of the superior rectal vein and a terminal branch of the superior rectal artery.
18.8 What is Goodsall's rule?
18.8 Goodsall's rule states that the external opening of a fistula situated behind the transverse anal line will open in to the anal canal in the posterior midline, but a fistula that opens anterior to this line is associated with a direct tract.
18.9 What are the lengths of the normal adult anal canal and rectum?
6718.9 The anal canal is approximately 4 cm long and the rectum is about 13 cm long.
18.10 What are the topographical relations of the anal canal?
18.10 The topographical relations of the anal canal are:
  • posteriorly: the anococcygeal body
  • laterally: the ischiorectal fossae
  • anteriorly in men: the perineal body, the urogenital diaphragm, and the membranous part of the urethra
  • anteriorly in women: the lower part of the vagina.
Station 19
19.1 Identify the parts of the bowel labelled A, B and C.
19.1 A Hepatic flexure or ascending colon
B Transverse colon
C Small bowel
19.2 What muscle is present at D?
19.2D Left psoas major muscle
19.3 Identify the structures labelled E, F, and G.
19.3 E Spleen
F Linea alba
G Rectus abdominis muscle
19.4 What are the posterior topographical relations of the ascending colon?
19.4 Posterior to the ascending colon lies:
  • musculoskeletal: iliac crest, iliacus, quadratus lumborum, transversus abdominis, and the right psoas.
  • organs: lower pole of the right kidney.
  • nerves: iliohypogastric and ilioinguinal nerves.
19.5 What are the posterior topographical relations of the descending colon?
19.5 Posterior to the descending colon lies:
  • musculoskeletal: iliac crest, iliacus, quadratus lumborum, transversus abdominis, and the left psoas.
  • organs: lateral border of left kidney
  • nerves: iliohypogastric, ilioinguinal, femoral nerves and the lateral cutaneous nerve of the thigh.
19.6 Which circumferential positions in the colonic wall are susceptible to acquired diverticular disease?
19.6 Diverticula are herniations of the mucosa through the circular muscle at points where the blood vessels pierce the muscle (natural points of weakness).
19.7 What does the transverse mesocolon attach to on the posterior abdominal wall?
19.7 The transverse mesocolon attaches the transverse colon to the posterior wall of the abdomen and the pancreas. It is continuous with the two posterior layers of the greater omentum (Figure 1.15).
19.8 What structures run within the transverse mesocolon?
19.8 The transverse mesocolon contains the transverse colon (in its free edge), the middle colic vessels and their branches, lymphatics, autonomic nerves, and extraperitoneal fatty tissue.
19.9 Of what clinical significance is the ileocaecal valve in a patient with large bowel obstruction?
19.9 An incompetent ileocaecal valve allows decompression of the large intestine in patients with large bowel obstruction and thereby reduces the risk of perforation.
19.10 Which part of the bowel most commonly undergoes volvulus?
19.10 The sigmoid colon has a long mesentery and may rotate upon it, causing an obstructed, often massively distended, loop of bowel. Volvulus of the caecum and transverse colon may also occur less commonly.
68
Station 20
20.1 Identify the arteries labelled A to F.
20.1 See Figure 1.12.
A Proper hepatic artery
B Common hepatic artery
C Left gastric artery
D Gastroduodenal artery
E Coeliac trunk
F Splenic artery
20.2 At what vertebral level does the coeliac artery (coeliac axis) leave the aorta?
20.2 Upper part of L1 (not T12, although many textbooks claim this).
20.3 What organs does D supply?
20.3 The gastroduodenal artery (D) supplies the stomach, the duodenum, and, indirectly, the pancreatic head and neck (via the anterior and posterior superior pancreaticoduodenal arteries).
20.4 Within what peritoneal structure does A run, and what topographical relationship does it have to the bile duct and portal vein?
Image (b) on the next page is a digital subtraction angiogram of the superior mesenteric artery and its branches (demonstrating normal anatomy).
zoom view
20.4 The hepatic artery (A) runs in the free border of the lesser omentum, anterior to the portal vein and left of the bile duct.
20.5 Identify the arteries labelled A to F.
20.5 See Figure 1.13.
A Right colic artery
B Ileocolic artery
C Appendicular artery
D Superior mesenteric artery
zoom view
Fig. 1.12: The coeliac axis.
69
zoom view
Fig. 1.13: Branches of the superior mesenteric artery.
E Main stem of jejunal arteries
F Main stem of ileal arteries
20.6 At what vertebral level does the superior mesenteric artery leave the aorta?
20.6 L1
20.7 At what vertebral level does the inferior mesenteric artery leave the aorta?
20.7 L3
20.8 Over what structures does the superior mesenteric artery pass before entering the root of the small intestinal mesentery.
20.8 After leaving the aorta, the superior mesenteric artery passes over the left renal vein, beneath the splenic vein and neck of the pancreas. It then passes over the uncinate process of the pancreas and the junction of the third and fourth parts of the duodenum, before entering the mesentery of the small and large bowel to give off its terminal branches.
20.9 Which part of the large intestine is particularly susceptible to ischaemia?
20.9 The distal third of the transverse colon/splenic flexure is termed a ‘watershed’ area, as there is a change in blood supply from the superior mesenteric to the inferior mesenteric artery. Watershed areas are vulnerable to ischaemia as they do not have good collateral supply.
Station 21
21.1 Identify the structures labelled A to H.
21.1 A Right renal vein
B Inferior vena cava
C Testicular vein (right)
D Left suprarenal organ
E Superior mesenteric artery
F Left renal vein
70G Abdominal aorta
H Left ureter
21.2 What is the ligament of Treitz?
21.2 The ligament of Treitz connects the duodenojejunal junction to the diaphragm (this ‘ligament’ actually contains muscular fibres that on contraction widen the angle of the duodenojejunal flexure assisting movement of intestinal contents). The ligament is commonly cut to access the aorta.
21.3 What are the lateral branches of the aorta?
21.3 The branches of the aorta are outlined in Table 1.7.
Table 1.7   Branches of the abdominal aorta
Three anterior visceral branches
  • Coeliac
  • Superior mesenteric
  • Inferior mesenteric
Three lateral visceral branches
  • Suprarenal
  • Renal
  • Testicular or ovarian
Five lateral abdominal wall branches
  • Inferior phrenic
  • Four lumbar branches
Three terminal branches
  • Two common iliac branches
  • Median sacral
21.4 What is the arterial supply of the adrenal glands?
21.4 The adrenal glands are supplied by the superior adrenal artery (from the inferior phrenic), middle adrenal artery (abdominal aorta), and inferior adrenal artery (renal artery).
21.5 What is the venous drainage of the adrenal glands?
21.5 The adrenal vein (the right adrenal vein drains into the inferior vena cava, the left adrenal vein drains in to the left renal vein).
21.6 At operation the inferior mesenteric artery can often be ligated without serious consequence. What is the anatomical explanation for this?
21.6 Due to collateral supply via the marginal artery of Drummond. It is sometimes ligated in operations such as open aortic aneurysm repair.
21.7 Regarding structure B:
21.7a at what vertebral level does it commence?
21.7a Commences at L5 behind the common iliac arteries.
21.7b as it ascends, what are its successive anterior topographical relations?
21.7b It is initially related anteriorly to the small intestine, the third part of the duodenum, the head of the pancreas, and the first part of the duodenum. It passes behind the epiploic foramen (in front of which is the portal vein, common bile duct and hepatic artery); it then ascends in a groove in the liver before traversing the diaphragm.
21.7c name the tributaries.
21.7c This can be remembered by the mnemonic: I L ike To Rise So High: Iliac, Lumbar, Testicular, Renal, Suprarenal, Hepatic.
Station 22
22.1 Identify the structures labelled A to E.
22.1 A Gallbladder
71B Right hepatic duct
C Left hepatic duct
D Right renal pelvis
E Common bile duct
22.2 What route does bile take to enter the intestinal tract? In gallstone ileus, what route does a gallstone usually travel to enter the intestinal tract?
22.2 Bile is stored in the gallbladder. It passes in to the cystic duct, which joins the common hepatic duct to form the common bile duct (Figure 1.14). This travels in the free edge of the lesser omentum (with the hepatic artery and portal vein). The duct is joined by the main pancreatic duct (of Wirsung) at the ampulla of Vater, which enters the second part of the duodenum past the sphincter of Oddi.
Gallstone ileus is the condition of a gallstone causing mechanical intestinal obstruction (hence the condition is not really ileus at all). Instead of travelling through bile ducts, gallstones usually erode through the wall of the gallbladder over a period of time. They often get lodged in the distal ileum.
22.3 What is the narrowest part of the extrahepatic biliary system?
22.3 Its opening in to the second part of the duodenum.
22.4 What is the function and composition of bile?
22.4 Bile is produced by hepatocytes in the liver. It is composed mainly of water (85%), bile salts, mucous, pigments, fats, inorganic salts, and cholesterol. Bile acts as a surfactant, emulsifying fats. The increased surface area allows for more efficient action of enzymes such as pancreatic lipase. Its other functions include: being the route of excretion for the haemoglobin breakdown product bilirubin, and neutralising excess stomach acid before it enters the ileum.
22.5 Which hormone stimulates the release of bile? What is the trigger for this hormone and where is the hormone synthesised.
22.5 Cholecystokinin is a peptide hormone that stimulates the contraction of the gallbladder and the relaxation of the sphincter of Oddi.
zoom view
Fig. 1.14: The biliary system.
It is synthesised by the 72‘I-cells’ of the mucosal epithelium of the small intestine, and secreted in response to Chyme entering the duodenum. Its other functions include: increasing the production of bile in the liver; stimulation of the release of digestive enzymes in the pancreas; causing relaxation of the stomach musculature.
22.6 What epithelium lines the extrahepatic biliary ducts?
22.6 Extra-hepatic ducts are lined by tall columnar cells interspersed with mucous glands.
22.7 What is Pringle's manoeuvre?
22.7 Pringle's manoeuvre temporarily prevents blood from entering the liver by compressing the hepatic artery and portal vein. Intraoperatively it can be performed by placing a finger within the foramen of Winslow and another on its anterior wall and squeezing.
22.8 What is Mirizzi's syndrome?
22.8 Mirizzi's syndrome is a cause of obstructive jaundice caused by one or more gallstones becoming impacted in Hartmann's pouch. The biliary obstruction can be caused by either external compression of the common hepatic duct by the gallstone, or fistulisation of the gallstone in to the common hepatic duct.
Station 23
23.1 What anatomical lines are indicated by C and D?
23.1 See Figure 1.8.
C Linea semilunaris
D Arcuate line
23.2 What plane is indicated by dotted yellow line B? What anatomical structures can be found at this level?
23.2 B The supracristal plane (Table 1.3). This is a transverse plane through the uppermost part of the iliac crest, at the level of the L4 vertebra. It usually passes close to the umbilicus. The plane divides the lower and upper quadrants of the abdomen. At this level the abdominal aorta bifurcates.
23.3 Which vessels in the anterior abdominal wall, indicated by point A, may have been punctured by the bullet?
23.3 The inferior epigastric artery is a branch of the external iliac artery, and enters the rectus sheath anterior to the arcuate line by piercing the transversalis fascia. It runs in the sheath posterior to the rectus abdominis muscle and supplies the anterior abdominal wall. It ends by anastomosing with the superior epigastric branch of the internal thoracic artery. The inferior epigastric vein follows a similar course and drains in to the external iliac vein.
23.4 What are the origins of these vessels?
23.4 The external iliac artery and vein.
23.5 Define the linea semilunaris.
23.5 The linea semilunaris (Figure 1.8 and Table 1.3) is a tendinous line lateral to the rectus abdominis, extending from the cartilage of the ninth rib to the pubic tubercle. It demarcates the lateral fusion of the anterior and posterior rectus sheath layers.
23.6 What is the innervation and action of the internal oblique?
23.6 The internal oblique is supplied by the lower six thoracic nerves, the iliohypogastric nerve, and the ilioinguinal nerve (also true for the external oblique and the transversus abdominis). It assists in flexion and rotation of the trunk.
23.7 List the functions of the muscles of the abdominal wall.
23.7 The muscles of the anterior and lateral walls have a number of functions. They assist during forced expiration by pulling down the ribs and sternum. During inspiration they aid the diaphragm by relaxing. They protect the abdominal 73contents from trauma. They can increase abdominal pressure during micturition, defecation, vomiting and parturition by contracting simultaneously with the diaphragm with a closed glottis.
Station 24
24.1 Identify the structure labelled 1? What are its parts? Which of these parts are mobile?
24.1 1 The duodenum
The first part of the duodenum begins at the pylorus and runs up and backwards (at the transpyloric plane). The second part of the duodenum runs vertically downward in front of the hilum of the right kidney. The bile duct and pancreatic duct enter the duodenum at the ampulla of Vater, with the accessory pancreatic duct nearby. The third part of the duodenum runs horizontally to the left on the subcostal plane, following the lower margin of the head of the pancreas. The fourth part of the duodenum runs up and to the left and ends at the duodenojejunal flexure, which is indicated by the suspensory ligament of Treitz.
Only the first few centimetres of the first part of the duodenum are intraperitoneal (mobile), the rest of the duodenum is retroperitoneal.
24.2 Identify the structure labelled 2? What are its branches?
24.2 2 Coeliac axis
The branches of the coeliac axis are the left gastric artery, the splenic artery, and the common hepatic artery (Figure 1.12).
24.3 Identify the structure labelled 3? Describe its blood supply.
24.3 3 Pancreas
The splenic and superior/inferior pancreaticoduodenal arteries supply the pancreas. The veins are named after the arteries and drain in to the portal system.
24.4 Identify the structures labelled A to D.
24.4 A Common hepatic artery
B Right lobe of the liver
C The spleen
D The splenic artery
24.5 What is the name of the fold of peritoneum that hangs from the greater curvature of the stomach? What blood vessels run in this tissue?
24.5 The greater omentum is a fold of parietal peritoneum that is suspended from the greater curvature of the stomach (Figure 1.15). Its anterior fold hangs over the small intestines before being reflecting back up on itself to reach the transverse colon, and then to the posterior abdominal wall. The right and left gastroepiploic vessels run in and supply the greater omentum. It also carries lymphatics (to the stomach) and autonomic nerves.
24.6 What is the epiploic foramen (foramen of Winslow)? Define its boundaries.
24.6 The epiploic foramen (foramen of Winslow) is the entrance to the lesser sac:
  • anteriorly: border of lesser omentum carrying the bile duct, hepatic artery, and portal vein
  • posteriorly: inferior vena cava
  • superiorly: caudate process of the liver
  • inferiorly: first part of the duodenum.
24.7 What are the topographical relations of the second part of the duodenum?
24.7 The topographical relations of the second part of the duodenum:
  • anteriorly: gallbladder, right lobe of the liver, transverse colon, and small intestine
    zoom view
    Fig. 1.15: The compartments of the abdomen.
    74
  • posteriorly: right renal pelvis
  • superiorly: head of the pancreas, bile ducts draining in to the duodenum
  • inferiorly: ascending colon, right colic flexure, and right lobe of the liver.
24.8 What is the definition and contents of the supracolic compartment?
24.8 The supracolic compartment is the division of the abdomen above the transverse mesocolon, and the infracolic compartment is the division below this level. The supracolic compartment contains the oesophagus, stomach, first part of duodenum, lesser omentum, spleen, liver, and gallbladder.
24.9 What are the contents of the splenorenal (lienorenal) ligament?
24.9 The splenic vessels and the tail of the pancreas.
24.10 What vessels are carried within the gastrosplenic ligament?
24.10 The short gastric and left gastroepiploic vessels.
24.11 Where is the root of the small bowel mesentery attached?
24.11 The left L2 transverse process to the right sacroiliac joint.
Station 25
25.1 Identify structures labelled A to J.
25.1 See Figure 1.16.
A Gallbladder
B Right lobe of liver
C Bile duct
D Inferior vena cava
E Quadrate lobe of liver
F Ligamentum teres hepatis and falciform ligament
75
zoom view
Fig. 1.16: The posterior surface of the liver.
G Left lobe of liver
H Common hepatic artery
I Portal vein
J Caudate lobe of liver
25.2 State the blood supply, lymphatic drainage, and nerve supply of structure A?
25.2 The cystic artery is the main blood supply to the gallbladder (A). This is usually a branch of the right hepatic, but there are several anatomic variants. There are also small vessels that run from the gallbladder to the liver in the gallbladder bed. Lymph drains via a cystic lymph node near the neck of the gallbladder to the hepatic then coeliac nodes. The gallbladder receives sympathetic and parasympathetic supply via the coeliac plexus.
25.3 What epithelium lines structure A?
25.3 The gallbladder (A) is lined by tall columnar epithelium. This epithelium does not secrete mucous.
25.4 What are the boundaries of Calot's triangle and what does the triangle contain?
25.4 Calot's triangle is now conventionally defined as the cystic duct, the common hepatic duct, and the inferior surface of the liver (Figure 1.17) (although the original description in 1891 described the triangle as formed by the cystic duct, the bile duct and the cystic artery). The cystic artery is constantly found in this triangle. Visualising the ducts and arteries is essential before removing the gallbladder to ensure that they are not inadvertently injured. Common bile duct injury, especially those injuries unrecognised at time of surgery, can be disastrous.
25.5 What is the upper limit of the normal diameter of the common bile duct on ultrasound in a 20-year-old?
25.5 The upper limit of the common bile duct diameter on ultrasound in adults is conventionally about 7 mm. In the elderly, and after cholecystectomy, the diameter increases.
25.6 What is the embryological origin of the ligamentum teres hepatis?
25.6 The ligamentum teres hepatis is the remnant of the left fetal umbilical vein.
76
zoom view
Fig. 1.17: Calot's triangle.
25.7 What is the embryological origin of the liver?
25.7 The liver and biliary tree appear in the third/fourth week as hepatic diverticula from the ventral wall of the distal foregut endoderm.
Station 26
26.1 Identify the structures labelled A to F.
26.1 A Rectum
B Internal iliac artery
C Ductus deferens
D Bladder
E Pubic symphysis
F Penis
26.2 Name the segments of the gastrointestinal tract that are immobile.
26.2 Immobile implies retroperitoneal. This includes most of the duodenum, the ascending and descending colon, and the distal two thirds of the rectum.
26.3 What are the branches of the inferior mesenteric artery?
26.3 Branches of the inferior mesenteric are: the left colic artery, branches to the sigmoid, and the superior rectal artery (the continuation of the inferior mesenteric artery) (Figure 1.18).
26.4 What epithelium lines the anal canal?
26.4 Longitudinal folds of simple columnar epithelium line the upper two-thirds. The lower one-third is lined by stratified squamous epithelium, blending with the skin. The dentate line divides these areas.
26.5 Where in the adult gut are the junctions between the embryological foregut and midgut, and the embryological midgut and hindgut?
26.5 The foregut runs from the mouth to the duodenum, as far as the entry of the bile duct (D2). The midgut ends two-thirds of the way along the transverse colon. The hindgut ends two-thirds of the way along the anal canal at the dentate line.
26.6 What is the superior boundary of the left paracolic gutter?
26.6 The paracolic gutters are peritoneal recesses on the posterior abdominal wall, lying lateral respectively to the ascending and descending colon. Their significance is that substances such as bile or pus can travel along their length and settle at sites remote from their origin. The left paracolic gutter is limited superiorly by the phrenicocolic ligament, and inferiorly by the attachment of the lateral limb of the sigmoid mesocolon at the pelvic brim. The right paracolic gutter is superiorly continuous with the hepatorenal pouch (Morrison's pouch), and inferiorly with the pelvis. The right paracolic gutter is continuous with the lesser sac.
77
zoom view
Fig. 1.18: Branches of the inferior mesenteric artery.
26.7 What is the most dependent part of the peritoneal cavity in the supine position?
26.7 The hepatorenal pouch (Morrison's pouch) is the most dependent part of the abdomen and is a common site for accumulation of fluid/pus/blood.
26.8 List the routes via which rectal adenocarcinoma spreads.
26.8 Rectal adenocarcinoma spreads via the following routes:
  • local spread: direct invasion of other structures in the pelvis
  • lymph node spread: regional and then distal
  • blood-borne distal spread: to the liver, lungs, and bone
  • peritoneal spread to other abdominal organs.
Station 27
27.1 What structure passes through the opening labelled A? At what vertebral level does this structure traverse the diaphragm and what accompanies it through this opening?
zoom view
27.1 A Inferior vena cava. Enters the diaphragm at T8, accompanied by the right phrenic nerve.
27.2 What structure passes through the opening labelled B?
27.2 B Oesophageal hiatus
27.3 Identify the structure labelled C. At what vertebral level does it traverse the diaphragm and what accompanies it through this opening?
27.3 C Aorta. Traverses the diaphragm at T12, accompanied by the thoracic duct and azygous/hemiazygous veins.
27.4 Identify the structures labelled D to F
27.4 D Sternum
E Vertebral body
F Spinal cord
27.5 At what vertebral levels do the following traverse the diaphragm:
27.5a the vagus nerves?
27 The vagi accompany the oesophagus through the diaphragm at T10.
27.5b the right phrenic nerve?
27 T8 or T9.
27.5c the left gastric artery?
27 T10.
Table 1.8 summarises the structures traversing the diaphragm.
78
Table 1.8   Structures traversing the diaphragm
Vertebral level
Main structure transmitted
Additional structures transmitted
T8
Inferior vena cava opening
Right phrenic nerve
T10
Oesophageal opening
  • Vagi, branches of the left gastric vessels
  • Lymphatics
T12
Aortic opening
  • Thoracic duct
  • Azygos, and hemiazygos veins
T12 (crura)
Splanchnic nerves
27.6 Where, respectively, do the splanchnic nerves and sympathetic chain traverse the diaphragm?
27.6 The splanchnic nerve traverses the crura of the diaphragm, and the sympathetic chain passes behind the diaphragm deep to the medial arcuate ligament.
27.7 What nerves provide the motor innervation of the diaphragm?
27.7 The phrenic nerves (C3– C5; mnemonic: ‘C3, 4, 5 keeps the diaphragm alive’). These contain motor, sensory, and sympathetic nerve fibers. There is sometimes an accessory phrenic nerve (often a branch of the nerve to the subclavius).
27.8 Describe the course of these nerves.
27.8 The phrenic nerves originate at the C3–C5 vertebral levels. They run vertically downwards over the anterior scalene muscles deep to the prevertebral layer of deep cervical fascia. They enter the thorax by passing over the subclavian arteries. The right phrenic nerve passes along the right side of the brachiocephalic artery, posterior to the subclavian vein, and then crosses anterior to the root of the right lung, over the pericardium of the right atrium, and then leaves the thorax by passing through the caval opening in the diaphragm. The left phrenic nerve travels lateral to the left subclavian artery and passes in front of the root of the left lung and over the pericardium of the left ventricle to pierce the muscular diaphragm to supply the peritoneum on its under surface.
27.9 What four components contribute to the embryological origin of the diaphragm?
27.9 The central tendon is formed by the septum transversum. The peripheral rim comes from the body wall. There are also contributions from the oesophageal mesentery and the pleuroperitoneal membranes.
27.10 Name two common types of acquired diaphragmatic hernias.
27.10 The most common acquired hernias are termed ‘sliding’ and ‘rolling’. Sliding hernias consist of the projection of the upper part of the stomach through the diaphragm in to the chest when the patient lies or bends. It predisposes to gastroesophageal reflux due to incompetence of the lower oesophageal sphincter. A rolling hernia describes the fundus of the stomach rolling up through the diaphragm in front of the oesophagus. Patients with this condition do not experience reflux.
27.11 Name two common types of congenital diaphragmatic hernias.
27.11 Herniation may occur posteriorly through the foramen of Bochdalek. This is the most common form of congenital diaphragmatic hernia and is due to 79developmental failure of the posterolateral diaphragmatic foramina. It most commonly occurs on the left side. A hernia through the foramen of Morgagni is located anteromedially, between the costal and sternal origins of the diaphragm. Other forms of congenital hernia include a deficiency of the central tendon, or a large oesophageal hiatus.
Station 28
28.1 At what vertebral levels do the kidneys lie?
28.1 T12–L3
28.2 Identify the structures labelled A, B and C.
zoom view
28.2 A Minor calyx of the left kidney
B Major calyx of the left kidney
C Left renal pelvis
28.3 What are the three natural narrowings in D?
28.3 D The right ureter. The three narrowest parts of the ureter are (i) the pelviureteric junction, (ii) where the ureter crosses the pelvic brim and (iii) the vesicoureteric junction.
28.4 What is the blood supply to D?
28.4 Like most long tubes the ureter has a segmental blood supply from vessels that it passes close to: the aorta, the renal artery, the testicular/ovarian artery, the internal iliac artery and the inferior vesical vessels.
28.5 What is the topographical relationship between the renal artery and the renal vein?
28.5 The renal vein is anterior to the renal artery, which is anterior to the renal pelvis.
28.6 Can you ligate the left renal vein without serious consequences and if so, why?
28.6 Yes, this is often cut during open aortic aneurysm repair (remember that the left renal vein reaches across the aorta to reach the inferior vena cava). This is possible due to sufficient collateral drainage via the adrenal and inferior phrenic veins.
28.7 Name the retroperitoneal structures of the abdomen.
28.7 Pancreas, kidneys, ureters, adrenals, aorta, para-aortic lymph nodes, lumbar sympathetic chain ascending/descending colon, the duodenum beyond the first few centimetres, inferior vena cava, rectum. The spleen is not retroperitoneal, a common incorrect answer given in the exam!
28.8 From what embryological structures are the kidneys and ureters derived?
28.8 The distal part of the pronephros develops in to the mesonephric duct. A diverticulum of the lower end of the mesonephric duct develops in to the metanephric duct. Tissue overlying the end of this duct develops in to the kidneys (metanephros), whilst the duct itself develops in to the collecting tubules, calyces, pelvis, and ureter.
28.9 What arrests the ascent of a horseshoe kidney from the pelvis during development?
28.9 The inferior mesenteric artery.
Station 29
29.1 Identify the structures labelled A, B, C.
29.1 A The rectum
B The pubic symphysis
C The bladder
29.2 Regarding structure D:
29.2a what is it?
29D The uterus
29.2b name the various parts of structure D.
8029.2b See Figure 1.19.
Fundus: lies above the entrance of the fallopian tubes.
Body: the part that lies below the fallopian tubes.
Cervix: projects in to the vagina.
Cavity: hollow space within the uterus.
Internal os: communication in to the uterus.
External os: communication in to the vagina.
29.2c what is the blood supply to structure D?
29.2 The uterine artery (internal iliac artery), and the ovarian artery (abdominal aorta).
29.2d what are different positions in which it can lie?
29 The most common position is anteverted, where the long axis of the uterus is bent forwards. The uterus is also usually anteflexed (bent forward at the level of the internal os). It follows that the uterus can also be retroverted and retroflexed.
29.3 Regarding structure E:
29.3a what is it?
29E The cavity of the vagina.
29.3b what are its topographical relations?
29 The relations of the vagina are important to know for the purposes of vaginal examination. Anteriorly lies the bladder, urethra, and symphysis pubis. Posteriorly lies the Pouch of Douglas (in which fluid and bowel may be felt) and the rectum. Laterally are the levator ani muscles, pelvic fascia and the ureters. At the apex of the vagina is the cervix.
29.3c what is the blood supply?
29.3C the vagina is supplied by the vaginal, uterine, internal pudendal, and middle rectal arteries (all branches of the internal iliac artery). It is drained by the vaginal vein (internal iliac vein).
29.3d what epithelium lines it?
29 the vagina and vaginal cervix are lined by stratified squamous epithelium. The uterine cervix is lined by tall columnar cells (which secrete the cervical mucus plug). The uterus is lined by cuboidal ciliated cells forming tubular glands.
zoom view
Fig. 1.19: The uterus.
29.4 Name the parts of the fallopian tube.
8129.4 See Figure 1.19. The infundibulum is the most lateral part and opens in to the peritoneal cavity via the ostium. This joins the wide ampulla, becoming the narrow isthmus before piercing the uterine wall.
29.5 Where are the Bartholin's glands and what is their function?
29.5 These are two glands located in the labium magus. They secrete mucus to provide vaginal lubrication. They can become obstructed, forming Bartholin's cysts, which are prone to infection.
29.6 What are the broad ligaments of the uterus?
29.6 The broad ligaments are folds of peritoneum that connect the lateral sides of the uterus to the pelvic sidewalls. The fallopian tubes lie in the free edge of the broad ligaments and open into the cornu of the uterus. The ligaments also carry the ovary (attached by the mesovarium to the posterior aspect of the uterus), the round ligament, the ovarian ligament, the uterine vessels and their branches, lymphatics and nerves.
29.7 What is the round ligament of the uterus?
29.7 The round ligaments maintain anteversion of the uterus during pregnancy. They are attached to the uterine horns (where the uterus and the fallopian tubes meet) and travel in the anterior layer of the broad ligament to leave the pelvis via the internal inguinal ring. They then pass through the inguinal canal to attach to the labium majora.
29.8 Name three other ligaments that support the uterus and vagina.
29.8 The cardinal (or cervical) ligaments pass laterally from the cervix and upper vagina to the sidewalls of the pelvis. The uterosacral ligaments pass backwards from the posterolateral cervix and from the lateral vaginal fornices to attach to the periosteum in front of the sacroiliac joints and the lateral part of the sacrum. The pubocervical fascia extends from the cardinal ligament to the pubis, either side of the bladder (acting as a sling).
Station 30
30.1 Identify the structures labelled A to H.
30.1 A Right psoas major muscle
B Right external iliac artery
C Left iliacus muscle
D Left ureter
E Superior hypogastric plexus
F Left internal iliac artery
G Left external iliac vein
H Left common femoral artery
30.2 What are the boundaries of the urogenital triangle? What are its contents in the male?
30.2 The anterior vertex is the pubic symphysis and the two other vertices are the ischiopubic rami of the pelvic bone. Its contents in males are the penis and scrotum. In females, the triangle contains the external genitalia, the urethra, and the vagina (Figure 1.11).
30.3 What is the blood supply of the bladder?
30.3 The bladder is supplied by the superior and inferior vesical arteries (internal iliac artery). It drains to the vesical venous plexus (iliac vein).
30.4 Describe the innervation of the bladder.
8230.4 The bladder is innervated by sympathetic and parasympathetic fibres. The sympathetic fibres originate at L1–2. These fibres inhibit contraction of the detrusor and stimulate tonic contraction of the internal urethral sphincter. The parasympathetic preganglionic fibres originate at S2–4 as the pelvic splanchnic nerves, and synapse with postganglionic neurones in the bladder wall. Most of the afferent impulses (initiated by stretch) travel through these fibres. The parasympathetic fibres stimulate contraction of the detrusor and inhibit the internal urethral sphincter. In the continent individual there are fibres originating in the cerebral cortex that inhibit the micturition reflex until it is required.
30.5 What layers, from skin inwards, does a needle pass through when aspirating a hydrocele?
30.5 A mnemonic for the layers of scrotum is: Some Damn Englishman Called It Testes: Skin, Dartos, External spermatic fascia, Cremaster, Internal spermatic fascia, Tunica vaginalis, Testis.
30.6 What is the lymphatic drainage of the testes?
30.6 The lymph drainage of the testis and epididymis is via the spermatic cord and ends at the para-aortic lymph nodes at L1. The scrotum, in contrast, drains to the superficial inguinal lymph nodes.
30.7 What is the origin of the left testicular artery?
30.7 The testicular arteries both branch off from the aorta at L2, they then travel in the inguinal canal to reach the testes. Note that it is the left testicular vein not artery that drains in to the left renal vein, whereas the right joins the inferior vena cava.
30.8 Name the parts of the male urethra. What is its approximate length in the adult male?
30.8 The male urethra is approximately 20 cm long. The prostatic urethra is the area surrounded by prostate. It is about 3 cm and has a central elevated area called the urethral crest, with a depressed area either side termed the prostatic sinuses (into which prostatic ducts empty). The crest has a short tract – the verumontanum – into which opens the prostatic utricle. The ejaculatory ducts open either side of the utricle. The membranous urethra is about 2 cm and traverses the external sphincter urethrae and perineal membrane. The spongy urethra is the area within the corpus spongiosum of the penis.
30.9 Is ejaculation mediated by the sympathetic or parasympathetic nervous system?
30.9 Ejaculation is a sympathetic process, whereas erection is parasympathetic. A useful mnemonic is Point = Parasympathetic, Shoot = Sympathetic.
30.10 Which epithelium lines the prostatic urethra?
30.10 Transitional cell epithelium.
Station 31
31.1 Identify the structures labelled A to I.
31.1 A Piriformis
B Coccygeus and sacrospinous ligament
C Obturator internus
D Internal pudendal artery
E Left common iliac
F L5 vertebral body
G Obturator nerve
H Anterior trunk of internal iliac
I External iliac artery
31.2 What is the origin and insertion of the muscle labelled A?
8331.2 The piriformis (A) originates from the anterior surface of the lateral mass of the sacrum. Its tendon traverses the greater sciatic foramen to insert in to the upper border of the greater trochanter. It is innervated by the nerve to the piriformis (L5–S2) and is an external rotator of the hip.
31.3 What are the origin, insertion, action and nerve supply of C?
31.3 The obturator internus (C) originates from the inner surface of the anterolateral wall of the pelvis and the obturator membrane. It inserts in to the greater trochanter. The muscle is innervated by the nerve to obturator internus (sacral plexus), and is a lateral rotator of the femur.
31.4 From which nerve roots does the lumbar plexus arise?
31.4 L1–L4.
31.5 What branches of the lumbar plexus emerge from the lateral border of the psoas?
31.5 All of the branches of the lumbar plexus arise from the lateral border of the psoas (iliohypogastric nerve, ilioinguinal nerve, lateral cutaneous nerve of the thigh, femoral nerve) except for the genitofemoral nerve (anterior aspect), and obturator nerve (medial border).
31.6 What are the root values for the femoral nerve?
31.6 L2–L4.
31.7 What are the root values for the sacral plexus?
31.7 L4–S4.
31.8 Describe the course and relations of the right and left lumbar sympathetic chains.
31.8 The lumbar sympathetic chain is a continuation of the thoracic chain as it passes under the medial arcuate ligament of the diaphragm and travels on the lumbar vertebral bodies. On the left, it runs posterolateral to the aorta, on the right underneath the inferior vena cava. They converge on the coccyx at a structure known as the ganglion impar.
31.9 What are the indications for lumbar sympathectomy?
31.9 Lumbar sympathectomy is performed for patients with non-reconstructible arterial disease or vasospastic conditions of the lower limbs. It involves excision of a variable number of the L1–L2 ganglia to denervate the sympathetic supply to the leg and hence increase its blood supply.
31.10 From where does the parasympathetic supply of the abdomen originate?
31.10 The vagus nerve is the main parasympathetic nerve of the abdominal organs. It supplies the gastrointestinal tract as far as the proximal transverse colon. There is also parasympathetic supply from S2–S4 in the form of the pelvic splanchnic nerves. These supply the distal transverse colon as well as the rectum, internal anal sphincter, bladder wall, internal vesicle sphincter, penis and clitoris.
Station 32
32.1 What type of study is this?
32.1 The image displays a barium meal.
32.2 Identify the parts of the stomach labelled A, B, C, D, F and G.
32.2 A Lesser curve
B Pylorus
C Antrum
D Fundus
F Greater curve
G Body
32.3 What do the vertical lines at point E indicate?
8432.3 E Stomach rugae. These are longitudinal folds in the mucous membrane of the stomach that flatten out when the stomach distends.
32.4 Describe the arterial supply of the stomach.
32.4 All of the arteries that supply the stomach are derived from the coeliac axis (Figure 1.12). The left gastric is the only direct branch of the axis, and passes upwards and to the left to reach the oesophagus (which it also supplies) before descending along the lesser curvature. The right gastric arises from the common hepatic artery and runs up the lesser curvature. The short gastric arises from the splenic artery at the hilum of the spleen and travels in the gastrosplenic ligament to supply the upper greater curvature. The left gastroepiploic also originates from the splenic artery and travels in the greater omentum to supply the greater curvature. The right gastroepiploic is a branch of the gastroduodenal artery (which in turn comes off the hepatic artery). It supplies the inferior greater curvature.
32.5 Describe the innervation of the stomach.
zoom view
32.5 Sympathetic fibres arise from the coeliac plexus. They carry afferent pain fibres, cause reduction in secretory and motor function, and cause constriction of the pylorus. The parasympathetic fibres arise from the vagus nerves. Parasympathetic fibres are secretomotor to the stomach and cause relaxation of the pylorus. The left vagus nerve forms the anterior vagal trunk and enters the abdomen on the anterior surface of the oesophagus. It gives off branches to the anterior stomach wall, the liver, and the pylorus of the stomach. The posterior vagal trunk enters the abdomen on the posterior surface of the oesophagus and supplies the posterior wall of the stomach. The posterior trunk also gives off branches to the coeliac and superior mesenteric plexuses to supply the pancreas and the colon as far as the splenic flexure.
32.6 What are the consequences of highly selective vagotomy?
32.6 Highly selective vagotomy is division of those branches of the anterior and posterior vagus nerves that supply the acid-secreting body of the stomach. The nerve of Latarjet is preserved, maintaining function of the pyloric antrum.
32.7 What are the anterior and posterior relations of the stomach?
32.7 The anterior relations of the stomach are: the anterior abdominal wall, the left costal margin, the left pleura and lung, the diaphragm, and the left lobe of the liver. The posterior relations are: the lesser sac, the spleen and splenic artery, the pancreas, the left suprarenal gland, the left kidney, and transverse colon and mesocolon.
32.8 What anatomical landmarks demarcate the duodenum from the stomach?
32.8 The junction of pylorus of the stomach from the duodenum is marked by an external constriction and the constant vein of Mayo.
32.9 Describe the cardiac sphincter of the stomach.
32.9 The cardiac sphincter is a physiological rather than anatomical sphincter. Tonic constriction of the circular layer of smooth muscle at this level prevents gastric contents from regurgitating upwards. It relaxes ahead of peristaltic waves caused by the swallowing of food. There are mucosal folds at the junction which act as valves, and the right crus of the diaphragm also exerts external pressure.
32.10 Describe the lymphatic drainage of the stomach.
32.10 The lymph drainage follows the arterial supply. The superior two-thirds of the stomach drain along the left and right gastric vessels. The right greater curvature of the stomach drains along the right gastroepiploic arteries to the subpyloric nodes. The left part of the greater curve drains alongside the short gastric and splenic vessels to the suprapancreatic nodes. All lymph eventually passes to the coeliac nodes.
85
Station 33
33.1 Identify the structures labelled A to E.
33.1 See Figure 1.20 demonstrating the vessels within the splenic hilum.
A Portal vein origin (confluence of the splenic and superior mesenteric veins)
B Splenic vein
C Spleen
D Origin of the coeliac axis
E Splenic artery
Note how the density of the splenic artery mimics that of the aorta in an arterial phase scan. This knowledge helps you identify that the vessel here labeled ‘E’ is an artery and not a vein.
33.2 Which pathogen causes upper respiratory tract infection and also predisposes the individual to splenic rupture?
33.2 The Epstein–Barr virus causes glandular fever and is associated with splenomegaly.
33.3 In what direction does a spleen enlarge and why?
33.3 The diaphragm ensures that the spleen enlarges downwards, but the left colic flexure and phrenicocolic ligament direct the spleen medially. The notched anterior border of the spleen is palpable as it projects below the costal margin.
33.4 What organs are in direct contact with the spleen?
33.4 Posteriorly: the diaphragm (behind which is the pleura, left lung and 9–11th ribs).
Anteriorly: the stomach, the tail of the pancreas.
Inferiorly: the splenic flexure of colon.
Medially: the left kidney.
33.5 Is the spleen intraperitoneal or retroperitoneal?
33.5 The spleen is intraperitoneal (and hence mobile).
33.6 What splenic ligaments must be cut during splenectomy?
33.6 Splenorenal (or lienorenal), gastrosplenic, splenocolic, and splenophrenic ligaments (or we also accept: ‘all of them’).
33.7 What are the branches of the splenic artery?
33.7 The splenic artery originates from the coeliac axis (see Figure 1.12).
zoom view
Fig. 1.20: The spleen.
It runs a tortuous course along the upper border of the pancreas giving off multiple 86branches to the pancreas (the largest of which is the arteria pancreatica magna), the short gastric artery, the left gastroepiploic artery, and the posterior gastric artery.
33.8 Describe the functions of the spleen.
33.8 In the foetus the spleen has haematopoietic properties up until the 5th month of gestation. In the adult it has immune functions (via humoral and cell-mediated pathways) and filters red blood cells.
33.9 What embryological structure does the spleen develop from?
33.9 The spleen develops as multiple thickenings of mesenchyme in the dorsal mesentery. In most people these masses fuse, although approximately 10% of people have more than one spleen.
33.10 What is the innervation of the adrenal glands?
33.10 The medulla of the adrenal gland receives preganglionic sympathetic fibres from the greater splanchnic nerve. It can be considered a specialised sympathetic ganglion, except that it releases its adrenergic secretions directly into the bloodstream. The cortex is regulated by hormones from the pituitary and hypothalamus, well as the renin–angiotensin system.
33.11 Name the different macroscopic regions of the adrenal gland and their respective functions.
33.11 The adrenal glands have an outer yellow cortex, and a dark brown inner medulla. The medulla secretes adrenaline and noradrenaline in response to sympathetic stimulation. The cortex produces corticosteroid hormones and is further divided in to a zona glomerulosa (producing mineralocorticoids), zona fasciculata (producing cortisol), and zona reticularis (producing androgens).
Station 34
34.1 Identify the structures labelled A to E.
34.1 A Common hepatic artery
B Common hepatic duct
C Portal vein
D Inferior vena cava
E Tail of the pancreas
34.2 What anatomical layer, related to the liver, may help contain hepatic bleeding?
34.2 The liver has a connective tissue layer, Glisson's capsule, which covers its surface and invests its blood vessels. Bleeding from the liver can be contained within this capsule, although this may rupture and blood can leak in to the peritoneal cavity.
34.3 Describe the functional divisions of the liver.
34.3 The gross liver can be divided into left and right segments by the attachments of the falciform ligament, ligamentum teres, and ligamentum venosum. However, it is functionally divided by a plane that passes through the gallbladder and the inferior vena cava fossae. These functional lobes have separate blood supply and biliary drainage, and can thus be resected separately. In the Couinaud or ‘French’ system, these functional lobes are divided into a total of eight sub-segments.
34.4 Describe the innervation of the liver.
34.4 The liver receives parasympathetic and sympathetic supply from the coeliac plexus. The anterior vagal trunk also gives off a branch to the liver.
34.5 What are the contents and attachments of the falciform ligament?
34.5 The falciform ligament is a two-layered fold of peritoneum that contains the ligamentum teres (the remnant of the umbilical vein, see Figure 1.16). It attaches the umbilicus to the anterior surface of the liver before splitting in to two layers on 87its posterior surface. The right layer forms the upper coronary ligament and the left the upper triangular ligament.
34.6 What is the ligamentum venosum?
34.6 The ligamentum venosum is the remnant of the foetal ductus venosus, which shunts blood from the umbilical vein to the inferior vena cava. It adheres to the left branch of the portal vein and travels in a fissure on the visceral surface of the liver to attach superiorly to the inferior vena cava.
34.7 How are the portal vein, hepatic artery, and bile duct related to each other in the free edge of the lesser omentum?
34.7 The bile duct lies anterior and to the right, the hepatic artery lies anterior and to the left, and the portal vein lies posteriorly.
Station 35
35.1 Identify the structures labelled A, B, D, E and F.
35.1 A Wing of the ileum
B Iliopectineal line (with the ischial spine behind it)
D Ala of sacrum
E Coccyx
F Superior pubic ramus
35.2 Name the foramen indicated by C. What structures pass through here?
35.2 C Obturator foramen. The obturator nerve and vessels pass through this space.
35.3 What type of joint is the sacroiliac joint?
35.3 The sacroiliac joint is a synovial plane joint.
35.4 What type of joint is the sacrococcygeal joint?
35.4 The sacrococcygeal joint is a secondary cartilaginous joint.
35.5 What are the boundaries of the pelvic inlet?
35.5 The pelvic inlet (or brim) is bounded anteriorly by the symphysis pubis, laterally by the iliopectineal lines, and posteriorly by the sacral promontory.
35.6 Define the false pelvis.
35.6 The false pelvis is a space within the abdomen bounded posteriorly by the lumbar vertebrae, laterally by the iliac fossae and iliacus muscles, inferiorly by the pelvic inlet, and anteriorly by the anterior abdominal wall.
35.7 What are contents of the pudendal (Alcock's) canal?
35.7 The pudendal (Alcock's) canal is a fascial space in the lateral wall of the ischioanal fossa containing the pudendal nerve and internal pudendal vessels.
35.8 What lies medial to the pudendal canal, and what lies laterally?
35.8 Medial is the ischioanal fossa, and laterally are the obturator internus and ischial tuberosity.
35.9 What are the clinical consequences of superior hypogastric plexus damage?
35.9 The superior hypogastric plexus is a continuation of the aortic plexus with contributions from the third and fourth lumbar sympathetic ganglia. It lies on the promontory of the sacrum and may be damaged during operations in the pelvis, e.g. open aortic aneurysm repair. Injury results in erectile dysfunction in males and bladder dysfunction in females.
35.10 What are the branches of the pudendal nerve?
35.10 The pudendal nerve is a branch of the sacral plexus. It leaves the pelvis via the greater sciatic foramen, and enters the perineum through the lesser sciatic foramen. Its branches are:
  • inferior rectal nerve: supplies the external anal sphincter, the mucous membrane of the lower half of the anal canal, and the perianal skin.
  • dorsal nerve of the penis/clitoris
    88
  • perineal nerve: has a superficial branch that supplies the skin of the posterior scrotum/labia majora, and a deep branch supplying the muscles of the urogenital triangle.
Station 36
36.1 Identify the structures labelled A to D.
36.1 See Figure 1.21.
A Corpora cavernosa of the penis
B Corporum spongiosum of the penis
C Right testis
D Epididymis
36.2 Trace the pathway of semen from the seminferous tubules to the terminal urethra.
36.2 The seminiferous tubules are located in the lobules of the testis. Each testis has 200–300 lobules and each lobule contains one to three coiled tubules. The tubules drain in to a plexus termed the rete testis, and thence in to efferent ductules. This pierces the tunica albuginea at the upper testis and passes into the head of the epididymis. The efferent ductules coalesce upon a single much coiled tube, which forms the body and tail of the epididymis. This is a 6 metre long tube that allows for storage and maturation of spermatozoa. The tube continues from the tail as the vas deferens, which travels through the inguinal canal in the spermatic cord. It emerges from the deep inguinal ring and then travels downwards and backwards on the lateral wall of the pelvis (intersecting the ureter at the ischial spine), before running medially and downwards on the posterior bladder. The final part of the vas forms an ampulla before combining with the duct of the seminal vesicle to form the ejaculatory duct. The two ejaculatory ducts pierce the posterior surface of the prostate to open in to the prostatic urethra either side of the prostatic utricle.
36.3 What is the blood supply to the testis?
36.3 The testis and epididymis are supplied by the testicular arteries, which come off the aorta at L2 and pass through the inguinal canal. The venous drainage is the testicular veins, via the pampiniform plexus. The right vein drains in to the inferior vena cava, and the left to the left renal vein.
zoom view
Fig. 1.21: The testis.
36.4 What are the functions of the Leydig and Sertoli cells of the testis?
8936.4 The Sertoli cells form the epithelium of the seminiferous tubules. Their function is to nurture developing sperm cells through spermatogenesis. The cell is activated by follicle-stimulating hormone (FSH) and in turn secretes a number of hormones and proteins. Leydig cells are interstitial cells that produce androgens in response to luteinizing hormone (LH).
36.5 List the possible locations for an undescended or incompletely descended testis.
36.5 The testes develop on the posterior abdominal wall and descend during the latter stages of pregnancy, explaining the distant origin of their vascular and nervous supply. If descent is incomplete they may be found at any point along this path: the abdomen, the inguinal canal, superficial ring, or high in the scrotum.
36.6 What is a varicocele? On which side are they most common?
36.6 A varicocele is a dilatation of the pampiniform plexus. The majority of varicoceles occur on the left side and this is probably due to the testicular vein on the left side entering the left renal vein rather than the inferior vena cava.
36.7 What is a hydrocele? How may peritoneal fluid enter the scrotum in some individuals?
36.7 A hydrocele is fluid within the tunica vaginalis, and may be associated with a patent processus vaginalis. The processus vaginalis is an embryological outpouching of peritoneum in to the scrotum, and surrounds the front and sides of the testis. The processus normally closes soon after birth. A persistent patent processus vaginalis allows for fluid and peritoneal contents to travel in to the scrotum. Persistency is more common on the right side. Hydroceles can also be caused by inflammation of the testis without communication to the peritoneal cavity.
36.8 What is the clinical significance and embryological origin of the appendix testis (hydatid of Morgagni)?
36.8 Both the testis and epididymis have appendages. The former is derived from the paramesonephric (Müllerian) ducts, and the latter from the mesonephric tubules. The testicular appendage is called the hydatid of Morgagni, and is present in most individuals. As these appendages exist on stalked bodies, they may both undergo torsion. This, in itself, is not a problem, but the clinical presentation may mimic torsion of the testes. Clinically a ‘blue dot’ is visible through the scrotal skin.
36.9 What is Fournier's gangrene?
36.9 Fournier's gangrene is a necrotising infection of the perineum and associated structures. The condition is associated with diabetes and immunosuppression, and a mixture of both aerobic and anaerobic organisms is usually responsible. It can be rapidly spreading and requires urgent admission, antibiotics and debridement.
Station 37
37.1 Identify the structures labelled A to E.
37.1 A Bladder
B Pubic symphysis
C Penis
D Prostate
E Rectum
37.2 What does ‘BPH’ stand for?
37.2 BPH stands for benign prostatic hyperplasia (not hypertrophy). Hyperplasia is the abnormal but benign proliferation of cells of the same type. It is often a response to a specific external stimulus. Hypertrophy, in contrast, is a benign increase in the size of the cells.
37.3 Which zone of the prostate becomes enlarged in BPH?
9037.3 The transition zone is affected in benign prostatic hyperplasia, compressing the surrounding peripheral zone. The zones of the prostate are outlined in Table 1.9.
37.4 In which zone of the prostate does cancer most commonly occur?
37.4 Prostate carcinoma usually occurs in the peripheral zone.
37.5 What is the arterial supply to the prostate?
37.5 The prostate is supplied by prostatic branches of the inferior vesical arteries (both ultimately originating from the internal iliac artery).
37.6 By what route may prostatic metastases travel to the spine?
37.6 Cancer can spread via the prostatic venous drainage. The veins of the prostate form a venous plexus in front of the vertebral bodies, outside of the prostatic capsule, before draining in to the internal iliac veins. There are connections between the prostatic venous plexus and the vertebral veins. The veins in the plexus do not have valves, and therefore during periods of raised abdominal pressure (e.g. coughing or straining) the direction of flow may be directed in to the vertebrals, allowing seeding to the vertebral bodies.
37.7 What is the seminal colliculus (verumontanum)?
37.7 The seminal colliculus or verumontanum is an elevation of the posterior wall of the prostatic urethra in the middle of the urethral crest. At its margins open the prostatic utricle and the ejaculatory and prostatic ducts. During transurethral resection of the prostate the surgeon works above this level to avoid damage to the urethral sphincter.
37.8 What is the prostatic utricle? What is its embryological origin?
37.8 The prostatic utricle is a blind ending pouch on the posterior wall of the prostatic urethra at the apex of the urethral crest, on the seminal colliculus (verumontanum). It is derived from the paramesonephric (Müllerian) duct, which in the female becomes the fallopian tubes, uterus and upper vagina.
37.9 Describe the structure of the body of the penis. In which part does the urethra run?
37.9 The body of the penis comprises two dorsal corpora cavernosa and a ventral corpus spongiosum. The corpus spongiosum expands distally to form the glans penis. The penis is enclosed in Buck's fascia and has a foreskin that is connected to the glans penis by the frenulum. The urethra travels within the corpus spongiosum.
37.10 What is the arterial supply to the penis?
37.10 The arterial supply is the internal pudendal artery (internal iliac). The corpus spongiosum is supplied by the artery of the bulb, the corpora cavernosa are supplied by the deep arteries of the penis, and the sheath of the corpora cavernosa are supplied by the dorsal artery of the penis.
Table 1.9   The zones of the prostate
Central
Wedge shaped region that surrounds the ejaculatory ducts (extends from bladder base to the veru)
< 5% of prostatic cancer
Peripheral
Posterolateral part of the prostate, surrounding the central zone
70% of prostatic cancer
Transition
Surrounds the prostatic urethra proximal to the veru
  • 20% of prostatic cancer
  • Benign prostatic hyperplasia arises here
Anterior fibromuscular zone (or stroma)
Anterior region composed of fibrous and muscular tissue
Not affected by cancer
91The veins of the penis drain to the internal pudendal vein.
Station 38
38.1 Identify the structures labelled A to E.
38.1 A Right ureter
B Bladder
C Uterus
D Rectum
E Levator ani
38.2 What is the blood supply to the ovaries?
38.2 The ovary is supplied by the ovarian artery, which branches from the abdominal aorta at L1. The corresponding veins drain in to the inferior vena cava on the right, and the left renal vein on the left. This is identical to the blood supply of the testicles.
38.3 What is the lymphatic drainage of the ovaries?
38.3 The lymph drainage follows the arterial supply into paraaortic nodes at the L1 level.
38.4 In which structure do the vessels and nerves of the ovary run?
38.4 The neurovascular and lymphatic structures travel via the suspensory ligament (also known as the infundibulopelvic ligament) of the ovary and thence through the mesovarium, to enter the hilum of the ovary.
38.5 Which ligaments are connected to the ovary?
38.5 The ovary is attached to the following ligaments:
  • the mesovarium: attaches the ovary to the broad ligament of the uterus
  • the round ligament of the ovary: connects the ovary to the lateral margin of the uterus.
  • the suspensory ligament of the ovary: connects the mesovarium to the lateral wall of the pelvis.
38.6 What is the name of the layer which envelopes the ovary?
38.6 The ovary (and testis) is surrounded by the tunica albuginea.
38.7 Which tumour marker rises in ovarian cancer?
38.7 Ca125.
Station 39
39.1 Identify structures A to D.
39.1 A Left ureter
B Psoas major
C Superior hypogastric plexus
D Left hypogastric nerve
39.2 Where is the lumbar plexus located?
39.2 The lumbar plexus is formed in the psoas muscle from the anterior rami of L1–L4.
39.3 What does the genitofemoral nerve supply?
39.3 The genitofemoral nerve (L1, L2) supplies the cremaster muscle of the scrotum and scrotal skin (genital branch) and the skin of the anterior thigh (femoral branch) (Table 1.10).
92
Table 1.10   The nerves of the lumbar plexus
Nerve
Motor supply
Sensory supply
Iliohypogastric (T12–L1)
  • External oblique
  • Internal oblique
  • Transversus abdominis
  • Lower abdominal wall
  • Buttock
Ilioinguinal (L1)
  • External oblique
  • Internal oblique
  • Transversus abdominis
  • Upper medial thigh
  • Root of penis and scrotum
  • Mons pubis
  • Labia majora
Genitofemoral (L1–L2)
Cremaster muscle
Anterior thigh
Lateral cutaneous nerve of the thigh (L2–L3)
None
Anterior and lateral thigh
Obturator (L2–L4)
  • Obturator externus
  • Gracilis, adductor brevis
  • Adductor longus
  • Pectineus
  • Adductor magnus (adductor part)
Medial thigh
Femoral (L2–L4)
  • Iliacus, sartorius
  • Pectineus
  • Quadriceps femoris
  • Anterior thigh
  • Medial leg and foot (saphenous branch)
Muscular branches
  • Psoas
  • Quadratus lumborum
  • Iliacus
None
39.4 What does the ilioinguinal nerve supply?
39.4 The ilioinguinal nerve supplies the external oblique, the internal oblique, transversus abdominis, the skin of the upper medial thigh, the base of the penis and scrotum, the mons pubis and the labia majora.
39.5 Describe the cremasteric reflex.
39.5 The cremasteric reflex can be elicited by lightly stroking the medial thigh. The afferent limb of the reflex is the femoral branch of the genitofemoral nerve. This causes contraction of the cremaster via the efferent limb, which is the genital branch of the genitofemoral nerve. The function of the cremasteric reflex is to raise the testis for warmth and protection. It may be absent in testicular torsion.
39.6 Where does the sympathetic trunk enter the abdomen?
zoom view
39.6 The sympathetic trunk enters the abdomen behind the medial arcuate ligament.